Top 20 Problem Solving Interview Questions (Example Answers Included)

Mike Simpson 0 Comments

problem solving type of question

By Mike Simpson

When candidates prepare for interviews, they usually focus on highlighting their leadership, communication, teamwork, and similar crucial soft skills . However, not everyone gets ready for problem-solving interview questions. And that can be a big mistake.

Problem-solving is relevant to nearly any job on the planet. Yes, it’s more prevalent in certain industries, but it’s helpful almost everywhere.

Regardless of the role you want to land, you may be asked to provide problem-solving examples or describe how you would deal with specific situations. That’s why being ready to showcase your problem-solving skills is so vital.

If you aren’t sure who to tackle problem-solving questions, don’t worry, we have your back. Come with us as we explore this exciting part of the interview process, as well as some problem-solving interview questions and example answers.

What Is Problem-Solving?

When you’re trying to land a position, there’s a good chance you’ll face some problem-solving interview questions. But what exactly is problem-solving? And why is it so important to hiring managers?

Well, the good folks at Merriam-Webster define problem-solving as “the process or act of finding a solution to a problem.” While that may seem like common sense, there’s a critical part to that definition that should catch your eye.

What part is that? The word “process.”

In the end, problem-solving is an activity. It’s your ability to take appropriate steps to find answers, determine how to proceed, or otherwise overcome the challenge.

Being great at it usually means having a range of helpful problem-solving skills and traits. Research, diligence, patience, attention-to-detail , collaboration… they can all play a role. So can analytical thinking , creativity, and open-mindedness.

But why do hiring managers worry about your problem-solving skills? Well, mainly, because every job comes with its fair share of problems.

While problem-solving is relevant to scientific, technical, legal, medical, and a whole slew of other careers. It helps you overcome challenges and deal with the unexpected. It plays a role in troubleshooting and innovation. That’s why it matters to hiring managers.

How to Answer Problem-Solving Interview Questions

Okay, before we get to our examples, let’s take a quick second to talk about strategy. Knowing how to answer problem-solving interview questions is crucial. Why? Because the hiring manager might ask you something that you don’t anticipate.

Problem-solving interview questions are all about seeing how you think. As a result, they can be a bit… unconventional.

These aren’t your run-of-the-mill job interview questions . Instead, they are tricky behavioral interview questions . After all, the goal is to find out how you approach problem-solving, so most are going to feature scenarios, brainteasers, or something similar.

So, having a great strategy means knowing how to deal with behavioral questions. Luckily, there are a couple of tools that can help.

First, when it comes to the classic approach to behavioral interview questions, look no further than the STAR Method . With the STAR method, you learn how to turn your answers into captivating stories. This makes your responses tons more engaging, ensuring you keep the hiring manager’s attention from beginning to end.

Now, should you stop with the STAR Method? Of course not. If you want to take your answers to the next level, spend some time with the Tailoring Method , too.

With the Tailoring Method, it’s all about relevance. So, if you get a chance to choose an example that demonstrates your problem-solving skills, this is really the way to go.

We also wanted to let you know that we created an amazing free cheat sheet that will give you word-for-word answers for some of the toughest interview questions you are going to face in your upcoming interview. After all, hiring managers will often ask you more generalized interview questions!

Click below to get your free PDF now:

Get Our Job Interview Questions & Answers Cheat Sheet!

FREE BONUS PDF CHEAT SHEET: Get our " Job Interview Questions & Answers PDF Cheat Sheet " that gives you " word-word sample answers to the most common job interview questions you'll face at your next interview .

CLICK HERE TO GET THE JOB INTERVIEW QUESTIONS CHEAT SHEET

Top 3 Problem-Solving-Based Interview Questions

Alright, here is what you’ve been waiting for: the problem-solving questions and sample answers.

While many questions in this category are job-specific, these tend to apply to nearly any job. That means there’s a good chance you’ll come across them at some point in your career, making them a great starting point when you’re practicing for an interview.

So, let’s dive in, shall we? Here’s a look at the top three problem-solving interview questions and example responses.

1. Can you tell me about a time when you had to solve a challenging problem?

In the land of problem-solving questions, this one might be your best-case scenario. It lets you choose your own problem-solving examples to highlight, putting you in complete control.

When you choose an example, go with one that is relevant to what you’ll face in the role. The closer the match, the better the answer is in the eyes of the hiring manager.

EXAMPLE ANSWER:

“While working as a mobile telecom support specialist for a large organization, we had to transition our MDM service from one vendor to another within 45 days. This personally physically handling 500 devices within the agency. Devices had to be gathered from the headquarters and satellite offices, which were located all across the state, something that was challenging even without the tight deadline. I approached the situation by identifying the location assignment of all personnel within the organization, enabling me to estimate transit times for receiving the devices. Next, I timed out how many devices I could personally update in a day. Together, this allowed me to create a general timeline. After that, I coordinated with each location, both expressing the urgency of adhering to deadlines and scheduling bulk shipping options. While there were occasional bouts of resistance, I worked with location leaders to calm concerns and facilitate action. While performing all of the updates was daunting, my approach to organizing the event made it a success. Ultimately, the entire transition was finished five days before the deadline, exceeding the expectations of many.”

2. Describe a time where you made a mistake. What did you do to fix it?

While this might not look like it’s based on problem-solving on the surface, it actually is. When you make a mistake, it creates a challenge, one you have to work your way through. At a minimum, it’s an opportunity to highlight problem-solving skills, even if you don’t address the topic directly.

When you choose an example, you want to go with a situation where the end was positive. However, the issue still has to be significant, causing something negative to happen in the moment that you, ideally, overcame.

“When I first began in a supervisory role, I had trouble setting down my individual contributor hat. I tried to keep up with my past duties while also taking on the responsibilities of my new role. As a result, I began rushing and introduced an error into the code of the software my team was updating. The error led to a memory leak. We became aware of the issue when the performance was hindered, though we didn’t immediately know the cause. I dove back into the code, reviewing recent changes, and, ultimately, determined the issue was a mistake on my end. When I made that discovery, I took several steps. First, I let my team know that the error was mine and let them know its nature. Second, I worked with my team to correct the issue, resolving the memory leak. Finally, I took this as a lesson about delegation. I began assigning work to my team more effectively, a move that allowed me to excel as a manager and help them thrive as contributors. It was a crucial learning moment, one that I have valued every day since.”

3. If you identify a potential risk in a project, what steps do you take to prevent it?

Yes, this is also a problem-solving question. The difference is, with this one, it’s not about fixing an issue; it’s about stopping it from happening. Still, you use problem-solving skills along the way, so it falls in this question category.

If you can, use an example of a moment when you mitigated risk in the past. If you haven’t had that opportunity, approach it theoretically, discussing the steps you would take to prevent an issue from developing.

“If I identify a potential risk in a project, my first step is to assess the various factors that could lead to a poor outcome. Prevention requires analysis. Ensuring I fully understand what can trigger the undesired event creates the right foundation, allowing me to figure out how to reduce the likelihood of those events occurring. Once I have the right level of understanding, I come up with a mitigation plan. Exactly what this includes varies depending on the nature of the issue, though it usually involves various steps and checks designed to monitor the project as it progresses to spot paths that may make the problem more likely to happen. I find this approach effective as it combines knowledge and ongoing vigilance. That way, if the project begins to head into risky territory, I can correct its trajectory.”

17 More Problem-Solving-Based Interview Questions

In the world of problem-solving questions, some apply to a wide range of jobs, while others are more niche. For example, customer service reps and IT helpdesk professionals both encounter challenges, but not usually the same kind.

As a result, some of the questions in this list may be more relevant to certain careers than others. However, they all give you insights into what this kind of question looks like, making them worth reviewing.

Here are 17 more problem-solving interview questions you might face off against during your job search:

  • How would you describe your problem-solving skills?
  • Can you tell me about a time when you had to use creativity to deal with an obstacle?
  • Describe a time when you discovered an unmet customer need while assisting a customer and found a way to meet it.
  • If you were faced with an upset customer, how would you diffuse the situation?
  • Tell me about a time when you had to troubleshoot a complex issue.
  • Imagine you were overseeing a project and needed a particular item. You have two choices of vendors: one that can deliver on time but would be over budget, and one that’s under budget but would deliver one week later than you need it. How do you figure out which approach to use?
  • Your manager wants to upgrade a tool you regularly use for your job and wants your recommendation. How do you formulate one?
  • A supplier has said that an item you need for a project isn’t going to be delivered as scheduled, something that would cause your project to fall behind schedule. What do you do to try and keep the timeline on target?
  • Can you share an example of a moment where you encountered a unique problem you and your colleagues had never seen before? How did you figure out what to do?
  • Imagine you were scheduled to give a presentation with a colleague, and your colleague called in sick right before it was set to begin. What would you do?
  • If you are given two urgent tasks from different members of the leadership team, both with the same tight deadline, how do you choose which to tackle first?
  • Tell me about a time you and a colleague didn’t see eye-to-eye. How did you decide what to do?
  • Describe your troubleshooting process.
  • Tell me about a time where there was a problem that you weren’t able to solve. What happened?
  • In your opening, what skills or traits make a person an exceptional problem-solver?
  • When you face a problem that requires action, do you usually jump in or take a moment to carefully assess the situation?
  • When you encounter a new problem you’ve never seen before, what is the first step that you take?

Putting It All Together

At this point, you should have a solid idea of how to approach problem-solving interview questions. Use the tips above to your advantage. That way, you can thrive during your next interview.

FREE : Job Interview Questions & Answers PDF Cheat Sheet!

Download our " Job Interview Questions & Answers PDF Cheat Sheet " that gives you word-for-word sample answers to some of the most common interview questions including:

  • What Is Your Greatest Weakness?
  • What Is Your Greatest Strength?
  • Tell Me About Yourself
  • Why Should We Hire You?

Click Here To Get The Job Interview Questions & Answers Cheat Sheet

problem solving type of question

Co-Founder and CEO of TheInterviewGuys.com. Mike is a job interview and career expert and the head writer at TheInterviewGuys.com.

His advice and insights have been shared and featured by publications such as Forbes , Entrepreneur , CNBC and more as well as educational institutions such as the University of Michigan , Penn State , Northeastern and others.

Learn more about The Interview Guys on our About Us page .

About The Author

Mike simpson.

' src=

Co-Founder and CEO of TheInterviewGuys.com. Mike is a job interview and career expert and the head writer at TheInterviewGuys.com. His advice and insights have been shared and featured by publications such as Forbes , Entrepreneur , CNBC and more as well as educational institutions such as the University of Michigan , Penn State , Northeastern and others. Learn more about The Interview Guys on our About Us page .

Copyright © 2024 · TheInterviewguys.com · All Rights Reserved

  • Our Products
  • Case Studies
  • Interview Questions
  • Jobs Articles
  • Members Login

problem solving type of question

15 Common Problem-Solving Interview Questions

HackerRank AI Promotion

In an interview for a big tech company, I was asked if I’d ever resolved a fight — and the exact way I went about handling it. I felt blindsided, and I stammered my way through an excuse of an answer.

It’s a familiar scenario to fellow technical job seekers — and one that risks leaving a sour taste in our mouths. As candidate experience becomes an increasingly critical component of the hiring process, recruiters need to ensure the problem-solving interview questions they prepare don’t dissuade talent in the first place. 

Interview questions designed to gauge a candidate’s problem-solving skills are more often than not challenging and vague. Assessing a multifaceted skill like problem solving is tricky — a good problem solver owns the full solution and result, researches well, solves creatively and takes action proactively. 

It’s hard to establish an effective way to measure such a skill. But it’s not impossible.

We recommend taking an informed and prepared approach to testing candidates’ problem-solving skills . With that in mind, here’s a list of a few common problem-solving interview questions, the science behind them — and how you can go about administering your own problem-solving questions with the unique challenges of your organization in mind.

Key Takeaways for Effective Problem-Solving Interview Questions

  • Problem solving lies at the heart of programming. 
  • Testing a candidate’s problem-solving skills goes beyond the IDE. Problem-solving interview questions should test both technical skills and soft skills.
  • STAR, SOAR and PREP are methods a candidate can use to answer some non-technical problem-solving interview questions.
  • Generic problem-solving interview questions go a long way in gauging a candidate’s fit. But you can go one step further by customizing them according to your company’s service, product, vision, and culture. 

Technical Problem-Solving Interview Question Examples

Evaluating a candidates’ problem-solving skills while using coding challenges might seem intimidating. The secret is that coding challenges test many things at the same time — like the candidate’s knowledge of data structures and algorithms, clean code practices, and proficiency in specific programming languages, to name a few examples.

Problem solving itself might at first seem like it’s taking a back seat. But technical problem solving lies at the heart of programming, and most coding questions are designed to test a candidate’s problem-solving abilities.

Here are a few examples of technical problem-solving questions:

1. Mini-Max Sum  

This well-known challenge, which asks the interviewee to find the maximum and minimum sum among an array of given numbers, is based on a basic but important programming concept called sorting, as well as integer overflow. It tests the candidate’s observational skills, and the answer should elicit a logical, ad-hoc solution.

2. Organizing Containers of Balls  

This problem tests the candidate’s knowledge of a variety of programming concepts, like 2D arrays, sorting and iteration. Organizing colored balls in containers based on various conditions is a common question asked in competitive examinations and job interviews, because it’s an effective way to test multiple facets of a candidate’s problem-solving skills.

3. Build a Palindrome

This is a tough problem to crack, and the candidate’s knowledge of concepts like strings and dynamic programming plays a significant role in solving this challenge. This problem-solving example tests the candidate’s ability to think on their feet as well as their ability to write clean, optimized code.

4. Subarray Division

Based on a technique used for searching pairs in a sorted array ( called the “two pointers” technique ), this problem can be solved in just a few lines and judges the candidate’s ability to optimize (as well as basic mathematical skills).

5. The Grid Search 

This is a problem of moderate difficulty and tests the candidate’s knowledge of strings and searching algorithms, the latter of which is regularly tested in developer interviews across all levels.

Common Non-Technical Problem-Solving Interview Questions 

Testing a candidate’s problem-solving skills goes beyond the IDE . Everyday situations can help illustrate competency, so here are a few questions that focus on past experiences and hypothetical situations to help interviewers gauge problem-solving skills.

1. Given the problem of selecting a new tool to invest in, where and how would you begin this task? 

Key Insight : This question offers insight into the candidate’s research skills. Ideally, they would begin by identifying the problem, interviewing stakeholders, gathering insights from the team, and researching what tools exist to best solve for the team’s challenges and goals. 

2. Have you ever recognized a potential problem and addressed it before it occurred? 

Key Insight: Prevention is often better than cure. The ability to recognize a problem before it occurs takes intuition and an understanding of business needs. 

3. A teammate on a time-sensitive project confesses that he’s made a mistake, and it’s putting your team at risk of missing key deadlines. How would you respond?

Key Insight: Sometimes, all the preparation in the world still won’t stop a mishap. Thinking on your feet and managing stress are skills that this question attempts to unearth. Like any other skill, they can be cultivated through practice.

4. Tell me about a time you used a unique problem-solving approach. 

Key Insight: Creativity can manifest in many ways, including original or novel ways to tackle a problem. Methods like the 10X approach and reverse brainstorming are a couple of unique approaches to problem solving. 

5. Have you ever broken rules for the “greater good?” If yes, can you walk me through the situation?

Key Insight: “Ask for forgiveness, not for permission.” It’s unconventional, but in some situations, it may be the mindset needed to drive a solution to a problem.

6. Tell me about a weakness you overcame at work, and the approach you took. 

Key Insight: According to Compass Partnership , “self-awareness allows us to understand how and why we respond in certain situations, giving us the opportunity to take charge of these responses.” It’s easy to get overwhelmed when faced with a problem. Candidates showing high levels of self-awareness are positioned to handle it well.

7. Have you ever owned up to a mistake at work? Can you tell me about it?

Key Insight: Everybody makes mistakes. But owning up to them can be tough, especially at a workplace. Not only does it take courage, but it also requires honesty and a willingness to improve, all signs of 1) a reliable employee and 2) an effective problem solver.

8. How would you approach working with an upset customer?

Key Insight: With the rise of empathy-driven development and more companies choosing to bridge the gap between users and engineers, today’s tech teams speak directly with customers more frequently than ever before. This question brings to light the candidate’s interpersonal skills in a client-facing environment.

9. Have you ever had to solve a problem on your own, but needed to ask for additional help? How did you go about it? 

Key Insight: Knowing when you need assistance to complete a task or address a situation is an important quality to have while problem solving. This questions helps the interviewer get a sense of the candidate’s ability to navigate those waters. 

10. Let’s say you disagree with your colleague on how to move forward with a project. How would you go about resolving the disagreement?

Key Insight: Conflict resolution is an extremely handy skill for any employee to have; an ideal answer to this question might contain a brief explanation of the conflict or situation, the role played by the candidate and the steps taken by them to arrive at a positive resolution or outcome. 

Strategies for Answering Problem-Solving Questions

If you’re a job seeker, chances are you’ll encounter this style of question in your various interview experiences. While problem-solving interview questions may appear simple, they can be easy to fumble — leaving the interviewer without a clear solution or outcome. 

It’s important to approach such questions in a structured manner. Here are a few tried-and-true methods to employ in your next problem-solving interview.

1. Shine in Interviews With the STAR Method

S ituation, T ask, A ction, and R esult is a great method that can be employed to answer a problem-solving or behavioral interview question. Here’s a breakdown of these steps:

  • Situation : A good way to address almost any interview question is to lay out and define the situation and circumstances. 
  • Task : Define the problem or goal that needs to be addressed. Coding questions are often multifaceted, so this step is particularly important when answering technical problem-solving questions.
  • Action : How did you go about solving the problem? Try to be as specific as possible, and state your plan in steps if you can.
  • Result : Wrap it up by stating the outcome achieved. 

2. Rise above difficult questions using the SOAR method

A very similar approach to the STAR method, SOAR stands for S ituation, O bstacle, A ction, and R esults .

  • Situation: Explain the state of affairs. It’s important to steer clear of stating any personal opinions in this step; focus on the facts.
  • Obstacle: State the challenge or problem you faced.
  • Action: Detail carefully how you went about overcoming this obstacle.
  • Result: What was the end result? Apart from overcoming the obstacle, did you achieve anything else? What did you learn in the process? 

3. Do It the PREP Way

Traditionally used as a method to make effective presentations, the P oint, R eason, E xample, P oint method can also be used to answer problem-solving interview questions.  

  • Point : State the solution in plain terms. 
  • Reasons: Follow up the solution by detailing your case — and include any data or insights that support your solution. 
  • Example: In addition to objective data and insights, drive your answer home by contextualizing the solution in a real-world example.
  • Point : Reiterate the solution to make it come full circle.

How to Customize Problem-Solving Interview Questions 

Generic problem-solving interview questions go a long way in gauging a candidate’s skill level, but recruiters can go one step further by customizing these problem-solving questions according to their company’s service, product, vision, or culture. 

Here are some tips to do so:

  • Break down the job’s responsibilities into smaller tasks. Job descriptions may contain ambiguous responsibilities like “manage team projects effectively.” To formulate an effective problem-solving question, envision what this task might look like in a real-world context and develop a question around it.  
  • Tailor questions to the role at hand. Apart from making for an effective problem-solving question, it gives the candidate the impression you’re an informed technical recruiter. For example, an engineer will likely have attended many scrums. So, a good question to ask is: “Suppose you notice your scrums are turning unproductive. How would you go about addressing this?” 
  • Consider the tools and technologies the candidate will use on the job. For example, if Jira is the primary project management tool, a good problem-solving interview question might be: “Can you tell me about a time you simplified a complex workflow — and the tools you used to do so?”
  • If you don’t know where to start, your company’s core values can often provide direction. If one of the core values is “ownership,” for example, consider asking a question like: “Can you walk us through a project you owned from start to finish?” 
  • Sometimes, developing custom content can be difficult even with all these tips considered. Our platform has a vast selection of problem-solving examples that are designed to help recruiters ask the right questions to help nail their next technical interview.

Get started with HackerRank

Over 2,500 companies and 40% of developers worldwide use HackerRank to hire tech talent and sharpen their skills.

Recommended topics

  • Coding Questions
  • Interview Preparation

Abstract, futuristic image generated by AI

6 REST API Interview Questions Every Developer Should Know

Explore Jobs

  • Jobs Near Me
  • Remote Jobs
  • Full Time Jobs
  • Part Time Jobs
  • Entry Level Jobs
  • Work From Home Jobs

Find Specific Jobs

  • $15 Per Hour Jobs
  • $20 Per Hour Jobs
  • Hiring Immediately Jobs
  • High School Jobs
  • H1b Visa Jobs

Explore Careers

  • Business And Financial
  • Architecture And Engineering
  • Computer And Mathematical

Explore Professions

  • What They Do
  • Certifications
  • Demographics

Best Companies

  • Health Care
  • Fortune 500

Explore Companies

  • CEO And Executies
  • Resume Builder
  • Career Advice
  • Explore Majors
  • Questions And Answers
  • Interview Questions

Problem-Solving Interview Questions And Answers (With Examples)

  • How To Answer Tell Me About Yourself?
  • Elevator Pitch
  • Where Do You See Yourself In 5 Years?
  • What Are Your Career Goals?
  • When Can You Start?
  • How Do You Define Success?
  • Describe Your Work Ethic
  • Where Are Your Current Duties?
  • What Are Your Learning Goals?
  • Intrinsic Vs Extrinsic Motivation
  • What Is Your Desired Salary?
  • What Makes You Unique?
  • Why Are You The Best Person For This Job?
  • Reasons For Termination
  • What Are Your Work Values
  • How To Make A Hard Decision?
  • What Are You Most Proud Of?
  • Personal Code Of Ethics
  • Problem Solving Interview Questions
  • Taking Initiative Example
  • How Do You Prioritize Your Work
  • Explain Gaps In Employment
  • Most Rewarding College Experience
  • What Is Your Work Style
  • Tell Me About A Time When You Made A Mistake On The Job
  • Tell Me About Gaps In Employment
  • What Are You Passionate About
  • What Skills Would You Bring To The Job
  • Who Is Your Mentor?
  • How To Answer Tell Me About A Time You Disagreed With Your Boss
  • How To Answer Common Screening Questions

Find a Job You Really Want In

Summary. Problem-solving questions are used to focus on a candidates past experience with managing conflicts and overcoming obstacles in the workplace. When answering these questions, be sure to make your answer relevant to the position that you are applying to and be honest about your strengths and weaknesses. Be sure to provide examples from previous experiences.

Are you in the process of searching for a new job ? If so, you might be getting ready to meet with a hiring manager or a recruiter for a job interview. And if you’re like the majority of job candidates, this stage of the job search process is probably making you feel a fair bit of trepidation.

And no wonder! The interview is a completely necessary step for any job search, but that doesn’t make it any less nerve-wracking to meet with a prospective employer and answer questions about your personality , skills, and professional background.

Key Takeaways:

Being able to solve problems is a skill that almost all job positions need.

Problem-solving questions assess a candidate’s ability to think on their feet, handle pressure, and find creative solutions to complex problems.

Make sure your answer to a problem-solving question tells a story of you as an effective team player.

Problem Solving Interview Questions And Answers (With Examples)

What Is a Problem-Solving Interview Question?

How to answer a problem-solving interview question, eight examples of common problem-solving interview questions and answers, interviewing successfully, curveball questions, problem-solving faq.

  • Sign Up For More Advice and Jobs

A problem-solving interview question is a question that focuses on a candidate’s past experience with managing conflicts and overcoming unexpected obstacles in the workplace.

Problem-solving questions can come up in many different forms. As a general rule, however, they will be aimed at uncovering your ability to handle stress and uncertainty in a wide variety of contexts.

When you’re answering problem-solving interview questions, there are a few important tips to keep in mind:

Make your answers relevant to the position that you’re applying to. Always bear in mind that the fundamental goal of any interview question is to provide a hiring manager with a glimpse inside the mind of a candidate.

By asking you a problem-solving question, your interviewer is trying to understand whether or not you’re the type of person that could be relied upon under pressure or during a crisis. Every role, furthermore, comes with its own particular type of pressure.

Be honest about your strengths ( and weaknesses ). Hiring managers tend to be quite good at reading people. Therefore, if you give them a bogus response, they’re very likely to see through that – and to subsequently consider you to be untrustworthy.

Of course, it can be tempting at the moment to fabricate certain details in your response in the attempt to make yourself seem like a better candidate. But inventing details – however small – tends to backfire .

Tell stories that will portray you as a team player. Hiring managers and employers are always on the lookout for job candidates who will collaborate and communicate well amongst a broader team.

Be sure to provide examples of moments in which you took charge. Leadership skills are another key quality that hiring managers and employers seek out in job candidates. And being presented with a problem-solving question, as it turns out, is the perfect opportunity to demonstrate your own leadership skills.

Now that we understand the basic principles of problem-solving interview questions and how to respond to them, we’re finally ready to break down some real-world examples. So without any further preamble, here are eight examples of common problem-solving interview questions (as well as some examples of how you might answer them):

Can you tell me about a time when you encountered an unexpected challenge in the workplace? How did you go about dealing with it?

Explanation: With this question , your interviewer will be attempting to get a sense of how well you’re able to adapt to unexpected difficulties. The critical thing to remember when you’re answering this question – as we briefly discussed above – is to recall an incident that will be directly relevant to the role and the organization that you’re applying to.

Here’s an example of a high-quality response to this question:

“I remember a particular day at my previous job when an important deadline was pushed up at the very last minute. As the project manager , it was my responsibility to implement the necessary steps that would enable us to meet this new and truncated deadline. “Many of my peers began to hang their heads, resigning themselves to their belief that there was no hope to meet the new deadline. But I’ve always prided myself on my ability to adapt and thrive within a dynamic and quick-paced work environment – and that’s precisely the personal skill set that I channeled on this occasion. In the end, I reorganized my team’s priorities so that we were able to accommodate the new deadline.”

How would you say you typically respond to problems in general, and in the workplace in particular?

Explanation: This question is primarily designed to gauge a candidate’s ability (or lack thereof) to remain cool, calm, and collected under pressure. The ideal response to this question, in other words, will include a brief personal anecdote that illustrates your level-headedness and your ability to make rational, clear decisions during times of uncertainty.

“I would say that one of the primary qualities that sets me apart from the crowd of other candidates is my ability to remain calm and centered when conditions in the workplace become chaotic. “Looking back, I think that I first began to cultivate this ability during my tenure as a product manager working with a major Silicon Valley start-up. That was a particularly stressful period, but it was also quite instructive – I learned a great deal about staying positive, focused, and productive after an unexpected challenge presented itself. “These days, when I’m confronted by an unexpected problem – whether it’s in my personal life or in my professional life – I immediately channel the conflict management skills that I’ve been honing throughout the duration of my career. This helps a great deal, and my skills in this regard are only continuing to improve.”

Can you tell me about a time when you’ve had to settle a workplace dispute between yourself and a manager or colleague?

Explanation: Always keep in mind that one of the fundamental goals of any problem-solving question is to help a hiring manager gain a clearer sense of a candidate’s ability to work with others.

This question, in particular, is designed to give your interviewer a clearer sense of how well you’re able to communicate and compromise with your colleagues. With that in mind, you should be sure to answer this question in a way that will display a willingness to be fair, empathetic, and respectful to your teammates.

“I recall an incident in my last job in which one of my colleagues felt that I had not provided him with adequate resources to enable him to be successful in a particular project. I was acting as team leader for that particular project, and so it was my responsibility to ensure that everyone in my team was equipped for success. Unfortunately, I had to learn through the proverbial grapevine that this particular colleague bore some ill will toward me. I’ve never been one to participate in idle gossip, and so I decided to speak with this person so that we could begin to find a solution and address his grievances. So I crafted an email to him asking him if he would be interested in joining me for coffee the following day. He accepted the invitation, and during our coffee break, we were able to talk at length about the damage that he felt had been done to him. We devised a mutually agreeable solution on the spot. From then on, we had no significant problems between us.”

Are there any steps that you’ll regularly take during the early stages of a new project to ensure that you’ll be able to manage unexpected problems that occur down the road?

Explanation: This question, above all, is designed to test your ability to plan ahead and mitigate risk. These are both essential qualities that employers typically seek out in job candidates, particularly those who are being vetted for a management or leadership role.

When you’re answering this question, it’s important to emphasize your ability to look ahead towards the future and anticipate potential risks. As with the previous examples that we’ve already examined, the best way to communicate this ability is to provide your interviewer with a concrete example from your previous work history.

“I live my life – and I conduct my work – according to a single, incredibly important motto: “Failing to prepare is preparing to fail.” I’m a firm believer, in other words, of the primacy of careful planning. Without it, projects are almost always doomed to fail. “In my previous role as a marketing content writer with a major software company, I strived to apply this motto to my work every single day. “Here’s an example: About a year ago, I was responsible for overseeing and launching a new content strategy aimed at driving up consumer engagement. From the very outset, I understood that that particular project could be run off the rails if we did not take into account a considerable number of factors. “I won’t bore you with all of the nitty-gritty details, but the point is that this was a particularly sensitive project that required diligent and careful risk assessment. “Having realized that, my colleagues and I devised a comprehensive and flexible strategy for managing many risks that we envisioned would be awaiting us down the road. That initial step – looking ahead towards the future and mapping out the terrain of potential hazards – proved to be an essential measure for the success of the project.”

Do you consider your problem-solving capabilities to be above average?

Explanation: Hiring managers are always on the lookout for job candidates that stand out from the crowd. It’s even better when they can find a job candidate who knows that they stand out and who expresses that knowledge by being confident in their abilities.

At the same time, it’s never in a job candidate’s best interests to come across as egotistical or arrogant. When you’re responding to a question like this (that is, a question that’s focused on your ability to assess your own talents), it’s important to do your best to come across as self-assured but not pompous.

“Yes, all things considered, I would say that I have a talent for risk assessment, problem-solving, and risk mitigation. “That said, I can’t claim complete ownership over these abilities. In most cases, my demonstrated success in managing risk and solving problems in the workplace can be attributed at least as much to my team members as it can to me. For me to be able to be a successful problem-solver, it helps to be surrounded by colleagues whom I can trust.”

How would you describe your typical immediate reaction to unexpected challenges? Do you prefer to jump straight into the problem-solving process, or do you more commonly take some time to analyze and assess the problem before you dive in?

Explanation: This question is aimed at gauging your patience levels. This one can be a bit tricky because employers will sometimes prefer different responses – it all depends on the type of position and employer you’re applying for.

If you’re applying for a role in a quick-paced working environment that demands swift action , it will benefit you to describe your problem-solving strategy as unflinching and immediate.

If, on the other hand, the role you’re applying to does not demand such immediate action, it will probably be better to describe yourself as a more removed and relaxed problem solver.

But as always, you should never lie to your employer. Most of us will fall somewhere in the middle of these two types of problem solvers and will thereby have no difficulty painting ourselves honestly as one or the other.

However, if you’re definitely one type or the other, then you should describe yourself as such. This will make it much more likely that you’ll end up in a position that will be maximally rewarding both for you and for your employer.

“In most cases, my response to an unexpected problem will entirely depend on the nature of the problem at hand. If it demands immediate action, then I’ll dive right in without hesitation. “If, however, I determine that it would be more beneficial to take a step back and analyze the nature of the problem before we begin to meddle with it, then that’s exactly what I’ll do. “Generally speaking, I would say that I prefer the latter approach – that is, to take a step back and think things through before I begin to try to find a solution. In my experience, this makes it much easier for everyone involved to arrive at a practical and sustainable solution. “That said, I’m also perfectly capable of jumping straight into a problem if it demands immediate attention.”

Can you tell us about a time in which you had to explain a technically complicated subject to a client or customer? How did you approach that process, and how did it turn out?

Explanation: Strong communication skills are essential in the modern workplace. That means that employers tend to seek out job candidates that communicate well with their colleagues and individuals who have varying professional backgrounds and skill sets, including clients, customers, and third-party professionals.

“I recall an incident from many years ago – while I was working as a software engineer for a prominent robotics company – in which I found myself in the position of having to describe incredibly complex engineering details to a client. “This client had no prior experience in software engineering or artificial intelligence, so I had to relate this esoteric information more or less in layman terms. “Thankfully, I was able to employ some useful metaphors and analogies to communicate the information in a manner that this client could appreciate and understand. We went on to establish a successful collaborative partnership that flourished for four years.”

How would you rate your ability to work and succeed without direct supervision from your managers?

Explanation: Employers always tend to place a high value on job candidates who are self-motivated and can maintain high levels of productivity without constant supervision.

This is especially true now that the COVID-19 pandemic has suddenly made it necessary for so many millions of employers to transition to a remote workforce model. This question is designed to assess a candidate’s ability to stay focused and motivated while working remotely or without supervision.

“I’ve always considered myself – and my resume and references will support this – to be an exceptionally self-motivated individual, even when I’m working from home. “In fact, like many employees, I often find that my productivity levels tend to increase when I’m working remotely. I strive to set a positive example for my colleagues, even when we’re not all working under the same roof.”

Generally speaking, the best strategy for success in interviewing for a new job is doing your research beforehand. That means that you should be intimately familiar with the role, department, and company that you’re applying to before you step into the room (or log on to the Zoom meeting ) on the day of your interview.

When you preemptively take the time to carefully research the organization as a whole – and the responsibilities of the job opportunity in particular – you’ll minimize your chances of being caught off guard by an unexpectedly difficult question .

Still, there is only so much background information that you can uncover about an organization and a role before a job interview. No matter how carefully you prepare and how much background research you conduct, there are very likely going to be curveball questions during your job interview that you can’t predict.

In fact, many employers prefer to ask curveball questions (in addition to more run of the mill job interview questions) because they provide an insightful glimpse into a job candidate’s analytical thinking skills – not just their ability to memorize and recite answers to more common interview questions .

To that end, many hiring managers will ask job candidates to answer one or more problem-solving questions during a typical job interview. In contrast to traditional interview questions (such as: “Why do you think that you would be a good fit for this role?”

Or: “What do you consider to be your greatest professional achievement up to the current moment?”), problem-solving questions are specifically designed to assess a job candidate’s ability to think on their feet, handle real pressure, and find creative solutions to complex problems.

They’re also commonly referred to as analytical skills interview questions because they’re designed to gauge a candidate’s ability to make analytical decisions in real-time.

What are problem-solving skills?

Problem-solving skills include skills like research, communication, and decision making. Problem-solving skills allow for you to identify and solve problems effectively and efficiently. Research skills allow for you to identify the problem.

Communication skills allow for you to collaborate with others to come up with a plan to solve the problem. Decision making skills allow you to choose the right solution to the problem.

Why do interviewers ask problem-solving interview questions?

Interviewers ask problem-solving interview questions to see how candidate will approach and solve difficult situations. Interviewers want to see how you handle stress and uncertainty before hiring you for a position. Problem-solving is an important part of the everyday workday so they need to be sure you are capable of solving problems.

How do you solve a problem effectively?

To solve problems effectively you should first break the problem down and try different approaches. Breaking the problem up into different parts will help you have a better understanding and help you decide what your next step is going to be.

Once you see the different parts of the problem, trying different approaches to solve the problem can help you solve it faster. This will also help you determine the appropriate tools you need to solve the problem.

U.S. Department of Labor – Interview Tips

How useful was this post?

Click on a star to rate it!

Average rating / 5. Vote count:

No votes so far! Be the first to rate this post.

' src=

Chris Kolmar is a co-founder of Zippia and the editor-in-chief of the Zippia career advice blog. He has hired over 50 people in his career, been hired five times, and wants to help you land your next job. His research has been featured on the New York Times, Thrillist, VOX, The Atlantic, and a host of local news. More recently, he's been quoted on USA Today, BusinessInsider, and CNBC.

Recent Job Searches

  • Registered Nurse Jobs Resume Location
  • Truck Driver Jobs Resume Location
  • Call Center Representative Jobs Resume Location
  • Customer Service Representative Jobs Resume
  • Delivery Driver Jobs Resume Location
  • Warehouse Worker Jobs Resume Location
  • Account Executive Jobs Resume Location
  • Sales Associate Jobs Resume Location
  • Licensed Practical Nurse Jobs Resume Location
  • Company Driver Jobs Resume

Related posts

problem solving type of question

How To Answer “What Is Your Teaching Philosophy?” (With Examples)

problem solving type of question

How To Describe Your Current Job Duties (With Examples)

problem solving type of question

How To Answer “When Can You Start?” (With Examples)

problem solving type of question

Common Teamwork Interview Questions (With Answers)

  • Career Advice >
  • Interview Questions >
  • Common Questions >

8 Problem-Solving Interview Questions You Should Ask

Post Author - Juste Semetaite

Employers need professionals who can cope with change. Especially in a modern workplace that is fast-paced and dynamic, problem-solving skills are more critical now than ever. Of course, having the right people starts with who and how you hire.

To find the best problem solvers, hiring managers rely on problem-solving interview questions and skills tests. In the interview, asking various behavioral-type questions can help assess a candidate’s ability to analyze complex situations, think critically , and develop innovative solutions.

In this article, we’ll explore eight different types of problem-solving interview questions and answers, how to identify any red flags in candidate answers, and a quick-fire list of tips to ensure you bring the best aboard your organization.

TL;DR – Key Takeaways

  • Problem-solving interview questions are designed to assess a candidate’s ability to think critically , analyze situations, and find innovative solutions.
  • Hiring managers use problem-solving questions in the job interview to evaluate critical skills and competencies such as analytical thinking, decision-making, adaptability, creativity, collaboration, and communication .
  • A predictor of future job performance is past performance. By understanding how they have dealt with problems in the past, you can get a better picture of how they might apply those skills to your organization.
  • Red flags to watch out for during the job interview include a lack of specific examples, vague or generalized answers, limited adaptability, poor decision-making, lack of collaboration or communication skills, and limited initiative or creativity.
  • Tips for using problem-solving questions to screen candidates include asking job-specific questions, encouraging candidates to use the STAR method, asking different types of problem-solving questions, and preparing responses .
  • Interviews are great for top-level evaluation of problem-solving skills. But if you want to get to the bottom of candidates’ job-specific competencies and have reliable data to compare top candidates, try skills assessments instead! See our test library for inspiration. 

People with strong problem solving skills will structure their answers, for example, using the STAR method.

What Are Problem-Solving Interview Questions?

Problem-solving interview questions are a type of behavioral question used to assess a candidate’s ability to think critically, gather and analyze data, and work through logical solutions. There often is no right or wrong answer , but a strong answer will check the boxes by explaining how they would come to a solution by walking through all the relevant steps.

questions can take many different forms, but they all share a common goal: to evaluate an individual's problem-solving skills in a specific context

For example, a problem-solving question might be to ask the candidate to describe a time when they had to change their planned course of action at the last moment. The interviewer is not only interested in hearing about how the candidate solved the specific problem but also in learning more about their problem-solving approach and what they did to manage the unexpected change.

It is often thought that past employee behaviour can predict the future. That’s why problem-solving interview questions are often designed to elicit specific examples from the candidate’s own work experience. By talking through concrete examples, interviewers can better understand the candidate’s problem-solving abilities and how they might apply those skills to the job at hand.

Want to know more about behavioral interview questions ?

30 Behavioral Interview Questions to Ask Candidates (With Answers)

Why Interviewers Ask Problem-solving Interview Questions

For most hiring managers, the interview is a critical step in the hiring process. In addition to using skills assessments to screen candidates for problem-solving skills, they need to ask problem-solving interview questions to get a deeper understanding of this skill.

Probing questions help hiring managers to evaluate candidates’ critical thinking skills , providing insight into how well they might perform on the job. This approach enables interviewers to understand the candidate’s problem-solving competency and the methods that they adopt.

Interviewers will be looking to understand their capacity to analyze information, generate innovative ideas, adapt to unexpected obstacles, make sound decisions, collaborate with others, and effectively communicate their ideas.

Therefore, an effective problem-solver will also demonstrate a range of other important skills, such as analytical thinking, decision-making, adaptability, creativity, collaboration, and communication.

problem solving type of question

8 Examples of Common Problem-Solving Interview Questions and Answers

Problem solving is one of many key interpersonal skills that a peer interview question can assess during a job interview.

Now for the main course of this article. We’re going to dive into eight types of example problem-solving questions that you can use during interviews, explaining why they are relevant and what makes a strong answer.

1. The challenging situation

Recall a difficult problem or challenging situation you encountered in a previous role. How did you analyze the problem, and what steps did you take to arrive at a solution?

The reason: Assesses a candidate’s ability to handle complex and challenging situations as well as their problem-solving approach, communication, and decision-making skills.

The answer: The candidate should share a specific instance of a problematic situation they faced in a previous role and describe their problem-solving approach. Specifically, how they analyzed the problem, including what information they gathered and resources they used to arrive at a solution.

Bonus points: If they can highlight any obstacles they faced and how they overcame them, as well as the positive outcomes of their solution.

2. Problem-solving process

Walk me through your problem-solving process . Explain your personal approach to problem-solving by taking me through the steps you typically follow.

The reason: To better understand a candidate’s problem-solving approach and methodology.

The answer: A solid answer consists of a brief description of the candidate’s personal problem-solving approach , highlighting the steps they typically follow, different options they would consider, and resources used to make informed decisions.

Bonus points: If they also mention any tools or techniques , such as the scientific method or SWOT analysis, and provide examples of times when their approach was successful.

3. Decision-making

Share an instance where you needed to make a quick decision to resolve an urgent problem. How did you decide on a course of action, and what was the outcome?

The reason: Test a candidate’s decision-making and problem-solving skills in stressful and unexpected situations.

The answer: The interviewee should describe how they gathered relevant information quickly, considered various options, and arrived at an informed decision all within a limited space of time.

Bonus points: If they can demonstrate competence in handling stressful situations , especially if the role may require it.

4. Creative thinking

Give me an example of a time when you had to think outside the box to solve a problem. How did you approach the situation differently or creatively, and what was the outcome?

The reason: Understand a candidate’s ability to think creatively and innovatively when faced with a problem.

The answer: The interviewee should describe a specific situation where they used creative thinking to solve a problem. They should explain their unique approach and any unconventional ideas or solutions they came up with.

Bonus points: If they can demonstrate exactly how their creative solution contributed to a successful outcome.

5. Teamwork

Describe a situation where you had to work with a team to solve a complex problem. Detail your role and contributions to the team’s overall success in finding a solution.

The reason: Understand a candidate’s ability to work collaboratively and effectively with others when solving difficult problems.

The answer: How do they narrate a particular scenario where they worked with a team to collectively solve a complex problem, specifying their role and that of the team in arriving at a solution.

Bonus points: If they can recognize the role of others and the strength of the team over the individual in solving the problem.

6. Overcoming obstacles

Can you share an example of a project or task where you had to overcome unexpected obstacles or challenges? How did you adapt and find a solution?

The reason: Handling unexpected obstacles or challenges and their problem-solving skills.

The answer: To answer this question, the interviewee should share a particular project or task where they faced unforeseen challenges or obstacles, how they adapted to the situation and found a solution.

Strong problem solving answers will showcase a candidate's past experience.

Bonus points: If they emphasize any creative or innovative methods they employed.

7. Dealing with recurring problems

Give me an example of a time when you identified a recurring problem in a process or system. What steps did you take to address the issue and prevent it from happening again?

The reason: This question assesses a candidate’s ability to identify and solve recurring problems and improve processes.

The answer: The job seeker should recount a specific instance of a recurring problem they detected in a process or system .

Bonus points: If they can explain exactly how they got to the root of the problem and the steps or measures they took to prevent its recurrence .

8. Multi-tasking

Tell me about a situation where you had to prioritize multiple tasks or projects with competing deadlines. How did you prioritize and allocate your time to ensure the successful completion of all tasks?

The reason: Tests a candidate’s capacity to organize, prioritize, and multitask to complete multiple assignments or tasks in a timely manner.

The answer: The interviewee should illustrate a specific instance where they successfully managed multiple projects or tasks simultaneously , elaborating on how they prioritized their workload and managed their time efficiently.

Bonus points: If they highlight any project management tools or techniques used, and if the project or task was delivered on time.

20 Steal-worthy Interview Questions for Managers

Now that we’ve gone over the best possible answers for these questions, let’s look at some of the negatives and red flags to keep an eye out for.

Red Flags for Interviewers Assessing Problem-solving Skills

HR managers should be aware of red flags during an interview that could indicate weakness in a candidate’s problem-solving skills.

problem solving type of question

Red flags to watch for include:

A lack of specific examples

If a candidate has a hard time recalling specific past problem-solving examples, this may signal they lack relevant experience or have difficulty remembering events.

Vague or generalized answers

Candidates who give vague, general, or unclear answers without describing the specifics of their problem-solving process may lack the ability to solve problems effectively. Is the candidate trying to avoid the question? When probed further, are they able to get more specific?

Limited adaptability

If the individual is unable to describe situations where they persevered through obstacles or utilized alternate solutions, it may display an absence of resilience, unwillingness or incapacity to be adaptable.

Poor decision-making skills

Candidates who lack the ability to explain their thought process, take into account alternative perspectives, or make unwise decisions likely possess weak decision-making skills. Look for candidates who contemplate decisions carefully, consider the pros and cons, and can articulate their reasons for choosing their final course of action.

Lack of collaboration or communication skills

Poor communication, collaboration, and teamwork skills can hinder problem-solving, especially in situations where input or feedback from stakeholders is required.

Limited initiative or creativity

Problem solvers who stand out demonstrate initiative, creativity, and a drive to think unconventionally. Those who cannot offer examples of inventive problem-solving or use only traditional techniques may not possess the ability to come up with creative solutions.

Tips For Using Problem-Solving Questions To Screen Candidates

Before you run off and start asking all of the above problem-solving interview questions, there are a few more factors to consider. To be specific, context is king when it comes to speaking to interviewees during the job interview. And the below tips will help you to understand them better.

  • Always be sure to ask job-specific questions
  • Start with a robust, written job description that details all the required skills, competencies, and experience to compare with the candidate’s answers
  • Keep a look out for generic answers
  • Do they use the STAR method to structure their thinking/answers?
  • Ask different types of problem-solving questions
  • Reword the question if a candidate is having trouble answering it
  • Ask how they handle a situation that doesn’t have an easy outcome or answer
  • Inquire if they have ever had disciplinary action taken against them and how they handled it
  • Ask them team-related questions
  • Prepare responses that you can play off of their answers
  • Check if they have ever tried to inspire their team
  • It’s not out of the ordinary to ask the candidate out-of-the-box questions (How would you escape a blender?) to understand how they solve problems

Structured Interview vs. Unstructured Interview: What’s the Difference?

You’re almost ready to integrate problem-solving questions into your job interview workflow, but there’s just one last topic to cover: Is there a piece of software that can help you to streamline the problem-solving interview process?

Yes, yes, there is.

Evaluating problem-solving skills beyond the interview

While interviews are a useful tool for recruiters and hiring managers to gauge candidates’ competence, they’re not quite sufficient for assessing candidates’ full skill set. That’s especially true when the role requires mastery of a certain technical or power skill, like problem-solving.

problem solving skills test

A better, more effective way to evaluate candidates ‘ abilities is to combine structured interviews with job-specific skills assessments. Here are some of the reasons why:

  • It allows for more objective evaluation. Interviews inherently favor candidates with advanced communication skills, charisma, and confidence. But! Just because a candidate interviews well, doesn’t mean they have what it takes to succeed in the role. Sadly, the interviewer’s perception of a candidate is almost always highly influenced by the candidate’s interviewing skills. Incorporating a skills assessment can help you assess candidates’ actual abilities in role-specific tasks.
  • It offers a practical demonstration. Interviews often rely on a candidate’s self-reporting of their skills and past experiences. However, candidates may overstate their abilities or have difficulty articulating their skills in an interview setting. Skill-specific assessments give candidates the opportunity to demonstrate their abilities in a practical, real-world context. This allows hiring managers to see the candidate’s skills in action, which can be a more reliable indicator of their ability to perform in the role.
  • It guarantees consistent metrics. Assessments provide a consistent set of metrics to compare all candidates. This can help to eliminate bias and ensure fairness in the hiring process. Interviews can be more subjective and may vary greatly depending on the interviewer or the specific questions asked. Having a standardized assessment ensures that all candidates are evaluated on the same criteria.
  • It helps to predict job performance. Research has shown that work sample tests, which are a type of skill-specific assessment, are one of the best predictors of job performance. They can provide valuable insights into how a candidate might perform in the job beyond what can be learned from an interview alone.
  • It makes the hiring process more efficient. Skill-specific assessments can also make the hiring process more efficient. If a candidate performs poorly on an assessment early in the process, this could save time for both the candidate and the company by indicating that the candidate may not be the right fit for the role.

Interested in exploring a skills-based hiring approach? Take no risks – start with our free account to browse all available assessment templates .

Juste Semetaite

Juste loves investigating through writing. A copywriter by trade, she spent the last ten years in startups, telling stories and building marketing teams. She works at Toggl Hire and writes about how businesses can recruit really great people.

Join 30,000+ subscribers getting the best tips on productivity, work management, hiring and more!

We promise we won't spam you and you can unsubscribe anytime.

You might also like...

Related to Talent Assessments

30 Examples of Employee Strengths Every Company Needs

30 Examples of Employee Strengths Every Company Needs

Toggl Blog, Read articles by Julia Masselos

How to Use Personality Tests for Hiring (The Right Way) 

Toggl Blog, Read articles by Elena Prokopets

Specialist vs Generalist: What Makes a Better Hire?

Toggl Blog, Read articles by Juste Semetaite

Take a peek at our most popular categories:

The Best Interview Questions for Assessing Problem Solving Skills

Metaview

No matter the domain, industry, or seniority level, problem solving skills are an essential tool for any employee. With carefully-crafted interview questions, you can get a solid understanding of how candidates will respond to real-world challenges they're likely to face at your organization. Zero in on candidates' problem-solving prowess with our curated list of the best interview questions.

Unlocking a Candidate's Problem Solving Abilities

By probing a candidate's problem solving skills, you'll get a more thorough understanding of their approach, thought process, and skill in solving challenges and having impact in the workplace. With problem solving interview questions, the objective is to get a better sense for how a candidate approaches:

  • Identifying and analyzing problems
  • Creating action plans
  • Implementing solutions
  • Evaluating results
  • Taking learnings forward for the future

In job interviews, you'll want to dig into specific examples of candidates' problem solving process in real-world scenarios, and test how they would deal with hypothetical problems that might arise in your own company.

Problem Solving Interview Questions

  • Describe a time when you solved a problem without input from someone more senior to you.
  • How do you decide when to handle a problem independently or seek help?
  • Tell me about a time when you preemptively addressed a complex problem before it escalated.
  • If faced with two urgent tasks simultaneously, how would you prioritize them?
  • Describe a time when you developed an innovative solution with limited resources or information.
  • Describe a time when you had to develop a solution for a problem when you had limited resources or information.
  • Tell me about a situation where you came up with a creative solution to a problem.
  • Walk me through an experience of tackling a daunting project.
  • Tell me about a time when you had to change your strategy last minute.
  • Describe a time when you faced significant obstacles to solving a problem.
  • Can you give me examples of metrics you use to measure the success of your problem-solving efforts?
  • How do you decide when a problem is "solved"?
  • Can you tell me about a time when you've solved a problem that required making tradeoffs between short and long-term outcomes?

General problem solving & decision making skills interview questions

Effective problem-solving goes hand in hand with sound decision-making. Identifying candidates who can independently tackle issues and make well-reasoned decisions is crucial.

Here are some interview questions to explore how candidates approach their decision making process in problem solving:

What to look for in responses:.

These interview questions will help assess how job candidates use critical thinking and initiative to tackle problems. Do you they wait for others to tell them when to fix something, or do they proactively find opportunities to make improvements? Look for answers demonstrating an analytical approach to the prioritization and execution of problem solving. Make sure you dig into the candidate's thought process behind how they assess tradeoffs and think about the impact of potential solutions.

Interview questions for assessing creativity in problem solving

Few problems can be solved without some degree of creativity. To get a sense for these skills in a potential candidate, you can pose problem-solving questions that dig into the their past experiences with delivering original and out-of-the box solutions.

To gauge a candidate's creative problem-solving skills, consider these job interview questions:

These questions uncover the candidate’s ability to think outside the box. If they struggle to come up with detailed answers, it's likely a sign they rely on tried and tested ways of doing things rather than searching for innovative solutions. Look for answers that showcase originality, inventive use of resources, and the ability to deliver practical solutions under constraints.

As when testing most competencies, it's important to hear real-world examples of problem solving rather than generic answers that don't reveal anything about how they approached complex situations in previous roles. As always, look for answers that use the STAR (situation-task-action-result) method to effectively demonstrate how the candidate uses creativity to solve problems.

Problem solving interview questions for adaptability and resourcefulness

The ability to adapt and be resourceful is essential in fast-paced work environments and is a key component of strong problem-solving capabilities.

Here are some interview questions you can use to assess these skills:

Responses to these questions should highlight the candidate’s flexibility and resourcefulness as demonstrated in previous experiences. Effective answers typically include examples of problem solving by adjusting strategies, on-their-feet thinking, and maintaining composure under pressure. You'll want to get a clear understanding of whether a candidate can thrive in challenging situations. Do they break down in stressful situations or do they maintain composure and find a way forward? Do you they know when to ask for help to maximize chances of success? These are the types of questions you'll want to address in your probing.

Problem solving interview questions that test results-orientation

Execution is one thing, but strong problem solving skills must also include a focus on results, measurability, and long-term impact.

Here are some interview questions to assess how a candidate thinks about the results of their problem solving:

You'll want to see that a candidate doesn't have a "box ticking" mentality, where they want to close out a problem just to check it off their list. Do they think critically about how to define and measure success, or do they take a binary problem solving approach? A candidate's problem-solving skills are only as good as their ability to understand the quality of their solutions and the tradeoffs of their impact.

Carefully crafted problem-solving questions can play a crucial role in assessing a candidate’s ability to navigate difficult situations and devise effective solutions. We hope this guidance on problem solving interviews helps you more thoroughly understand a candidate's approach.

Other resources

problem solving type of question

MockQuestions

Problem Solving Mock Interview

To help you prepare for your job interview, here are 25 interview questions that will test your problem solving ability.

Get More Information About Our Problem Solving Interview Questions

Question 1 of 25

Tell me about the most challenging problem you have encountered in your professional career.

How to Answer

Answer example.

Everyone has had their share of challenges in their career. The interviewer knows that you are not perfect; however, they need to know that you can professionally overcome work-related roadblocks. Maybe you had a significant project that almost went sideways. Perhaps you had a conflict in the workplace that you could have handled more professionally. Explain your approach to resolving the issue and be sure to highlight the steps you took to reach that resolution.

"The most challenging problem I have encountered in my professional career was with my most recent employer. I had an incredibly important project that made up the majority of my annual budget. The client was challenging to work with as he was rarely available for comment, due to extensive international travel. I needed this deal to work out so, for the 6-month span of the project, I made my work hours reflect his time zone. This shift allowed us to communicate via Skype on a daily basis which meant a fair share of late night and early morning calls for me! It was a sacrifice, and I would do it again. I understand that sacrifices need to happen to gain successful outcomes."

"The most challenging problem that I encountered in my career was when my former company experienced a major merger. It was a lot to adjust to but, after some time, I was able to get a good pace again."

"The most significant challenge I have faced as a manager would be the labor dispute and lockout that our company went through in 2016. Many of our permanent employees are union based. We could not come to a new collective agreement, and so I ended up having to utilize a lot of temporary staffing options during that time. It was a lot of re-training, and strain on the company culture overall."

"The biggest challenge that I face as a marketer, and it's an ongoing challenge, is to manage my expectations on projects. I lean on the side of perfectionism and often put more pressure on myself than even a client would. The positive side of this; however, is that I always deliver an immaculate product."

"I'd say the most challenging problem I have encountered was when my manager suddenly resigned. I was then in charge of the department. Now, I was mostly ready for the responsibility, as the assistant manager in the department. However, I had never completed inventory reconciliation, and on the first day, this was my first task. I was asked to give projections so that our buyer could stock us for next season. I had no idea what to do, so I researched until I came up with the answer. Also, other managers in other departments helped to guide me. Ultimately the work paid off because our next season projections were perfect. Since then, I've learned more effective ways to do our inventory management and projections, but I don't think I've ever learned anything as quickly as I did that week."

"The most challenging problem I've encountered is the misstep of taking my current role. The initial pitch to me on company growth and my duties is not my reality. This factor has been a challenge to my career growth. I know that even if it was a misstep, there are lessons to be learned, and I approach each day with interest and a positive attitude to try to learn those lessons and grow professionally."

"The most significant challenge I've faced is nearly having my department eliminated due to budgetary cuts. I was lucky to have an active parent community rally behind me and the department which saved the program, in the end. The other challenge that comes to mind was getting back into the swing of teaching after taking a few years off to be home with my children. There was a learning curve on getting up to speed with curriculum and the lesson planning, but my love for teaching made it all that much easier!"

Next Question

25 Problem Solving Interview Questions & Answers

Below is a list of our Problem Solving interview questions. Click on any interview question to view our answer advice and answer examples. You may view 5 answer examples before our paywall loads. Afterwards, you'll be asked to upgrade to view the rest of our answers.

1. Tell me about the most challenging problem you have encountered in your professional career.

2. in your opinion, what makes you a great problem solver.

Employers want to know that you have a methodical approach to problem-solving. Consider the skills and qualities that help you successfully face problems. Perhaps you have a keen eye for detail. Maybe you can see opportunity when others can only focus on the issue. Share your strengths as a problem solver, and your ability to come up with innovative solutions. Strong problem solvers are: - Systematic thinkers - Open minded - Okay with being wrong sometimes - Always researching and exploring - Able to identify the core problem - Able to reverse engineer a challenge to avoid future issues - Able to come up with multiple avenues that work well for all stakeholders - Are do-ers and not worriers

"I am a great problem solver because I can compartmentalize all aspects of a problem before studying it. I also like to bring more experienced team members in to add to the solution. I will never try to be a hero and solve a complicated problem without tapping into the resources around me."

"What makes me a great problem solver is that I have a keen ability to research, read, and explore so that my recommendations are based on fact and study rather than guesses."

"I have been told that I am an excellent problem solver and I believe this is because I have a bit of an engineering mind. I can take the issue, work backward to solve it, and then use that resolution as a basis for avoiding future issues to come up. I am also a big-picture thinker which allows me to come up with various resolutions per problem."

"I am a great problem solver because I do not allow stress to cloud my judgment and mute my creativity. I am a keen observer with a great memory which allows me to recall unique solutions or ideas."

"I am a great problem solver because I draw from the experience of others, whether solicited advice or through my prior observations and then I improve upon that, where possible. My memory and years in the industry have exposed me to many types of situations and problems, so I feel I have a vast amount of experience to draw from, allowing me to be creative and effective in the way I approach any challenge. Not to mention, I'm not afraid to ask for help or advice along the way. I know that I don't know everything, so I like to ask for input when I feel I am not fully equipped to do the job alone. There is no shame in that."

"I believe I am a great problem solver because I am sure to gather as many facts as possible, I look at the problem and its potential solutions from multiple angles, and I am not afraid to make a creative decision, that might seem off the beaten path."

"I consider myself a great problem solver and believe my skills are in my emotional intelligence. I can be really in tune with the tone of the group, who is feeling what, and how they are each best reached. This skill applies to both adults and children, so it is beneficial both inside of the classroom and out! By being aware of what is at the heart of the matter and how each person needs his or her needs met, I'm able to accomplish a lot while avoiding many common landmines."

Anonymous Interview Answers with Professional Feedback

Anonymous Answer

problem solving type of question

Cindy's Feedback

3. Tell me about a time when you discovered a problem and went beyond regular expectations to fix it.

Your innovative approach may be exciting and unconventional, but can you implement it realistically? Ideas are one thing, but putting them into practice and providing measurable results is where you can add genuine value. Think of a time you worked long hours and made sacrifices to overcome a challenging problem. Demonstrate your impact and the significance of your solution.

"During our busy tax season I noticed that one of our primary spreadsheets was not formulated properly. I am not an expert with Excel; however, with everyone being in peak stress mode - I decided it was something I could learn on my own. I watched a few online tutorials and ended up resolving the issue without the need to involve the rest of the team."

"When I worked as an admin assistant at my last job, I was in charge of purchasing office and kitchen supplies. I noticed we had been spending quite a bit of money on paper and plastic-ware. I compared the cost of disposables to the cost of buying permanent dishes and utensils for the kitchen. It turned out we were able to save the company hundreds of dollars each year by simply investing in dishes and silverware!"

"I had a staff member who was stealing supplies. Rumors were going around that she was dishonest; however, there was no evidence. I carefully waited and, after two days, the rumored infractions were caught on camera. At that point, I was able to terminate her employment. I went beyond regular expectations by gaining evidence before terminating her. I knew this would prevent a human resources issue down the road, and it also saved my company from having to pay this employee any severance pay."

"Our agency performed a major client launch last month that tested well. Upon implementing, I noticed that their new website was not functioning correctly. I wanted our client to be happy with our services, so I worked late into the night with our IT team to troubleshoot the site and ensure that by morning, there were no more kinks to work out. In the end, our client was thrilled with my dedication, and they ended up writing an amazing review online and even mentioned me in the review!"

"I managed a coat department previously and, depending on the season; these coats were very high ticket items. I had two salespeople who were consistently battling for the sale. It was unbecoming, to say the least, and impacted the department's morale. To incentivize everyone to go for the sale, I made a sales contest on non-coat merchandise. The more items they upsold, despite being a smaller sale, the more tickets they received towards various other compensation incentives like gift cards or extra time for breaks. The other sales reps felt reinvigorated, and it pushed my two coat-fighters to step outside of their perceived territory."

"In my first role, there was a regular lane of shipments that was difficult to cover. The issue didn't cause us to fall short as far as the customer was concerned. However, we were in danger of potentially having the customer poached due to waiting times. After several late nights attempting to come through for a key customer, I got tired of running in a hamster wheel. I decided to find some carriers that could assist. Long story short, after staying late many days and making some creative calls to find a backhaul, I was able to secure a new carrier, at a great rate, and keep the customer happy."

"When I was reworking lesson plans, I noticed that there was a gap between the programs and some policy. So, rather than hand them back to the team to fix, I took it upon myself to write the remaining lessons and tweak the existing ones to make them cohesive. It took about seven days of working on my own time, but it was worth it when I saw the lessons in action during the school year."

4. Tell me about a time where you had to analyze a set of data and then make a recommendation.

Talk about your attention to detail and sharp focus when it comes to data and statistics. You may not consider yourself a highly analytical person. However, this is a skill that you have indeed exercised in the past.

"I worked for a financial firm last year and had a client who was looking for investment recommendations. I gathered data on the stocks they were interested in, sorting through 12-month trends and further historical data to determine the most promising returns. The client was happy with my findings, and my manager was quite impressed with the research that I conducted."

"My boss recently asked me to make a case for Oracle on Demand versus SAP Business ByDesign. Our business was growing so fast, and we needed a new CRM fast. I called both companies who took me through a webinar and a couple of online tutorials. I then gathered the data and made an informative PowerPoint presentation. My boss was very impressed with how thorough I was, and I was happy to learn something new!"

"Each time I onboard a new client, I analyze a set of data before I make any recommendations on their strategy. This data includes their current analytics, primary sales sources, key customers, and more. I have a formula that I follow for the most part to help me assess and then give the best strategic recommendations that I can."

"My current employer wanted to know the exact impact our social media campaigns were making. I gathered our Facebook analytics for him and created a short PowerPoint presentation from the data. My recommendation was to increase our keywords in the geographical areas where our ads received the highest click-through rates. My research and recommendations certainly helped as our Facebook reach grew exponentially."

"As department manager, I'm responsible for forecasting what our sales will be for the upcoming season so that our buyer can accurately purchase the proper inventory. I have to look at our current inventory, last year's trends, YOY growth, and what the industry is doing as a whole, especially with the impact of online retailers. I then make a recommendation and forecast that will either set us up for success or not. If I under or over forecast, we end up with not enough inventory or too much to sell through and the cost is either opportunity in missed sales, or having to discount unnecessary items. To date, I've been nearly exact in my predictions."

"When doing annual reviews with my clients, I would analyze the past year's shipments, trends, and overall data. I would then make recommendations for improved efficiencies, rates, and better service contracts in the upcoming year. I would make not only carrier recommendations based on service level and pricing, but also made suggestions on new routes or ways in which we could be creative, like consolidating the shipments in our warehouses, to save cost when possible. I managed two of the most significant accounts in the office, so my recommendations were fundamental to our bottom line, and I'm happy to report that they were consistently adopted, resulting in more business."

"I am responsible for analyzing the results of our unit tests given across the department quarterly. I had not only to compile the results and make recommendations as to what units to keep and what to remove for the following year but also diagnose what ineffective and how we could remedy that. This task is a critical one as it shapes the future of the department and our efficacy as teachers."

problem solving type of question

Stephanie's Feedback

5. When a problem requires a quick solution, how do you respond?

When it comes to complex problem solving, decisions are not always readily reached. It takes practice, experience, and confidence to learn what sorts of decisions yield the best results. Walk the interviewer through your process when it comes to making quick decisions. Do you rely on past experiences? Perhaps you go with a gut feeling. Maybe you have read case studies that you lean on in these instances. Problems that require you to act quickly can be emergency situations such as knowing where the fire extinguisher is and grabbing it fast enough to put out a small grease fire in the company kitchen. Other quick decisions could be if you are asked to take on a new responsibility and are only given five minutes to decide if it's something you are prepared to take on. Going with your gut is a skill, and the more you learn to trust your intuition, the easier it becomes to make these types of decisions. Demonstrate that you are confident and able to react swiftly when the need arises.

"Our Controller recently came down with pneumonia on a week where we had a major client presentation to give. He sent me what he had prepared, and I had to fill in the blanks. As an Analyst it was a bit out of my wheelhouse, being in a client facing role, but I adapted quickly, and reminded myself that my team needed me."

"When an urgent problem arises at work, I always try to respond in a calm and assuring manner. I am a natural leader which means that my team often looks to me for answers. One instance of my fast-thinking was just last week when we had an administrative employee no-show on a significant day for us. I called a temp agency, and they had the position filled in just one hour."

"In logistics, there are often split-second decisions that can either get the freight to a customer on time or cause a shut-down of a production line. Sometimes, these decisions have to be made after hours. On more than one occasion, I've received a phone call from our central dispatch asking me how to handle a late driver. I have to remember the details of the particular shipper or receiver, my customer, and the actual load in question but also get creative with how they can make sure to meet customer expectations. Due to the urgent nature of the business, as well as the drivers, it has to be a very quick decision to be successfully resolved. Luckily, due to following my gut, I've been able to make very fast, split-second decisions in the best interest of the branch and customer."

"As a Marketing Director, I need to make a multitude of decisions, on the fly, for varying projects. I rely partially on the instinct that I have built as an expert in the marketing industry and part in past experiences that may be similar. I am sure always to exude an air of control when making decisions."

"I thrive under pressure and always have, so when I'm given a time-sensitive situation to address, I light up and get down to business. I am more impactful and even more creative when I have little time to do much besides jump in and take charge. This ability to make fast decisions is especially helpful in my role as manager when there is an inventory, personnel, or customer issue."

"Just like with negotiations, I react swiftly in emergency situations. Perhaps my skills come from my years as a parent, having to think fast and put out fires! If a quick solution is required, I will do a fast overview of the facts and make a decision based on risk factors considering the potential financial loss."

"I am certainly a take charge and tackle a project kind of gal - as a teacher and a mom, too! I feel I have a powerful and accurate intuitive sense and I follow it instinctively. It's very rarely steered me wrong."

6. When it comes to problem solving, are you a strong collaborator?

Show off your teamwork skills by giving an example of when you successfully collaborated with your coworkers. Be sure to demonstrate how you communicated your thoughts or opinions. Highlight how your contributions, or ability to ask for help, made a difference. Explain how you are a team player who enjoys working alongside others.

"Last month, I recruited a couple of coworkers to help me solve a problem for a client. We were looking at their financials, but something didn't add up, and I didn't have the analysis background that these two co-workers had. Together we molded our areas of expertise and created a bulletproof financial plan for our client. I enjoyed the collaboration and would do it again in a heartbeat."

"I am most certainly a strong collaborator! Being an executive assistant, I am often in need of strong collaboration to complete a project for the VP who I support. I love learning new things from my coworkers and those who I report to."

"I love having impromptu brainstorm sessions with my team. It keeps everyone on their toes! When an issue comes to light, I will approach the problem with the entire team and open the floor, at the end of the meeting, for suggestions."

"In marketing, it is imperative to collaborate and gain different sides of the story, and new opinions. I try to seek out my team's opinions on projects all the time. I find everyone has something to contribute and can help me see a problem or strategy in a way that I may not have ever considered."

"I would consider myself an active collaborator and believe that two heads are almost always better than one. Three is the best, in my opinion. This way the team is odd-numbered, so if there's a dispute you can take a vote on it! Multiple viewpoints are almost always a great idea."

"I am a strong collaborator. I am always willing to listen to others' opinions, hear their perspective, and work together to build a solution that will fit for everyone. I am always looking to draw from others' experience and expertise to bring about the best solution for the client and the branch as a whole. When drafting a pitch for a client, I am always sure to bring on a manager or carrier sales rep so that I will have multiple perspectives to help bring us to the best collaborative solution."

"I believe I'm a skilled collaborator and am confident that my coworkers would agree. I come to our bi-weekly department meetings full of ideas and with an open spirit, ready to collaborate with the rest of the team. We always have engaging discussions that result in great takeaways for the teachers as well as our students."

7. When you cannot seem to find the right solution to a problem, how do you deal?

Sometimes, problems just seem too impossible to solve, at first glance. Your creative problem-solving skills may be at a stand-still from time to time, and the interviewer wants to know how you deal with that. Taking a brief break and stepping away from the problem can help you to see things from a different perspective. When you are in a rut, you can waste time plugging away at something, resulting in a decline in productivity. Discuss with the interviewer how you handle being in a rut like this.

"If I am stuck on a particular problem, I will take a break from trying to figure out what's wrong and ask a coworker for advice. Getting another person's perspective when you start to feel like you're hitting a wall can help one to see a problem with a fresh set of eyes. As humans, sometimes we overthink! The biggest hurdle can be asking for help, and I am not above asking for help when I'm stuck."

"If time allows - I will sleep on it! When faced with tough decisions where an answer does not come to me easily, I will take a moment to feel the issue out. When necessary I will also bring in the opinion of the administrators in a different department."

"If I cannot come to a solution that feels right I will check in with other leaders whom I work with and, depending on the situation, my business mentor. It's important to check in with those that I admire as they have unique ideas and some have more industry tenure as well."

"As a marketer, I am hired to find the solution for others. As you can imagine, when that solution seems elusive, it is incredibly challenging for me to accept. For this reason, I love brainstorm sessions with my team. I will also look to the outside in the form of resources online such as blogs and forums by other marketing professionals."

"It can be frustrating when a solution does not come fluidly. However, sometimes trying a solution and seeing it fail, will lead you to a lightbulb moment. I am an active person, so I like to walk and talk things out. Usually, as I do that, I don't filter my ideas. This way, something slips out that I would have edited out as "ridiculous" if I were writing down a list. I have found that this free-flowing problem-solving session often leads to the most creative and impactful solutions which I would have nixed from the get-go had another not failed."

"If I'm stuck on a problem, I try to take some time away from the issue, ideally by taking a step away from the screen and get my blood flowing. Walking away seems to help me get reinvigorated and more creative. I also find it valuable to talk it out with someone, even if that person is not a stakeholder in the situation."

"If I am stuck in a rut or can't seem to figure out the best approach, I am fortunate enough that I have so many other tasks and classes that I can focus on. Usually, if I clear my mind and fill it with something else, a great idea hits me when I least expect it. If I am stuck on a problem and cannot take the time to step away, I usually rely on my students to help me shake it off!"

8. When faced with a problem, how do you decide on the best solution?

There may be more than one solution to a problem, and the interviewer would like to know how you make a final choice when you're in a situation like that. Effectively comparing and contrasting, or weighing the pros and cons, is essential when choosing the best way to solve a problem. The interviewer wants to see that you are capable when it comes to calculating risk vs. reward. Think about a time when you have compared the risk and reward to a potential solution.

"If I have a problem with multiple solutions, I always go back to the classic pros vs. cons method. I fully understand that although no solution is perfect, and some solutions offer lesser sacrifice while others pose potential loss. I have been trained to take the solution that is 'closest to the money' which means that if I am stuck between a rock and a hard place, I will choose the solution that is most beneficial to the company's bottom line."

"When it comes to problem-solving, I will always weigh the pros and cons before making a decision. I will also bounce my thoughts off of some co-workers if I still feel conflicted after that."

"My decisions are always based on three factors. One, what is best for the company. Two, what is best for our clients. Three, what will boost employee morale. Now, not all decisions will be popular with all three groups, and I do keep that in mind. In those instances, it is my job to watch our bottom line but ensure customer satisfaction at all times."

"Rock, paper, scissors! Kidding - of course! Our team will collaborate on tough decisions, and we often vote. Majority wins in our office for many creative decisions."

"When I face a problem, I am sure to draw on previous experiences both as a customer and an employee in retail. I then use these experiences to make the most informed decision that I can about the problem at hand. Generally speaking, if I've already seen or experienced a very comparable situation, I can be impactful and exact in my approach by drawing from those experiences."

"As I consider a problem and its solutions, I make a note of what my gut tells me what to do. Then I take a step back and reflect on times that I have faced the situation before. I recall the actions that I took, the outcome, and then pivot as necessary. I trust my instinct because I am heavily knowledgeable in this industry, but I believe in relying on fact as well."

"I am typically a follow-my-gut type of person, so I follow my instinct when possible. I make a note of what my initial inclination was and then I make sure to compare and contrast solutions. Once I have identified the best solution, I check in to see if it feels right. More often than not, my initial instinct is correct. Of course, I am sure to be analytical as I weigh out each decision."

9. How do you prioritize multiple projects when they all seem equally important?

Prioritizing is a skill that requires practice. There are many approaches you can take. Here are some suggestions: 1) Make a list. By thinking through and writing down each item that needs completion, you can see it on paper. 2) Mark what is urgent or essential. Take into account deadlines and meetings. 3) Order each task based on effort and estimated value. 4) Consider due dates and how long it will take to do each item. When answering this question, show the interviewer that you have a system in place that helps you to think through what needs to happen, and when. The better you can prioritize, the more productive you will be, making you an asset to their company!

"I aim to be as effective and efficient as possible and make sure I can use all minutes of a day for a project. I have a few things going at once most of the time. I am the lead on some, the delegator on others, and the reviewer on another, for instance. This way, by splitting up the work to the appropriate parties, both my team and I can be the most efficient with our time."

"I often have multiple projects due at a time, since I am the assistant to three different executives. I ask my executives to rank their need from 1-5 in the level of urgency, including its due date. I start my work on that list. If there is more than one urgent need, I will work overtime, or through my lunch, to ensure that I deliver everything on time."

"I had to utilize creative problem solving last month when we found ourselves short-staffed and unable to hire new employees due to budget cuts. I changed our schedule to include some split shifts and received approval for a small amount of overtime spending. The problem is solved, at least temporarily, until our company comes out of our spending freeze."

"In my current department, we are very systematic in our customer delivery promises; however, that is not to say that doubling up on client deliveries does not happen. When situations occur where I have to prioritize, I will do so by the size of the client and budget. It may seem unfair at times; however, our largest clients with the most significant spend always rule out."

"I prioritize based on urgency and time required for the project. I have a list of what needs to be done, by when, and how long I estimate that it will take to accomplish. I am great under pressure, but try to make sure that I don't get myself or my team into a sticky situation by not allotting enough time for any particular project."

"I love to keep running lists of everything that I need to do, big or small. Mostly because I love crossing things off of the to-do list, but also because it helps me keep track of everything. Lately, I've started utilizing a free project management software that I use to make those lists, categorize the tasks, and mark them by the level of urgency. I take care of the most time-sensitive issues first and then move along to the equally important, but perhaps less time-sensitive to-dos. I also estimate how long each task will take, so if I have a few minutes in between projects, I can tackle the quick to dos and use that time effectively, rather than use it to figure out 'what's next.'"

"I follow the tried and true practice of making lists and assigning each item a priority and tackling the list that way. I love to check things off my list, as it gives me a feeling of accomplishment. Also, I am a believer in following my intuition. If I feel that something lower on the to-do list needs to be bumped up in priority, I will tackle that right away. As a teacher, there are always a lot of simultaneous to do items, so in addition to prioritizing, I have to be good at multitasking; something I find I do quite well as both a teacher and a mom."

problem solving type of question

Marcie's Feedback

10. Tell me about a recurring problem that you run into in your current position, and how you handle it.

The interviewer wants to see that, despite this recurring problem, you take action to find a resolution. They want to make sure they aren't hiring a chronic complainer who is easily defeated! Be careful to avoid complaining about your current (or most recent) position. A recurring problem could be a glitchy software system, an employee who is regularly late, or even an unpredictable work schedule. Remain optimistic in your reply!

"A recurring problem that I have in my current position is the fact that our client management software is not user-friendly. Any entry that I need to make is incredibly time-consuming which poses a real problem when a deadline is present, or when we have clients waiting for an answer. I have found that the best workaround for this is not to allow my paperwork to build up. The more proactive I am, the better I can keep ahead of schedule."

"A problem that I am currently running into is a lack of office supplies. My boss has been running very lean, financially speaking, since our industry took a downturn. I have to time my ordering with client invoices at this point. This situation has certainly helped me to become more aware of spending and budgets, that's for sure!"

"The greatest issue in my current position is that we have so much employee turnover. It started to feel like I was constantly training new staff. I came up with an employee referral bonus program which greatly helped. For every successful referral, our employees get $400 plus another $400 after their referral stays for three months. I believe this has been successful because the quality of our employees has greatly increased."

"In my current office, we have more clients than we can handle - which is a great thing! However, it's been tough to find the best marketers to join our team because we are a small organization. This hiring situation has meant a lot of overtime hours, which I am certainly happy to do for the most part. I do look forward to working with a bigger team, like yours."

"Unfortunately, a recurring issue in my current company is employee tenure. It's just really part of the industry as we need some holiday and seasonal associates and they typically don't want to stay on, or we don't have the budget to keep them on. This turnover means we are continually becoming a new team and learning how to work with our new coworkers. Scheduling often has a learning curve with a new team, too, because you have to take into account the availability of all parties, and who works well together. That said, it's something I'm used to. I make it a bit of a personal challenge or game for myself. How quickly I can learn their available days, how fast I can learn who works best together."

"A recurring issue at my current job is lack of reliable inventory that my clients are requesting, which can be incredibly frustrating. I am working hard to land a client, get them to buy into our program, both literally and figuratively, and then we fall short of expectations when our inventory doesn't meet their standards. That said, I continue to go out, land new clients, and try to source the proper inventory for them."

"A recurring issue revolves around my lack of a classroom and the friction that can arise at times because of it. Without the flexibility of my own classroom, I sometimes find myself in an awkward situation since I have to abide by the other teacher's rules, which sometimes conflict with mine. I do my best to follow the teachers' class rules, and make sure that we have a good understanding."

11. Tell me about a time when you failed to solve a problem. How did you overcome the failure?

'Success is bouncing from failure to failure without losing momentum,' or so they say. Your resilience shines through when you can learn from your mistakes and keep going. Give an example that shows you can accept fault and learn from challenging experiences.

"I failed to meet an important deadline in my first job out of college because I didn't know how to prioritize properly. I kept letting other menial tasks get in the way rather than focusing on finishing the project. I learned how to manage my time wisely by setting reasonable goals and reminders on my calendar. This technique helped me to manage my time more effectively."

"Last month we were having issues with our GoToMeeting application, and it was right before a major client meeting. I was on a call with the service provider, trying to troubleshoot and unfortunately, did not deliver a fix on time. After the initial frustration, I decided to talk to my boss about having backups in place. Now, we have Skype, and Google Hangouts set up for these emergency situations."

"I was asked to solve our issue of employee turnover which ended up being much more difficult than I originally thought. My initial goal was to improve turnover by 70% but in the end, only reached 40% improvement. Although I did not reach my goal, I am still happy that my action plan made a difference."

"I had a customer who was not happy with my delivery, and I chose to take care of the situation without involving my boss. It wasn't that I was trying to sweep the situation under the rug, I just honestly thought I had been successfully dealing with the situation on my own. Unfortunately, I was wrong because the client sent a nasty email to my boss a short time after. I should have gone to my boss right away and filled him in. It's something that I've learned from, and I'm ready to involve my boss with every sticky client situation."

"In a previous role as a personal shopping assistant, I was tasked with taking on a notoriously difficult client. She spent a lot of money in the store in the past but was very demanding. This challenge seemed like the perfect opportunity to prove myself. A few months in, I made the misstep of mentioning something she'd complained about at an earlier date. Apparently, she was offended that I brought it up, even though I meant it very innocently. I owned up to it immediately to my manager and came up with a plan to win her back. I wrote a snail-mail card apologizing to her and let several weeks pass before reaching out in any other way. By the time I did, two months later, she was perfectly lovely, dismissed my apology as though she didn't know what I was talking about, and we moved along in a better fashion than we had prior."

"In my first role out of college, I was working to solve a lane issue with a carrier that kept falling through. I went through every solution I could come up with including pitching consistency, to leveraging my current relationships, and asking for favors. Those favors and workarounds ran out, and we fell short of client expectations. While I did all that I knew how at the time, I still fell short, and it was disappointing. In retrospect, I would have involved more people in supervisory positions earlier on in the process to learn from their shared experiences."

"The problem I've failed to solve that still keeps me up at night is a successful inclusion of one of my students with an IEP. He loves Spanish and in a one-on-one setting excels at it, but cannot handle the behavior expectations in class because he gets too excited. I've tried multiple approaches to get him to regulate, and participate, but so far nothing has allowed him to participate in the class without disrupting the other students and causing a meltdown for himself. This fact weighs on me since I want him to experience inclusion at all times. As a result, he comes to my office a few days each week, and we have our Spanish class together. I feel this exemplifies who I am as a teacher. I will go the extra mile for my students to make sure they get their fair shake at life."

12. What sources do you look to when you need to solve a complicated problem?

The interviewer wants to know that you can think outside the box, or even ask for help when you are stuck on a complicated problem. Maybe you look to a mentor or boss for advice. Perhaps you have handbooks, manuals and systems you turn to for help. Offer some relevant examples based on your industry. If you work in the medical field, you may turn to textbooks, online research, colleagues or even patient's history to find the right solution. If you work in customer service, you may ask the customer what they need to find the best way to solve the problem. Show the interviewer that you are knowledgeable and equipped to handle these types of scenarios.

"When I am faced with a complicated problem, I will look to the resources that my current company has provided me. The answer is almost always in there. If it's more of a moral dilemma vs. a knowledge-based dilemma, I will ask my supervisor for his thoughts and opinion since I value him as a mentor and expert in our industry."

"I have a variety of manuals and online tutorials that I lean to when I need to solve a complicated problem. Usually, the issues are surrounding Excel troubleshooting, so it is easy to find answers without involving anyone else and interrupting their day."

"I have a business mentor that I turn to for significant problems. She and I are in the same industry; however, she is much more tenured than I am. I recommend that everyone have a mentor. Even though I run a team of my own now, there are times when I do not have the answers."

"When I need to solve a complicated problem I will turn to marketing forums and blogs that I follow. There is a plethora of information on the internet, and it would be a shame not to take advantage of them!"

"To solve a complex issue, I will reach out to a manager or mentor from a previous role to ask them how they've handled such issues in the past. I am always ready to dive back into our handbook, but these types of scenarios are often not covered there, which is why I value a human, experiential approach. I know that there are so many folks in the industry who have so much to teach me and have probably already "been-there-done-that," so I love to utilize them as a resource."

"If there's a complicated problem, I'll write out what I think the possible solutions would be. Then, I will weigh those potential solutions against one another and list the complications that may arise as a result of each choice. Also, I am always open to input or suggestions from those with more experience than I. I will often turn to my organizations' training resources, as well as talk the problem out with coworkers or my boss."

"I have a vast cohort of teachers with whom I work currently, or have worked in the past, so if I am stuck on a problem or feel I need some additional help, I reach out to these educators. If nothing else, they're there to lend an ear and let me bounce my ideas off of them. They almost always have some real-life experience in a very similar situation. I value this collaborative, supportive group that I've amassed over the years."

13. After you implement a solution to a problem, how do you test the effectiveness of that solution?

The interviewer wants to see that you have strong follow-through skills and the ability to use data and analytics to support your decisions. The only way to test the effectiveness of a new solution is to keep a close eye on the immediate, and often longer-term, results! Depending on the situation, you can use data, run reports, and compare/contrast your findings. If you have records of the data before your problem-solving solution, you can track the results of your new solution and analyze in a month, or beyond. It can take time to see the results, so having a method for measuring them is essential. Give an example of a time you implemented a solution and found a way to measure the results to check its efficacy.

"Last year, our company was having a very high rate of turnover due to employee burnout during overtime hours worked. I implemented a third shift which alleviated the need for excessive overtime. Yes, it did increase our payroll costs by 33%; however, it decreased our turnover which was costing us more and more every year. From the analytics I have been watching, the change will pay for itself by the end of year two."

"One solution that I recently implemented was the use of Google calendar with the executive that I support. She was rarely updating her Outlook calendar because she found it to be too difficult to do on her smartphone. Since this implementation, we have minimized our crossed wires significantly! I have measured the effectiveness of this new calendar strategy by marking down any appointments that need to be rescheduled. So far, for the month, the number of reschedules is down by 80%."

"I always look at the data to gauge the efficacy of policy or new solution. I am big on numbers as they do tell the full, and true, story. I love the reliability of spreadsheets and numbers!"

"Once our team comes up with a new marketing strategy for a client we will conduct two focus groups. One test group will be on the original marketing plan and the second, on the plan that we want to pitch. The use of focus groups is the best way for us to measure if our new strategy will be effective enough to justify the changes for the client."

"I like to collect data, as well as anecdotal assessments of new policies. It's great to have data to confirm if it was or was not effective, but I am a firm believer, too, in getting the team on board. Plus, as you implement a solution, sometimes those doing the actual day-to-day work with customers or in the actual implementation have a more accurate understanding of what's going on or what could be improved. Therefore, I am sure to ask the staff how they think it's going, if it's impactful, or what they still see as an area for growth."

"To test the effectiveness of any solution, you have to be objective and see if it genuinely addressed the problem it set out to solve. Everything in our business runs on KPIs, so when we introduce any initiative, we can see how it is or is not impacting those measurements. One example of this was when I assigned specific accounts to my team of buyers, instead of just attaching as they came up. The idea was to get a buyer to become an expert on that account, their buying habits, and therefore be more effective in the long term at sourcing for their needs. At first, it didn't seem all that impactful, as the close rate was still around 42% overall. However, over the course of 10 weeks, we saw an uptick in close ratios on the assigned, dedicated accounts versus the randomly distributed ones, resulting in 53% close ratio. It's something that became so effective that other sales pods adopted it as their practice as well."

"For me, numbers play an important part in teaching but do not paint the full picture. So, after implementing a change, it is certainly important for me to collect data from our unit tests to gauge the efficacy of the lessons we're teaching and the lesson plans we are using. However, I also am sure to check-in with the students on a more regular basis to check for comprehension. Testing is only truly reflective of the way some students learn, whereas others are terrible test takers, even though they've learned the material. That is why I like to take a two-pronged approach."

14. When a major problem arises, what is your first reaction?

The interviewer wants to know if your reactions to problems reflect maturity and professionalism. How you react will significantly determine how you fit with their existing team. Perhaps your computer crashes, and you realize you may have just lost all of your hard work. Or maybe you are limited on time and have a deadline rapidly approaching. Demonstrate to the interviewer that you take a very methodical approach to problem-solving, rather than reacting impulsively when a problem occurs.

"When a major problem arises, my first instinct is to take a step back and absorb what just happened. I then go into 'brainstorm' mode, jotting down potential ways to resolve the issue. From there, I can use a pros and cons list to determine the best course of action for a fast and amicable resolution."

"I have taught myself to become much calmer with my first reactions when problems arise. Now, I will step back and review my options for solving the problem rather than allow myself to become frustrated. If I feel that I cannot solve the issue on my own, I will ask for help from my superiors."

"Depending on the situation, I will gather my resources and team and collaborate on making the necessary happen on a shortened timeline so that we can deliver our results in the most efficient manner possible. Usually, we learn something about ourselves, the team, or a more effective approach to the next problem in the process."

"When a major problem arises, my first instinct is to jump in and fix the issue. I am a do-er and also think in a reverse-engineering manner. I start with the desired result, and work my way backward from there, figuring out where the snag occurred."

"I am resistant to stress but cannot completely avoid it. When a major issue arises, I will take a quick walk, if possible, so I can best assess how to address the issue while clearing my head. Then, I get to work. I delegate whenever possible so that I can oversee the effectiveness, but am not at all afraid to jump in and do the dirty work myself."

"In the event of a significant problem or setback, my first reaction is to freeze in disbelief for a moment or two while I gather myself, then I jump into action. I know that I need to work harder and faster to recover the time and effort lost. My salesperson mind goes into overdrive until the issue comes to a resolution."

"My first inclination in the event of a major problem is to roll up my sleeves and jump in to fix it or help mitigate some of the potential blow out. This initial reaction is especially true when the problem involves a student's feelings or wellbeing."

15. What steps do you take when you have to make an immediate decision without all the relevant information?

Sometimes we have to make decisions without all of the pertinent information at our fingertips. The interviewer wants to know that you are capable of taking educated guesses and that you are confident enough in your abilities that you can make a firm decision without all pieces of the problem being present.

"When I need to decide without all of the information, I weigh the pros and cons and come up with a solution that makes the most sense. Common sense can take you a long way! Next, I may ask the opinion of someone I trust to see what they think. Even though I trust my decision-making ability, I still think it's important to get a second opinion when it comes to situations involving money or decisions that make a significant impact on others."

"Being organized, I do have a checklist that I follow on all policy-related decisions and changes. If I do not have all necessary information to make an important decision, I can usually find answers in our company resource database, or I will consult an administrator more tenured than I."

"Immediate decisions are required of me on a daily basis. For instance, what do I do when a forklift driver doesn't show up for their shift? How do I react to a chemical spill in the warehouse? I find that the most effective method for making immediate decisions is to forget about what you don't know and focus on what you do know. That's the best anyone can do, and there is no sense wasting time on the what ifs, especially in my industry when the safety of others could be at risk."

"In my current company, we have a rule always to do what will make the client happiest. So, when I am in a situation where I need to make an immediate decision on a client file, I will ask myself what I would want if I were the client. Then, I jump into action to make that happen."

"Often when a customer is worked up, I only have a piece of the puzzle to go off of, whether because they haven't given the full story, or I'm pulled in by the associate who heard the full story. In either case, it's something I'm accustomed to and deal with daily. As far as customer problems go, they tend to follow the same general pattern, so I assess quickly what category the problem seems to fall in, and go from there."

"I am a strong believer in following my gut, and for the most part, it has not steered me wrong. I try to gather as much information as possible, but when all of the pieces are not accessible, I assess the situation using my prior knowledge of similar situations, and I follow my intuition. If I'm not certain or feel conflicted, I don't hesitate to bring in another person to help me come to the best decision for the company."

"I feel comfortable making an immediate decision, even if I don't have all of the relevant information, for the most part. I have great confidence in my situational knowledge as an experienced educator. One example that comes to mind was the class when there was a behavior outburst. I immediately leaped into action to diffuse the situation the best way I knew. By acting quickly, I can prevent the situation from further escalating."

16. How do you deal with distracting coworkers who stand in the way of your progress?

Even the most well-meaning coworkers can distract you from getting things done at work from time to time. The funny and entertaining coworkers who like to chat online and send YouTube videos are often the ones who can get in the way of your productivity if you let them. How do you respond? Show off your ability to set professional boundaries, when needed.

"I typically just set a kind, but clear, boundary and tell my coworker that I need to focus. I will offer an alternate time for a catch-up, over lunch for example. It is important for the sake of workplace culture to set aside time to be social with coworkers, so I usually just let them know when I'll be available for a quick break in the day."

"I understand working relationships are significant, and I'm sure to make time for them so that I can be useful but also enjoy myself at work. With that said, I know where these relationships fall regarding prioritization of my day. I make sure that others know that, too, without being off-putting."

"I am always interrupted by my team - that is par for the course being a manager. To deal with any lost time, I will simply stay late or come to work a bit earlier the next day. My day is unpredictable, and I have accepted that fact."

"I am very open with my colleagues and will let them know if they are a distraction. Currently, I can take my work home as well so if there is a part that I cannot get past due to distractions; I will take a day to work from my home office."

"I try to make the workplace as fun as possible, within reason. I love to make it a place people want to go to, instead of dread. That said, there are always the people that ruin it for the rest of the team by taking advantage. To combat this, I make it very clear what the expectations of allowed and prohibited behaviors are, and am sure to reinforce those expectations."

"There are always going to be co-workers who are there for the gab, rather than the work, or who are content just being in their position with no intent of advancing through the ranks. Early in my career, this bothered me. Why weren't they motivated to grow and learn? Then, I realized that it's important to have those people since a company can't have all its people always vying for the top. If there's a distracting coworker, I try to make my priorities clear and engage kindly and courteously with them as humans, and then get back to work. I am sure to remain friendly, while also firm, as needed, to communicate that I am here for work first as a priority."

"Very rarely do I find that my coworkers successfully distract me- even in a department meeting, I find I'm able to remain on task. I was always taught to ignore the behavior you wish to cease. If my coworkers are distracting and seeking attention, I try to ignore it as much as possible and only address it when it's detracting from a productive work environment."

17. Tell me about a time when you had to troubleshoot to solve a problem.

Troubleshooting is like reverse engineering - it takes skill, effort, and patience. You have to understand the problem to know how to work backward from it to find a solution. Knowing how to solve problems with technical equipment is always a solid skill, and a great way to demonstrate your example. Show that you are insightful in your approach.

"Last week, while operating the ultrasound machine, I was receiving a repeated error. I entered in a few different codes, but that didn't solve the issue. I then did a hard reset, removing all power sources. Then, I referred to the online manual for additional suggestions. It took a little time and patience, but I was able to resolve the issue without calling a technician."

"We do not have an IT department in my current office so whenever an issue arises, I am the person that my team calls. Troubleshooting is fun for me - it's like a new challenge every time. Google and IT forums are often my best friend!"

"We had a major complication in our system and our entire production line shut down. Our network administrator could not be reached so I had to go old-school and manually enter the orders so that my team could continue with production. The entire debacle lasted half of a day, and my system worked well as a placeholder."

"One of our clients called me in a panic, saying that Facebook rejected their ad campaign that we so carefully crafted. I researched on ad policy forums and learned that it was not approved because we did not set our demographic targets to people only over the age of 21. The ad was for a craft beer company, and we did not put into consideration the legal age in most states. Once I was able to narrow down the issue, I tweaked the ad, and it was approved."

"One horrific day at work, our systems went down entirely. We had no backup for how to check customers out, so I had to dig in the deep recesses of the back room and find the card imprint machines, and we wrote out tickets by hand and made imprints of the cards. I tried all the usual tricks to get our registers up, but couldn't get them to come online as it was a network error. I found the way around it with the handprint cards and then opening the cash drawer with a key."

"In a troubleshooting situation, I approach it like a maze and work backward. There are usually multiple factors contributing to any one issue, so I try to discern what they are, weigh those out and try to conclude what the potential best solution is. As far as technically speaking, my go-to in many situations, as rudimentary and childish as it may be, is often turn it off and turn it back on. Ha. I know it sounds too simple, but it often works best."

"I do everything I can to test out the technology before I bring it into the classroom- the day is so packed that we don't have any time to spare on figuring out technology if it acts up. I also always have a backup plan in mind in case the smart board or whatever we're utilizing that day doesn't cooperate, so we don't lose precious learning time." However, I believe that troubleshooting applies to more than just technology. Problems that occur offline also need troubleshooting as they arise, including figuring out a lesson plan and how it works or doesn't. It's all about working backward to see what issues, if any, may arise in its implementation during a dry run. By preparing in advance and being aware of what issues may come up, I'm able to flush out problems that would have otherwise arisen during the class time. "

18. Tell me about a time when your analysis of a problem was deemed to be incorrect. What would you have done differently?

Everyone makes mistakes when analyzing a situation. The interviewer isn't concerned with perfection; instead, they want to know how you deal the aftermath of rejection! Sometimes you can't correct your mistakes, but you can certainly learn from them. Highlight your ability to learn from your mistakes and move on, professionally.

"It was my first job as a physician's assistant, and I was trying to diagnose a patient who had severe pain in her abdomen. After running some tests, the doctor and I believed she was suffering from a gallbladder problem. We treated her, but she came back to the ER a week later. It turned out she had a problem with her pancreas. Even though we misdiagnosed her initially, we were able to use this mistake to help us identify the real problem. I've learned that sometimes making a mistake is a part of the process of solving a more complicated problem."

"I was asked recently to work on balancing an accounts receivable report. Math is not my strongest suit; however, I was confident in my ability to make it happen. Through a bit of research, I carefully worked on the document and was quite proud of my result. It turns out, I skipped a few important steps, and my work was, in fact, incorrect. I took it as a learning opportunity but also realized that my strengths are in other areas of business. I should have asked for the project to be placed with someone else, but I do not regret trying."

"We had incredibly high turnover rates when I first started in my current role. Going in guns blazing, wanting to make a strong first impression, I did a complete overhaul of the training manual thinking that was the problem. It turns out the training manual was just fine. The culprit to the turnover was one employee who was a complete bully on the job. The moment I terminated that person, the issue was solved. At least now I have a fancy new training manual! Moving forward, I now poll my team regularly for job satisfaction. I encourage a transparent workplace culture where people feel safe bringing their issues to me."

"I had a client, earlier in my career, who was not seeing the same results from their Facebook advertising as they once did. I changed the headlines, increased the budget, and even did multiple A-B tests. What I failed to see were the strategic algorithm changes that Facebook had made, which directly affected the visibility of my clients' ads. Now, I have alerts and subscribe to a couple of blogs solely dedicated to these changes, so I never miss a beat."

"Unfortunately, this happened not too long ago where I misjudged a customer complaint. The associate needed to escalate the conflict to a manager but did not accurately portray the customer's concern, and I jumped into action based off of the limited information given. Due to not gathering enough information from the customer herself or clarifying the misunderstanding with the associate, I took a misstep with the customer and did not resolve the issue as quickly as I would have liked. Ultimately, I was able to clarify the situation and get to a resolution that worked for everyone, leaving the customer happy. However I have some regrets. It was a learning process, and something I have been sure not to repeat since. Were I to do it again, I would clarify the situation with the customer, rather than taking the associate's word for it."

"When pitching an existing client on increased volume next year, I had made a recommendation on the most effective carrier for a lane. I based this recommendation on historical data and projected future rates. However, a merger occurred after the time of the pitch, and their prices skyrocketed since they were the only viable carrier for that area. Without competition, they didn't have to remain competitive in their rates. While I could not have predicted the merger, I could have quoted out with a higher margin on our part so that if there were some snag like this, we are covered. Since we lock in the rates for the customer, we took a loss each time they moved freight this way. As a backup method, whenever possible, I attempted to send the freight another way, so that we would lose some money but not take as large of a hit. That was a big learning experience for me and has helped me be better prepared to pitch other customers in a more effective, CYA type way."

"While teaching, the kids told me that I needed to quiet down at one point. I assumed it was the teacher whom I shared a wall with, that planted the seed, which was irksome. This type of situation had happened before. This time, however, I was wrong. I asked her to avoid delivering messages to me through the students, and she said she had not. Apparently, the students knew she had a headache that day, so they were all watching their volume level. I was glad that I did address the situation with the teacher, but made sure not to be accusatory or make assumptions about motives again."

19. Tell me about the most challenging aspect of your previous job. How did you overcome it?

Sometimes the most significant workplace challenge is a difficult task that puts you outside of your comfort zone. It could be something that requires skills you haven't mastered yet or qualities where you are not the strongest. Explain to the interviewer why it was difficult but be sure to spend more time highlighting the actions you took to overcome the challenge.

"The most challenging aspect of my last job was troubleshooting some of the older technology. We needed some serious upgrades, but they weren't in the budget. Learning how to work around this problem was quite a challenge, but I learned how by referring to old manuals and online forums. I ended up to become one of the stronger users of this program, in our entire office! I quickly became the go-to person when anyone had questions about the technology."

"In my current role, we have global offices that span four time zones. It is an incredible challenge to be continually calculating the difference in my mind when I call or email on updates for projects, for instance. I now have each locations time added to my desktop, my smartphone, and four individual clocks on my wall. These small and inexpensive changes made all the difference."

"The most challenging aspect of my previous job was the constant need to pivot when it came to trends in the industry. We would gain footing, and then the next greatest product would arrive. It made it difficult to feel loyalty to any of it. I started to express loyalty to the company's ability to discern great products instead of narrowing in on the products themselves. This shift in thought helped with mine and my teams' performance when it came to sales."

"My previous role was with a small agency where budgets were always a concern by the clients. Although I liked the clients, they were usually independent businesses with less than ten employees. They had a hard time thinking big-picture. I overcame this by coming up with a questionnaire that would address their greatest pain points and needs for their business. I would then focus on their small goals versus what I felt their company could be. Some business owners are more comfortable being comfortable, versus ruling the world, and that's okay! I just needed to wrap my marketing brain around that."

"The most challenging part of my prior role was navigating the landscape as the newest manager on the team. I needed to work on gaining the trust and respect despite my being green. I worked hard to build individual relationships with each associate and forge a bond with them. I also shared information about myself, including my experience in the industry, and who I am as a person. I know that this made me more human, approachable, and also solidified my credentials, so I know how to get the team on my side."

"The most challenging part of my previous job was relying on another team to be efficient. I am all for teamwork, but for me to be paid, this team needed to deliver timely and quality work. Meanwhile, their goals and metrics remained disconnected to any sales outcomes. This situation made it tough to motivate them. In the short term, I sat down with them and explained why it was vital for myself them, and the company that we work together on the same timeline. I incentivized them with coffee or store gift cards. Bigger picture, I sat down with the management of both teams and shared the issues we were having, suggesting a solution that would tie their KPIs and financial incentives to our outcomes, to make them invested. In the end, the short- and long-term approaches proved useful."

"I think the most challenging aspect of my current job is the fact that I share a classroom with another educator. Without having my space, other obstacles come up such as teachers trying to influence how I run my class, or them holding small conversations with their aides during my teaching time. I make sure to address this up front with the classroom teachers- that while it is also their room, please treat it as though it were mine during the 40 minutes that I am teaching. If there is ever an issue, I am sure to address it quickly and directly, so we can move past it."

20. When faced with a problem, are you more likely to jump into solving it, or are you the type to carefully assess the issue first?

The interviewer would like to know more about your problem-solving skills, and your personality. Discuss how you tackle problems when they arise, and keep your answer work-related if you can. Whether you are the type to jump right into solving a problem or you are more methodical in your approach, highlight to the interviewer that you are capable of handling issues professionally while using sound judgment.

"When faced with a problem, I am more likely to jump right into solving it. I believe that you cannot leave a problem to fester or become bigger than it already is. You have to take ownership of the issue, and involve yourself in the resolution right away. With that said, I am responsible for my decision making and certainly don't jump in blind. If I am unsure of what action to take, I will ask my leader for advice."

"I am careful and calculated in every step taken when it comes to problem-solving. This effort is because as an administrator, one error in judgment can throw off the timing of an entire project. I would say that I am the particular type who thoroughly assesses situations."

"As a manager, responsible for a team of 18 individuals, I need to be very calculated in most decisions that I make. I cannot act on the fly, or by emotion alone because others are relying on me."

"In marketing, I feel that I often have to do both. Some smaller decisions simply cannot be over-thought and others, especially when it comes to strategy, will need extra thought. I can provide both sides when appropriate."

"I think it depends on the situation at hand, honestly. In a familiar situation, I am ready to jump right in and tackle the problem. However, when the stakes are high, or tension is high, I am more inclined to take a step back, slow down, and be more tactful in my approach."

"I'm a "roll up my sleeves" kind of person. I see a problem, envision a solution, and begin to tackle it, figuring it out as I go and asking for help along the way. I think it can become a 'bury your head in the sand' issue, or the team will have the bystander effect, thinking someone else is going to take care of it, so I jump in and take action. I rally the troops, gather the appropriate supplies or resources as needed, and get to work."

"I'm the type of teacher who jumps in, head first and gets the work done. I know that the longer I wait to address a problem, the bigger it becomes, so I make sure to get right to it. This approach applies to interpersonal issues as well as curriculum missteps."

21. Give me a recent example of a valuable lesson you learned from a problem you faced at work.

One of the best aspects of problem-solving is that you always have the opportunity to learn from the experience. Seeing problems as opportunities to grow, is what makes you an excellent employee! Show the interviewer that you can learn valuable lessons when there is a problem at hand. Use a work-related example, if you can.

"Last month our sales team was facing a major challenge when we lost one of our primary distributors. I took action and started cold-calling, other potential distributors. I brainstormed with my team in some other ways that we could avoid a negative impact on our bottom line. We were quite successful in our recovery, and I would say that the biggest lesson I learned from the experience is that you are often only successful if you have motivated people in your corner."

"The most valuable lesson I learned from problem-solving at work is that not everyone will see your solution as the best one. Accepting change is difficult for some people, so I have found that not everyone will be on board right away."

"I recently had an employee express their disinterest in the job and the company. Rather than coach them out, I selfishly wanted to 'save' the employee. I put in extra hours mentoring and training her, just to see her quit anyways. It's a valuable lesson as a manager to put your energy into those who want to be there. Other efforts are often just a temporary fix for the inevitable."

"Marketing is always shifting so I often learn new, valuable lessons. One lesson I recently learned was to double check the documents that I send out for any needed updates. A lot of the manuals and how-tos that we send clients are evergreen; however, some are not. I accidentally sent an old social media guide to a client, and they ended up being incredibly confused. My lack of attention to detail at that moment was a bit embarrassing but lesson learned!"

"A recent learning experience was when I misjudged what the customer was upset about, and I didn't take the time to learn what it was that she was looking for. It reminded me to slow down, go back to the basics, and not assume that all situations fit the mold of the 'typical' customer. It was a perfect reminder that though I've seen most everything, I need to remember that each person and situation is unique."

"A recent valuable lesson for me has been not putting all of my eggs in one basket, as the old saying goes. Over 64% of my sales came from one group of stores, and they've always been a big contributor to the entire company's sales numbers. However, they were put on "hold" recently by their corporate, due to some restructuring issues. This event threw me for a loop. I was in real danger of not hitting my monthly sales target, and therefore I would have fallen short on my quarterly quota as well. I had to work extra long hours and hustle my other clients and fence-sitters to get them into "buy" mode to make up for the void in my numbers. It took a ton of effort, long nights, and creative pitches, but I was able to make up for the gap. I learned just how important it is to diversify my portfolio so that I don't find myself, or the company, in this position again."

"When working on curriculum development, I learned an important lesson. Two of our teachers wanted to keep a lesson in, because of personal connections to the lesson, but the other three were quite against it, with me being the uncertain one. I saw the validity in both sides. So, rather than find ourselves with a divisive issue on our hands, I proposed that we have a "freebie" lesson when we each got to pick one that we thought would culturally enrich our students. I learned that by thinking outside of the box, the team and our students would all benefit."

22. When change occurs in the workplace, it can create new problems. Do you see these as inconvenient problems, or opportunities to learn?

When a change occurs in the workplace, often problems arise due to new implementations and procedures, or unforeseen kinks needing to be worked out. Do you approach these problems positively or do you resist the change? Talk to the interviewer about how you can adapt to the inevitable issues that come with the change in the workplace.

"I fully understand that when the change occurs in the workplace, some new problems may arise because of it. I embrace workplace change because it often gives me the opportunity to learn a new skill or even teach a colleague a new skill."

"As an executive assistant, I see change all the time. Policy changes, travel changes, issues in scheduling, and the like. Although they are often inconvenient or threaten to throw my day off, I am always prepared with a Plan B. Each time these situations occur, I learn something new."

"Change is inevitable when you work with people because you cannot control everyone's reactions in a day, or whether they even show up to work. Recently I had a major shift in my team and, overnight, went from being completely confident in my team to the need of reassessing our strategy. I saw this as an opportunity to stretch outside of my comfort zone. I embrace change and learning opportunities."

"One change that we always go through in this industry are shifts related to social media platforms and online trends. These tools are ever evolving, and when you think you have it - poof - changes are made. I don't mind this, however. I believe that each shift is a chance to learn something new."

"I like to approach every day and situation as an opportunity to learn and grow, so even though it's uncomfortable, I like to think that there's something valuable to take away from any situation that involves change."

"I'm all about taking everything in stride and jumping on opportunities for growth and improvement. My latest job has been a year-long exercise in that: a start-up that pivoted entirely from the direction it had been going in when I was brought on, with an entirely new team and even intended client base. I decided to take it as a growth opportunity. I took a deep breath, rolled up my sleeves, and got to work learning and adapting to the new product, clients, and management. I think that the experience will serve me well in the future since I became quite flexible and learned a lot about myself and sales in the process."

"I am adaptable to change. As a teacher, I have to be open to change! Nothing stays the same in education and students challenge everything. I am capable of pivoting when needed and am not thrown off my game, easily."

23. Rate your problem solving skills from 1-10. How do you justify your rating?

The interviewer wants to know how you would rate your problem-solving skills. Of course, you want to give yourself a healthy rating; however, it's crucial that you remain realistic. Try to avoid giving yourself a 10, and nobody is perfect, and you do not want to come across as overly confident or someone who has no room for feedback and improvement. Alternately, avoid giving yourself too little credit. You do not want to paint the picture that you are a problem-solving dud! Try to remain in the 7.5-9.5 range while staying honest and accurate. Everyone has room to learn and improve! Be sure to justify your score as well.

"I rate my problem-solving skills as an 8/10. I will, on occasion, have times when I am not as efficient as I would like to be but all in all, I do feel that my problem-solving skills are above average. My supervisor and co-workers will attest to my fast reflexes when a problem arises, and they would also say that I remain calm under pressure."

"I will rate myself an eight because I value problem-solving but, just like most people, I have things to learn. Some ways to ensure that I can effectively solve issues are by utilizing multiple knowledge resources when looking for answers."

"I will rate myself an 8.5 because I consider myself a strong problem solver, especially when it comes to important matters that affect my team. Solid problem-solving skills are the foundation of success in business. I am always striving to be a better problem solver, so I leave the rest of the scale as an aspirational measure."

"Problem-solving is at the heart of what we do in marketing. We have to solve branding and sales issues for our clients all the time. I am an exceptional problem-solver, and quite creative with my strategies. For that reason, I will rate myself as a 9/10 and always improving."

"I'd rate my problem-solving skills as an 8/10. I believe I'm always a willing learner who brings creativity to the table, no matter what the situation. I am still full of ideas on how to solve a problem, and yet I am also open to the opinion and input of others. I like to collaborate but am not afraid to take charge and make it happen. There's room for growth, which is why I give myself only an 8!"

"I would say I get a solid 8.3 on a scale of 10. Seems weird to give myself something like a .3, but I think of it as an 83%, which is a B minus teetering on a solid B. It's a solid grade, with definite room for improvement, since I'm certainly not perfect. The reason for the B-/B grade would be that I'm quick to take action and figure out the solution as I go, but sometimes I could benefit from taking a moment to pause and reflect or gather other contributors before taking action. That said, I believe I get the best outcome possible when faced with a challenge."

"I would say I'm a strong problem solver and would rate myself an 8/10. I follow my gut and problem solve creatively, but know there is still room for improvement. I think my teamwork and problem strategies highlight my strengths in problem-solving. I can hear what people find essential and flush out the things on which we can compromise. Then, I come up with a great outcome that makes the teachers happy and is in the best interest of our students."

24. What do you think might be the greatest challenges faced in this job? How will you overcome these challenges?

Even though it may seem like a dream job, the interviewer wants to know that you have realistic expectations of the role and that you will not be blindsided if problems or challenges present themselves. Keep your answer simple. It is okay to ask for clarification on the position if you do not fully understand what challenges are in store for you.

"I think the greatest challenges in this role will be to learn the proper operation of the equipment. Another challenge will be the physical aspect of the position as I will be required to stand and walk around most of the day. I will be sure to pay keen attention to training and ask questions along the way. In regards to the physical component - I will get used to the additional activity after just a couple of days, I'm sure."

"I believe that the greatest challenge in this job will be to learn the ins and outs of your systems. I am familiar with SAP; however, will need to navigate some modules that will be new to me. If you don't mind, I would like to gain a head start on these by studying online for the next weekend or so."

"As a new manager, the biggest challenge is always to earn the trust of my new team. I plan to do this by getting to know everyone through genuine interest and conversation. I do understand that solid trust develops over time, but it's important to me to get started on the right foot."

"The greatest challenge is going to be getting to know your clients and their preferences. Every client has their quirks that need to be kept the top of mind during projects. I plan to read as many project notes as possible before diving into face-to-face meetings. I intend to come across to your clients as well-prepared and earnest."

"I think the added responsibility of running one of the highest volume departments in the store will be an adjustment, but it's a welcome challenge. I am looking forward to tackling it head on and growing through the challenges, because I know on the other side of those challenges, of that responsibility, lies the biggest opportunity yet."

"I would say the greatest challenge I'll face in this role is learning the industry ins and outs to be perceived as an expert when making the pitch to new clients. I want to be sure to immerse myself in the industry jargon, attend as many seminars and conventions as possible, and I've already begun subscribing and reading the leading industry publications so that I can get into the nitty-gritty of how it all works. Of course, I will also seek out mentorship opportunities where I can learn from folks who have been in the industry for years. I find they love to share their knowledge and it gives me a leg up."

"I believe the greatest challenge faced in this new position would be getting accustomed to the new curriculum. I am accustomed to my lesson plans and the curriculum I've had a hand in developing over the last ten years, so something new will have a bit of a learning curve, but welcomed. I am looking forward to a new challenge and to tackle a new set of lessons!"

25. What steps do you take to solve a problem?

The interviewer would like to know that you understand the importance of taking calculated steps when problem-solving in the workplace. Most candidates want to sound like go-getters, and their first instinct would be to say that they jump right in. Jumping right in can cause costly mistakes and oversights. Assure the interviewer that you will workshop the issue before diving in! Here are some steps to take: 1. Identify The Problem. Proper problem solving involves ensuring that you are very clear on the nature of the problem. Be sure that you fully understand the core of the problem before trying to repair it. 2. Identify The Stakeholders. Ask yourself, what the best case resolution will be for all stakeholders, not just for yourself. Ask yourself what is best for the company, your coworkers, and your clients. 3. List Your Options. The third step is to figure out what your options are when it comes to your course of action. Write them down if you need to. 4. Evaluate Your Options. Take a look at your list of potential actions and see if you can solve the problem using just one, or a blend of them. 5. Execute! Finally, execute your well-researched action plan. Be sure to set up a follow-up time to ensure that your solution worked.

"When I need to solve a problem, I first stop to ensure that I understand the issue at hand. Once I do, I will think of potential fixes and the pros and cons of each. Whichever solution or a blend of solutions is best for the customer; I will choose that option."

"My current company is very team-focused, and we train everyone to problem-solve with "what is best for team morale" being the question at hand. I have been with the company for twelve years so most problems I have a pretty clear idea of what will work for us, but when I need to workshop an idea, I will call in my team and have a brainstorm session."

"Problem-solving in Marketing can be unique because you have to truly balance the customers' pain point with the solutions that are currently available. Also, some clients like trying new marketing methods and others want to remain conservative, using only tried and true advertising methods, for instance. When I approach a problem, I first identify the personality of the client and their business and research options from there."

"Problem-solving in a retail environment is challenging in the sense that the issue is often something that needs to be fixed immediately, like a faulty product or an upset customer. When faced with a problem, I ask questions first, to ensure that I fully understand the core of the issue. Once I fully understand the core of the problem, I can more easily troubleshoot from there."

"Every customer is different, with unique needs, so when I need to problem-solve, I am often coming across a brand new problem or a different version of a problem I have seen before. Our company is big on chasing the money, and so I have been trained that every solution I choose must have the business' bottom line top of mind. My process is to understand the issue, address who the stakeholders are, and create a solution where everyone feels they won in some small way."

"Problem-solving in the classroom is a challenge because it is often on the fly. Or, a student will ask a question in a new way and I won't necessarily have the answer! When a problem arises, I like to involve my class, have a brainstorm session, and discuss as a group what we could do. This method turns an issue into a conversation where we have the opportunity to come up with some unique solutions."

Career Sidekick

26 Expert-Backed Problem Solving Examples – Interview Answers

Published: February 13, 2023

Interview Questions and Answers

Actionable advice from real experts:

picture of Biron Clark

Biron Clark

Former Recruiter

problem solving type of question

Contributor

Dr. Kyle Elliott

Career Coach

problem solving type of question

Hayley Jukes

Editor-in-Chief

Biron Clark

Biron Clark , Former Recruiter

Kyle Elliott , Career Coach

Image of Hayley Jukes

Hayley Jukes , Editor

As a recruiter , I know employers like to hire people who can solve problems and work well under pressure.

 A job rarely goes 100% according to plan, so hiring managers are more likely to hire you if you seem like you can handle unexpected challenges while staying calm and logical.

But how do they measure this?

Hiring managers will ask you interview questions about your problem-solving skills, and they might also look for examples of problem-solving on your resume and cover letter. 

In this article, I’m going to share a list of problem-solving examples and sample interview answers to questions like, “Give an example of a time you used logic to solve a problem?” and “Describe a time when you had to solve a problem without managerial input. How did you handle it, and what was the result?”

  • Problem-solving involves identifying, prioritizing, analyzing, and solving problems using a variety of skills like critical thinking, creativity, decision making, and communication.
  • Describe the Situation, Task, Action, and Result ( STAR method ) when discussing your problem-solving experiences.
  • Tailor your interview answer with the specific skills and qualifications outlined in the job description.
  • Provide numerical data or metrics to demonstrate the tangible impact of your problem-solving efforts.

What are Problem Solving Skills? 

Problem-solving is the ability to identify a problem, prioritize based on gravity and urgency, analyze the root cause, gather relevant information, develop and evaluate viable solutions, decide on the most effective and logical solution, and plan and execute implementation. 

Problem-solving encompasses other skills that can be showcased in an interview response and your resume. Problem-solving skills examples include:

  • Critical thinking
  • Analytical skills
  • Decision making
  • Research skills
  • Technical skills
  • Communication skills
  • Adaptability and flexibility

Why is Problem Solving Important in the Workplace?

Problem-solving is essential in the workplace because it directly impacts productivity and efficiency. Whenever you encounter a problem, tackling it head-on prevents minor issues from escalating into bigger ones that could disrupt the entire workflow. 

Beyond maintaining smooth operations, your ability to solve problems fosters innovation. It encourages you to think creatively, finding better ways to achieve goals, which keeps the business competitive and pushes the boundaries of what you can achieve. 

Effective problem-solving also contributes to a healthier work environment; it reduces stress by providing clear strategies for overcoming obstacles and builds confidence within teams. 

Examples of Problem-Solving in the Workplace

  • Correcting a mistake at work, whether it was made by you or someone else
  • Overcoming a delay at work through problem solving and communication
  • Resolving an issue with a difficult or upset customer
  • Overcoming issues related to a limited budget, and still delivering good work through the use of creative problem solving
  • Overcoming a scheduling/staffing shortage in the department to still deliver excellent work
  • Troubleshooting and resolving technical issues
  • Handling and resolving a conflict with a coworker
  • Solving any problems related to money, customer billing, accounting and bookkeeping, etc.
  • Taking initiative when another team member overlooked or missed something important
  • Taking initiative to meet with your superior to discuss a problem before it became potentially worse
  • Solving a safety issue at work or reporting the issue to those who could solve it
  • Using problem solving abilities to reduce/eliminate a company expense
  • Finding a way to make the company more profitable through new service or product offerings, new pricing ideas, promotion and sale ideas, etc.
  • Changing how a process, team, or task is organized to make it more efficient
  • Using creative thinking to come up with a solution that the company hasn’t used before
  • Performing research to collect data and information to find a new solution to a problem
  • Boosting a company or team’s performance by improving some aspect of communication among employees
  • Finding a new piece of data that can guide a company’s decisions or strategy better in a certain area

Problem-Solving Examples for Recent Grads/Entry-Level Job Seekers

  • Coordinating work between team members in a class project
  • Reassigning a missing team member’s work to other group members in a class project
  • Adjusting your workflow on a project to accommodate a tight deadline
  • Speaking to your professor to get help when you were struggling or unsure about a project
  • Asking classmates, peers, or professors for help in an area of struggle
  • Talking to your academic advisor to brainstorm solutions to a problem you were facing
  • Researching solutions to an academic problem online, via Google or other methods
  • Using problem solving and creative thinking to obtain an internship or other work opportunity during school after struggling at first

How To Answer “Tell Us About a Problem You Solved”

When you answer interview questions about problem-solving scenarios, or if you decide to demonstrate your problem-solving skills in a cover letter (which is a good idea any time the job description mentions problem-solving as a necessary skill), I recommend using the STAR method.

STAR stands for:

It’s a simple way of walking the listener or reader through the story in a way that will make sense to them. 

Start by briefly describing the general situation and the task at hand. After this, describe the course of action you chose and why. Ideally, show that you evaluated all the information you could given the time you had, and made a decision based on logic and fact. Finally, describe the positive result you achieved.

Note: Our sample answers below are structured following the STAR formula. Be sure to check them out!

EXPERT ADVICE

problem solving type of question

Dr. Kyle Elliott , MPA, CHES Tech & Interview Career Coach caffeinatedkyle.com

How can I communicate complex problem-solving experiences clearly and succinctly?

Before answering any interview question, it’s important to understand why the interviewer is asking the question in the first place.

When it comes to questions about your complex problem-solving experiences, for example, the interviewer likely wants to know about your leadership acumen, collaboration abilities, and communication skills, not the problem itself.

Therefore, your answer should be focused on highlighting how you excelled in each of these areas, not diving into the weeds of the problem itself, which is a common mistake less-experienced interviewees often make.

Tailoring Your Answer Based on the Skills Mentioned in the Job Description

As a recruiter, one of the top tips I can give you when responding to the prompt “Tell us about a problem you solved,” is to tailor your answer to the specific skills and qualifications outlined in the job description. 

Once you’ve pinpointed the skills and key competencies the employer is seeking, craft your response to highlight experiences where you successfully utilized or developed those particular abilities. 

For instance, if the job requires strong leadership skills, focus on a problem-solving scenario where you took charge and effectively guided a team toward resolution. 

By aligning your answer with the desired skills outlined in the job description, you demonstrate your suitability for the role and show the employer that you understand their needs.

Amanda Augustine expands on this by saying:

“Showcase the specific skills you used to solve the problem. Did it require critical thinking, analytical abilities, or strong collaboration? Highlight the relevant skills the employer is seeking.”  

Interview Answers to “Tell Me About a Time You Solved a Problem”

Now, let’s look at some sample interview answers to, “Give me an example of a time you used logic to solve a problem,” or “Tell me about a time you solved a problem,” since you’re likely to hear different versions of this interview question in all sorts of industries.

The example interview responses are structured using the STAR method and are categorized into the top 5 key problem-solving skills recruiters look for in a candidate.

1. Analytical Thinking

problem solving type of question

Situation: In my previous role as a data analyst , our team encountered a significant drop in website traffic.

Task: I was tasked with identifying the root cause of the decrease.

Action: I conducted a thorough analysis of website metrics, including traffic sources, user demographics, and page performance. Through my analysis, I discovered a technical issue with our website’s loading speed, causing users to bounce. 

Result: By optimizing server response time, compressing images, and minimizing redirects, we saw a 20% increase in traffic within two weeks.

2. Critical Thinking

problem solving type of question

Situation: During a project deadline crunch, our team encountered a major technical issue that threatened to derail our progress.

Task: My task was to assess the situation and devise a solution quickly.

Action: I immediately convened a meeting with the team to brainstorm potential solutions. Instead of panicking, I encouraged everyone to think outside the box and consider unconventional approaches. We analyzed the problem from different angles and weighed the pros and cons of each solution.

Result: By devising a workaround solution, we were able to meet the project deadline, avoiding potential delays that could have cost the company $100,000 in penalties for missing contractual obligations.

3. Decision Making

problem solving type of question

Situation: As a project manager , I was faced with a dilemma when two key team members had conflicting opinions on the project direction.

Task: My task was to make a decisive choice that would align with the project goals and maintain team cohesion.

Action: I scheduled a meeting with both team members to understand their perspectives in detail. I listened actively, asked probing questions, and encouraged open dialogue. After carefully weighing the pros and cons of each approach, I made a decision that incorporated elements from both viewpoints.

Result: The decision I made not only resolved the immediate conflict but also led to a stronger sense of collaboration within the team. By valuing input from all team members and making a well-informed decision, we were able to achieve our project objectives efficiently.

4. Communication (Teamwork)

problem solving type of question

Situation: During a cross-functional project, miscommunication between departments was causing delays and misunderstandings.

Task: My task was to improve communication channels and foster better teamwork among team members.

Action: I initiated regular cross-departmental meetings to ensure that everyone was on the same page regarding project goals and timelines. I also implemented a centralized communication platform where team members could share updates, ask questions, and collaborate more effectively.

Result: Streamlining workflows and improving communication channels led to a 30% reduction in project completion time, saving the company $25,000 in operational costs.

5. Persistence 

Situation: During a challenging sales quarter, I encountered numerous rejections and setbacks while trying to close a major client deal.

Task: My task was to persistently pursue the client and overcome obstacles to secure the deal.

Action: I maintained regular communication with the client, addressing their concerns and demonstrating the value proposition of our product. Despite facing multiple rejections, I remained persistent and resilient, adjusting my approach based on feedback and market dynamics.

Result: After months of perseverance, I successfully closed the deal with the client. By closing the major client deal, I exceeded quarterly sales targets by 25%, resulting in a revenue increase of $250,000 for the company.

Tips to Improve Your Problem-Solving Skills

Throughout your career, being able to showcase and effectively communicate your problem-solving skills gives you more leverage in achieving better jobs and earning more money .

So to improve your problem-solving skills, I recommend always analyzing a problem and situation before acting.

 When discussing problem-solving with employers, you never want to sound like you rush or make impulsive decisions. They want to see fact-based or data-based decisions when you solve problems.

Don’t just say you’re good at solving problems. Show it with specifics. How much did you boost efficiency? Did you save the company money? Adding numbers can really make your achievements stand out.

To get better at solving problems, analyze the outcomes of past solutions you came up with. You can recognize what works and what doesn’t.

Think about how you can improve researching and analyzing a situation, how you can get better at communicating, and deciding on the right people in the organization to talk to and “pull in” to help you if needed, etc.

Finally, practice staying calm even in stressful situations. Take a few minutes to walk outside if needed. Step away from your phone and computer to clear your head. A work problem is rarely so urgent that you cannot take five minutes to think (with the possible exception of safety problems), and you’ll get better outcomes if you solve problems by acting logically instead of rushing to react in a panic.

You can use all of the ideas above to describe your problem-solving skills when asked interview questions about the topic. If you say that you do the things above, employers will be impressed when they assess your problem-solving ability.

More Interview Resources

  • 3 Answers to “How Do You Handle Stress?”
  • How to Answer “How Do You Handle Conflict?” (Interview Question)
  • Sample Answers to “Tell Me About a Time You Failed”

picture of Biron Clark

About the Author

Biron Clark is a former executive recruiter who has worked individually with hundreds of job seekers, reviewed thousands of resumes and LinkedIn profiles, and recruited for top venture-backed startups and Fortune 500 companies. He has been advising job seekers since 2012 to think differently in their job search and land high-paying, competitive positions. Follow on Twitter and LinkedIn .

Read more articles by Biron Clark

About the Contributor

Kyle Elliott , career coach and mental health advocate, transforms his side hustle into a notable practice, aiding Silicon Valley professionals in maximizing potential. Follow Kyle on LinkedIn .

Image of Hayley Jukes

About the Editor

Hayley Jukes is the Editor-in-Chief at CareerSidekick with five years of experience creating engaging articles, books, and transcripts for diverse platforms and audiences.

Continue Reading

12 Expert-Approved Responses to ‘What Makes You Unique?’ in Job Interviews

15 most common pharmacist interview questions and answers, 15 most common paralegal interview questions and answers, top 30+ funny interview questions and answers, 60 hardest interview questions and answers, 100+ best ice breaker questions to ask candidates, top 20 situational interview questions (& sample answers), 15 most common physical therapist interview questions and answers.

LOGO

Your favourite senior outside college

Home » Job Tips » Interview Guide » Problem Solving Interview Questions

Top 17 Problem-Solving Interview Questions for Freshers & Experienced Professionals

Problem Solving Interview Questions

Problem-solving skills are essential for success in almost any job position. Employers are in search of individuals who possess the ability to think critically, tackle obstacles and situations systematically, and develop efficient resolutions. In this guide, you will get different problem-solving interview questions and answers and valuable tips to equip you to prepare for your upcoming interview.

Table of Contents

What are Problem-Solving Interview Questions?

Problem-solving interview questions are questions that focus on a candidate’s aptitude for collecting information, evaluating an issue, considering its advantages and disadvantages, and arriving at a sound conclusion. Employers use these questions to understand and analyze one’s critical thinking abilities and ability to make informed decisions.

These questions are designed to assess a candidate’s critical thinking and decision-making skills. You can develop the right attitude and approach to solving a problem by checking out this complete guide on what are problem-solving skills .

Find and Apply Banner

Problem-Solving Interview Questions and Answers for Freshers

Below are the common problem-solving interview questions you might likely come across as a fresher. You can also check out this interview preparation course to equip yourself with common interview etiquette .

Q1. When faced with a problem, what is your action plan?

Answer: When I encounter a problem, my first step is to explore how others have successfully addressed similar challenges. This research provides me with diverse solutions, enabling me to choose the most fitting approach for both myself and the organization. Subsequently, I collaborate with my managers and colleagues, ensuring clear communication as we implement the selected solution.

Q2. What factors do you implement to weigh the pros and cons of a decision?

Answer: When evaluating decisions, I consider various factors, such as the potential impact the decision has on both short-term and long-term goals, assess the risks involved, seek input from relevant team members, and how relevant the decision aligns with the organization’s goals and values. With this comprehensive approach, anyone can make a well-informed decision.

Q3. How do you know when to seek assistance or tackle an issue on your own?

Answer: I evaluate the complexity and urgency of the issue. If it’s something I can handle within a stipulated time frame, I will address it on my own. However, for more complex or time-sensitive issues, I promptly seek help from colleagues or supervisors to ensure a quick and effective resolution.

Q4. Describe a situation when you identified an issue early on and resolved it before it got out of hand?

Answer: In my previous position as an intern, I discovered inaccuracies in the data during a project. Without hesitation, I took it upon myself to thoroughly examine and resolve these discrepancies to ensure precise outcomes. This valuable experience taught me the importance of attention to detail and enhanced my commitment to quality work.

Q5. Describe a situation when you had a task but lacked the abilities needed to finish it?

Answer: In my entry-level role, I successfully managed a sudden increase in customer inquiries caused by a website glitch. I worked with the team to address the issue, prioritized urgent cases, communicated transparently with customers, and provided temporary solutions until a permanent fix was implemented. This situation demonstrated my proficiency in managing high-pressure situations and delivering exceptional service to customers.

Q6. Describe a situation where you handled a crisis well?

Answer: During my internship as a client relations specialist, we encountered an unforeseen rise in customer discontentment due to concerns about the quality of our products. Working closely with relevant departments, I identified the root cause and devised a plan of action. By prioritizing urgent cases and maintaining open communication with affected customers while providing prompt updates, we effectively restored their satisfaction and prevented any further damage to our brand reputation.

Q7. Give an example of a challenging circumstance you experienced at work that called for quick thought and decisive action?

Answer: In a previous internship role, I faced a challenging project with strict deadlines and limited resources. To overcome this obstacle, I employed strategic resource allocation techniques, prioritized tasks effectively, and worked closely with my team members. Through careful planning and delegating responsibilities efficiently, not only did we meet clients’ expectations, but we surpassed the client’s expectations by delivering the completed project within the given deadline.

Q8. How would you respond to a disappointed and angry client?

Answer: When faced with an unhappy client, my focus is to remain calm and positively interact with them to prevent the situation from getting worse. I start by engaging in dialogue to comprehend the reasons for their discontentment, gathering all the essential information needed for effective problem-solving.  Once I have a clear understanding of the issue at hand, I reassure the customer that we are dedicated to resolving it quickly. By providing frequent updates throughout the resolution process, we strive to keep our clients informed and build trust in our efforts toward finding a satisfactory solution.

Q9. What metrics do you usually use to monitor your strategies? 

Answer: I use key performance indicators (KPIs) particular to the objectives of the project or work at hand to keep monitoring my methods. Conversion rates, customer satisfaction ratings, and project schedules are a few examples of these KPIs. By monitoring these metrics regularly, I can evaluate the performance of my strategies and alter them based on data as necessary.

Q10. How would you assess the impact of potential issues?

Answer: To evaluate the effects of potential problems, I utilize a methodical strategy. Initially, I determine the type and extent of the issue by assessing its potential to cause disruptions or hinder project objectives. Then, I consider how it may impact related tasks and timelines. After prioritizing these issues based on severity and their overall impact on goals, I create contingency plans proactively and allocate resources efficiently. This approach allows me to effectively manage challenges before they escalate, ultimately minimizing any negative consequences for the success of the project.

Also Read: Common Interview Questions for Freshers .

Problem-Solving Questions with Answers for Experienced Candidates

Here are some problem-solving questions and answers for experienced individuals.

Q11. How would you approach a new idea that has enormous profit potential but could have legal ramifications for the business?

Answer: When faced with a project that involves both financial opportunities and possible legal consequences, I would prioritize caution and thorough evaluation. I would conduct in-depth research and seek guidance from legal specialists to fully understand the implications and compliance requirements involved.  Then, I would collaborate with lawyers, cross-functional teams, and stakeholders to develop a comprehensive plan that minimizes potential legal risks while maximizing revenue possibilities.

Q12. Give an example of a work or project that looked too big at first. What methods did you employ to guarantee its effective completion and how did you approach it?

Answer: In a former position, I was tasked with an assignment that demanded thorough data analysis and timely reporting. Despite feeling overwhelmed at first, I tackled the project by dividing it into manageable tasks. Moreover, I devised a comprehensive schedule to ensure the project stayed on course.  By prioritizing crucial components and collaborating with team members who possessed specialized skills in certain aspects, we successfully accomplished the task together in an efficient manner. With effective time management skills and dedicated effort from our team’s collaboration, we met the deadline and had exceptional results.

Q13. Have you ever solved a problem without managerial input? What was the outcome and how did you handle it?

Answer: In a previous position, I encountered a technical challenge that disrupted our operations. As the leader of the team, I took charge by collecting information and analyzing the problem. Together with my team, we conducted a brainstorming session to come up with potential solutions and collaborated with the IT department to resolve it. Our proactive approach helped minimize any further disruptions and enabled us to restore normalcy within 24 hours.

Q14. How do you respond when your supervisor asks for your opinion or recommendation?

Answer: When my supervisor asks for my input, I make sure to offer a thoughtful response. To start, I evaluate the circumstances and take into account any pertinent details while also considering possible consequences. Then, I communicate my perspective directly and succinctly and back it up with evidence or illustrations. Furthermore, I remain receptive to constructive feedback, promoting an environment of cooperation where ideas can be shared for the best course of action.

Q15. How do you assess a solution’s effectiveness?

Answer: Evaluating the effectiveness of a solution involves a systematic assessment procedure. To begin, specific metrics and key performance indicators are established in accordance with the nature and goals of the problem at hand. These indicators are continuously monitored, comparing data before and after implementation to detect any positive changes or discrepancies. Gathering feedback from individuals involved, such as team members and end-users, offers valuable perspectives on how well the solution is performing in practical settings. Consistent reviews and necessary adjustments guarantee and support long-term objectives effectively.

Q16. Describe how you learn from your experiences. 

Answer: Once a project or task is completed, I take time to conduct a thorough evaluation. By looking back at both achievements and difficulties encountered, I identify key factors that contributed to success or hindered progress. This introspection allows me to identify areas for development and fine-tune my approach for future ventures. Furthermore, getting feedback from my colleagues and supervisors offers unique viewpoints that contribute to a more holistic understanding of the experience.

Q17. Do you consider yourself a great problem solver?

Answer: I possess strong problem-solving abilities. My approach to challenges is proactive, breaking down complex problems into manageable parts. By examining the underlying causes and utilizing both creativity and critical thinking, I have a history of developing successful solutions. Moreover, I am open to collaborating with others and appreciate diverse viewpoints that contribute to comprehensive problem-solving approaches. While there is always room for growth, my past achievements showcase my determination to confront obstacles head-on and devise innovative resolutions.

Also Read: Behavioral Interview Questions .

Problem-Solving Interview Questions: Common Mistakes to Avoid

Below are relevant tips to aid you in answering problem-solving interview questions.

  • Avoid Giving Easy Responses- Individuals who opt for easier responses are considered to lack critical thinking.
  • Avoid Giving Hasty Responses- Take your time in addressing the issue at hand and make sure you have a thorough understanding of it. If there are any unclear points, ask for clarification before giving your response. This shows that you value accuracy and precision.
  • Avoid Taking Too Much on One Question- It is important to be brief and thorough when responding within a reasonable timeframe.

When answering an analytical question during an interview, endeavor to demonstrate the right mindset for solving problems. Problem-solving interview questions are an opportunity for you to showcase your analytical skills, creativity, and ability to handle challenges. You can make the right impression by preparing thoroughly, practicing different types of questions, and emphasizing your problem-solving ability.

Drop us a comment below if this blog has been helpful to you. Also, check out how to ace interviews with proven tips .

  • ← Previous
  • Next →

problem solving type of question

Shobha Saini, the Head of Human Resources at Internshala, has maintained a stellar track record in employee relations and talent acquisition. With eight exceptional years of experience, she specializes in strategic planning, policy-making, and performance management. A multi-talented individual, she has played a major role in strategizing HR practices in the organization.

Related Post

problem solving type of question

Top 45 Oracle Java Developer Job Interview Questions & Answers

Oracle is a multinational computer technology corporation that offers a wide range of software products and services. This includes cloud engineering and applications, enterprise resource planning (ERP) software, human capital

problem solving type of question

Top 45 Zoho Interview Questions [with Sample Answers & Tips]

Zoho Corporation is a technology company that builds software solutions for businesses across industries like sales, marketing, project management, finance, and IT management. The company was set up in 1996

problem solving type of question

Oracle Database Administrator Interview Questions: A Detailed Guide

Oracle database software is a powerful tool that enables organizations to efficiently store, manage, and retrieve large amounts of data. The software is widely used in various industries, such as

problem solving type of question

Zoho Web Developer Interview Questions for Freshers & Experienced Candidates

Zoho is an innovative tech company founded in 1996 that provides cloud-based solutions and mobile application development services to help businesses stay productive. A web developer at Zoho is responsible

  • Bipolar Disorder
  • Therapy Center
  • When To See a Therapist
  • Types of Therapy
  • Best Online Therapy
  • Best Couples Therapy
  • Best Family Therapy
  • Managing Stress
  • Sleep and Dreaming
  • Understanding Emotions
  • Self-Improvement
  • Healthy Relationships
  • Student Resources
  • Personality Types
  • Guided Meditations
  • Verywell Mind Insights
  • 2024 Verywell Mind 25
  • Mental Health in the Classroom
  • Editorial Process
  • Meet Our Review Board
  • Crisis Support

Overview of the Problem-Solving Mental Process

Kendra Cherry, MS, is a psychosocial rehabilitation specialist, psychology educator, and author of the "Everything Psychology Book."

problem solving type of question

Rachel Goldman, PhD FTOS, is a licensed psychologist, clinical assistant professor, speaker, wellness expert specializing in eating behaviors, stress management, and health behavior change.

problem solving type of question

  • Identify the Problem
  • Define the Problem
  • Form a Strategy
  • Organize Information
  • Allocate Resources
  • Monitor Progress
  • Evaluate the Results

Frequently Asked Questions

Problem-solving is a mental process that involves discovering, analyzing, and solving problems. The ultimate goal of problem-solving is to overcome obstacles and find a solution that best resolves the issue.

The best strategy for solving a problem depends largely on the unique situation. In some cases, people are better off learning everything they can about the issue and then using factual knowledge to come up with a solution. In other instances, creativity and insight are the best options.

It is not necessary to follow problem-solving steps sequentially, It is common to skip steps or even go back through steps multiple times until the desired solution is reached.

In order to correctly solve a problem, it is often important to follow a series of steps. Researchers sometimes refer to this as the problem-solving cycle. While this cycle is portrayed sequentially, people rarely follow a rigid series of steps to find a solution.

The following steps include developing strategies and organizing knowledge.

1. Identifying the Problem

While it may seem like an obvious step, identifying the problem is not always as simple as it sounds. In some cases, people might mistakenly identify the wrong source of a problem, which will make attempts to solve it inefficient or even useless.

Some strategies that you might use to figure out the source of a problem include :

  • Asking questions about the problem
  • Breaking the problem down into smaller pieces
  • Looking at the problem from different perspectives
  • Conducting research to figure out what relationships exist between different variables

2. Defining the Problem

After the problem has been identified, it is important to fully define the problem so that it can be solved. You can define a problem by operationally defining each aspect of the problem and setting goals for what aspects of the problem you will address

At this point, you should focus on figuring out which aspects of the problems are facts and which are opinions. State the problem clearly and identify the scope of the solution.

3. Forming a Strategy

After the problem has been identified, it is time to start brainstorming potential solutions. This step usually involves generating as many ideas as possible without judging their quality. Once several possibilities have been generated, they can be evaluated and narrowed down.

The next step is to develop a strategy to solve the problem. The approach used will vary depending upon the situation and the individual's unique preferences. Common problem-solving strategies include heuristics and algorithms.

  • Heuristics are mental shortcuts that are often based on solutions that have worked in the past. They can work well if the problem is similar to something you have encountered before and are often the best choice if you need a fast solution.
  • Algorithms are step-by-step strategies that are guaranteed to produce a correct result. While this approach is great for accuracy, it can also consume time and resources.

Heuristics are often best used when time is of the essence, while algorithms are a better choice when a decision needs to be as accurate as possible.

4. Organizing Information

Before coming up with a solution, you need to first organize the available information. What do you know about the problem? What do you not know? The more information that is available the better prepared you will be to come up with an accurate solution.

When approaching a problem, it is important to make sure that you have all the data you need. Making a decision without adequate information can lead to biased or inaccurate results.

5. Allocating Resources

Of course, we don't always have unlimited money, time, and other resources to solve a problem. Before you begin to solve a problem, you need to determine how high priority it is.

If it is an important problem, it is probably worth allocating more resources to solving it. If, however, it is a fairly unimportant problem, then you do not want to spend too much of your available resources on coming up with a solution.

At this stage, it is important to consider all of the factors that might affect the problem at hand. This includes looking at the available resources, deadlines that need to be met, and any possible risks involved in each solution. After careful evaluation, a decision can be made about which solution to pursue.

6. Monitoring Progress

After selecting a problem-solving strategy, it is time to put the plan into action and see if it works. This step might involve trying out different solutions to see which one is the most effective.

It is also important to monitor the situation after implementing a solution to ensure that the problem has been solved and that no new problems have arisen as a result of the proposed solution.

Effective problem-solvers tend to monitor their progress as they work towards a solution. If they are not making good progress toward reaching their goal, they will reevaluate their approach or look for new strategies .

7. Evaluating the Results

After a solution has been reached, it is important to evaluate the results to determine if it is the best possible solution to the problem. This evaluation might be immediate, such as checking the results of a math problem to ensure the answer is correct, or it can be delayed, such as evaluating the success of a therapy program after several months of treatment.

Once a problem has been solved, it is important to take some time to reflect on the process that was used and evaluate the results. This will help you to improve your problem-solving skills and become more efficient at solving future problems.

A Word From Verywell​

It is important to remember that there are many different problem-solving processes with different steps, and this is just one example. Problem-solving in real-world situations requires a great deal of resourcefulness, flexibility, resilience, and continuous interaction with the environment.

Get Advice From The Verywell Mind Podcast

Hosted by therapist Amy Morin, LCSW, this episode of The Verywell Mind Podcast shares how you can stop dwelling in a negative mindset.

Follow Now : Apple Podcasts / Spotify / Google Podcasts

You can become a better problem solving by:

  • Practicing brainstorming and coming up with multiple potential solutions to problems
  • Being open-minded and considering all possible options before making a decision
  • Breaking down problems into smaller, more manageable pieces
  • Asking for help when needed
  • Researching different problem-solving techniques and trying out new ones
  • Learning from mistakes and using them as opportunities to grow

It's important to communicate openly and honestly with your partner about what's going on. Try to see things from their perspective as well as your own. Work together to find a resolution that works for both of you. Be willing to compromise and accept that there may not be a perfect solution.

Take breaks if things are getting too heated, and come back to the problem when you feel calm and collected. Don't try to fix every problem on your own—consider asking a therapist or counselor for help and insight.

If you've tried everything and there doesn't seem to be a way to fix the problem, you may have to learn to accept it. This can be difficult, but try to focus on the positive aspects of your life and remember that every situation is temporary. Don't dwell on what's going wrong—instead, think about what's going right. Find support by talking to friends or family. Seek professional help if you're having trouble coping.

Davidson JE, Sternberg RJ, editors.  The Psychology of Problem Solving .  Cambridge University Press; 2003. doi:10.1017/CBO9780511615771

Sarathy V. Real world problem-solving .  Front Hum Neurosci . 2018;12:261. Published 2018 Jun 26. doi:10.3389/fnhum.2018.00261

By Kendra Cherry, MSEd Kendra Cherry, MS, is a psychosocial rehabilitation specialist, psychology educator, and author of the "Everything Psychology Book."

Intentional practices. Amplified leadership. Done simply. Join the Leading to Learn Accelerator starting March 7th. Enroll Now!

🎁 Give the Gift of Learning for December – Check out the NEW Leading to Learn Accelerator and take advantage of special pricing!

Discover how to develop your people while achieving important goals – Join Katie Anderson and Isao Yoshino for a workshop to Expand Your Chain of Learning. Enroll Now.

Enter for your chance to win a copy of Gene Kim’s and Dr. Steven J Spear’s new book Wiring the Winning Organization .

Take my new FREE Change Katalyst™ self-assessment now to fast-track your change leadership effectiveness!

problem solving type of question

Effective Questions to Support Problem-Solving Thinking

  • September 9, 2015

How do we ask effective questions to support problem-solving thinking?

Asking better questions — and learning to “break the telling habit” has been a passion of mine since I deeply started studying and applying the art of better question-asking around 2010.

In fact, the importance of asking questions (the right questions) is one of the foundations of my work. 

It’s one of the most popular topics here on my blog, in my custom learning programs for organizations, and my on-demand workshops (such as “Break the Telling Habit”) and guided coaching programs such as the Leading to Learn Accelerator.

Problem-Solving in Support of Developing People

Japanese copy of "Managing to Learn"

If your organization is looking for support in developing your people as problem solvers, I recommend starting with focusing on A3 problem-solving thinking.

It’s applicable to all leaders and not only hones people’s ability to solve problems and learn by going to see what is actually happening, but also is a great way of deepening coaching skills of asking effective questions and listening.

A Chain of Learning: Managing to Learn and Leading to Learn

John Shook’s book “Managing to Learn” is the best book I’ve found to describe the A3 problem-solving process and is the basis for this A3 thinking class.

Fun fact — One of the models for the character of Sanderson, the manager / coach in the book, is Isao Yoshino — my friend and subject of my best-selling book Learning to Lead, Leading to Learn — who was one of Shook’s first managers at Toyota in Japan. Shook wrote the foreword to my book and shares reflections about how he learned to ask questions and think more

I learned so much about asking questions from my mentor Margie Hagene, who herself was mentored by John Shook. This is my chain of learning that I now link with you to as we learn together.

There are many questions that we can ask in support of problem-solving, but they differ in the intention and outcome of problem solving ownership. I like to categorize questions into four categories, inspired by the work of Edgar Schein (whose books such as  Humble Inquiry and Helping have been foundational in my learning about asking questions).

Humble Open Inquiry

The most effective questions are either those of pure humble inquiry which are questions for which you don’t have the answer. Humble inquiry questions keep the problem-solving thinking with the problem owner, not the person asking the question.

Diagnostic Inquiry

Other effective questions that can be asked, once the problem is clearly defined, are diagnostic questions.

Diagnostic inquiry questions help the problem solver discover cause and effect, more clearly define the root causes for the gap between the way things should be happening and the way they are happening today, and the link between proposed countermeasures and the problem.

Prompting Inquiry: WARNING!

problem solving type of question

Prompting questions can take away the ownership of the thinking from the problem solver. These are closed-ended questions, multiple-choice questions, or your idea or suggestion with a question mark at the end.

These questions are like a wolf dressed up in sheep’s clothing — they are really your idea or suggestion with a question mark on it!

Process questions

It’s important to not just focus on solving the problem…but also to understand the process of solving the problem.

Good questions to ask during problem-solving coaching are not always related to the problem at hand, but to the human interaction. Process questions help us understand where we are in the problem-solving process or in the conversation.

If you are practicing asking more effective questions and wanting to improve, you could ask a process question such as “What question was most helpful to you to advance your thinking?”.

Want to learn more about questions — get the guide!

If you don’t have it yet you can  download my free guide “3 Tips to Break the ‘Telling Habit’” ! You’ll learn more about the types of questions, and how to ask them so that you too can break your telling habit!

Asking the right questions is often  about intention .

Get this resource to help you ask more effective questions today!

Examples of effective questions

Below are some of the examples of effective questions that participants in a two-day A3 Thinking and Coaching workshop I led many years ago. When coaching leaders to ask more effective questions, I like to capture real-time examples of questions being asked well.

What do you notice about these questions? (I’ll share some reflections below the image).

Examples of effective questions asked today to develop problem solving thinking in others. #lean #coaching @HCValue pic.twitter.com/aXzVhtI2bb — Katie Anderson (@kbjanderson) August 19, 2015

Qualities of effective questions

Just like in the image above, the most effective questions to support problem-solving share many of the following qualities:

  • Start with WHAT or HOW
  • Do not introduce a solution or idea

Additional categories of problem solving questions

Source: Harvard Business Review "Relearning the Art of Asking Questions", March 2015

An HBR article titled “Relearning the art of asking questions”  highlights four categories of questions that can be used when problem solving:

  • clarifying and

This can be a helpful structure for you to think about how to ask questions to help the other person think more creatively (divergent thinking) or get more clarity (convergent thinking).

Of the questions that are listed above from the A3 thinking class, how would you categorized them based on the HBR article’s definition?

Intention and Practice Leads to Improvement

It’s always inspiring to see, with some dedicated thinking time and practice time, how much people’s skills in problem solving, question asking, and listening can advance.

For example, in this class over two days, people got at least 90-120 minutes to work through a problem they own, and then practice 3-4 sessions of “catchball” (asking and receiving questions) with others as both the problem owner and the coach.

Their questions are dramatically improved on the second day with just some focused practice (see image of questions above)!

This is the same structure of practice that you can learn when you join my self-paced “Breaking the ‘Telling Habit’” workshop,  and unlock your team’s potential by discovering  how  to ask more effective questions. You can take the class at any time, at your own pace.

Additional resources

To learn more tips from me about asking effective questions, listening, and coaching for problem-solving, check out these additional articles:

  • “Leading Daily Improvement: Creating New Habits and Practices to Support Continuous Improvement”
  • “When to ‘A3’: 3 Problem-Solving Tools to Match the Complexity of Your Problem”
  • How to Ask Problem-Solving Questions
  • “How to Get Out of the Habit of Telling”
  • “Be More Than a Coach, be a Coach That Listens” .
  • How to ask questions even if you already know “the answer” .

What is your experience?

How have you practiced becoming a better problem solver or a better coach in support of problem solving? What have you found helpful in asking more effective questions?

Leading to Learn Accelerator

Learn to be a more intentional, people-centered leader in just 10 weeks, and build a meaningful legacy., be part of my chain of learning.

If you enjoyed this post and want to continue your learning journey with me, sign up for my periodic newsletter below where you’ll be the first to know about new articles on leadership, coaching, and continuous improvement, more author interviews and giveaways, and other opportunities to deepen your learning.

If you are already a subscriber, thank you!

Leading to Learn Accelerator

Join my Chain of Learning ® !

Register below for my newsletter and be the first to know about new articles, podcast episodes, and other inspiration to deepen your learning and leadership impact. let's grow our chain of learning -- together, related posts.

16-leveraging-analytical-systems-thinking-to-drive-improvement-mark-graban

16 | Leveraging Analytical Systems Thinking to Drive Improvement with Mark Graban

Ep15 5 Steps to Revitalize Lifelong Learning

15 | 5 Steps to Revitalize Lifelong Learning

Chain of Learning Episode 14 - Becoming an Astute Political Navigator with Betsy Jordyn

14 | Becoming an Astute Political Navigator with Betsy Jordyn

Ep13 3 Ways to Break the Telling Habit® and Create Greater Impact

13 | 3 Ways to Break the Telling Habit ® and Create Greater Impact

Keep learning with me.

Join my Chain of Learning ® and be the first to know about new blog posts, podcasts and learning opportunities.

© 2023 Katie Anderson | Privacy Policy | Terms of Use | Disclaimer

Get my free guide 3 Tips to Break The Telling Habit & learn how to ask better questions with intention.

3 Tips to Break the Telling Habit

Take my FREE Change Katalyst™ self-assessment now!

Sign up today to get a free copy of the Take my FREE Change Katalyst™ self-assessment.

Get your own copy of the 4-Box Problem-Solving Tool

Sign up today to get a free copy of the 4-box problem-solving tool.

Download My Plan-Do-Check-Adjust Framework

I want the "leading to learn: people centered practices to develop a culture of learning" webinar slides.

In addition to the webinar slides, you will also be signed up for Katie’s periodic newsletter, which you can opt out of at any time.

Get the Create a Life Tapestry Art Project Instructions

How to ask effective questions.

All newsletter subscribers get a copy of Isao Yoshino’s tips on “How to Ask Effective Questions” from our joint session on asking effective questions. Sign up here!

Download Learning to Lead, Leading to Learn Book Sample

Dive into Isao Yoshino’s Letter to the Reader and learn from his first mistake at Toyota. By sharing your information, you will also be subscribed to Katie’s periodic newsletter to be the first to know about new articles, events, and other learning experiences!

Download a PDF of the article "If You Think Lean is Inherently Japanese, Think Again"

Get personal improvement a3 coaching tips.

Develop your coaching skills to develop others. Download the Personal Improvement A3 Coaching guide!

Start living and leading with intention today!

Do you want improve yourself as a leader, coach or learner? Getting started with an intentional practice of daily reflection can accelerate your learning. Enter your email address below to download the Daily Reflection Template.

Isao Yoshino’s Leadership Credo

Sign up here and get your copy of Isao Yoshino’s leadership credo!

Learning to Lead Leading to Learn Book

Top 10 Toyota Leadership Lessons

Receive a PDF of the first top 10 leadership lessons and insights that I learned from Mr. Isao Yoshino, a leader at Toyota for over 40 years. These lessons and more inspired us to create the bestselling book “Learning to Lead, Leading to Learn.”

Access the Book Bonus Resources

Get the downloadable bonus material and additional resources referenced throughout the book. By sharing your information, you will receive access to all the bonus resources — as well as new resources as they become available.

35 problem-solving techniques and methods for solving complex problems

Problem solving workshop

Design your next session with SessionLab

Join the 150,000+ facilitators 
using SessionLab.

Recommended Articles

A step-by-step guide to planning a workshop, how to create an unforgettable training session in 8 simple steps, 47 useful online tools for workshop planning and meeting facilitation.

All teams and organizations encounter challenges as they grow. There are problems that might occur for teams when it comes to miscommunication or resolving business-critical issues . You may face challenges around growth , design , user engagement, and even team culture and happiness. In short, problem-solving techniques should be part of every team’s skillset.

Problem-solving methods are primarily designed to help a group or team through a process of first identifying problems and challenges , ideating possible solutions , and then evaluating the most suitable .

Finding effective solutions to complex problems isn’t easy, but by using the right process and techniques, you can help your team be more efficient in the process.

So how do you develop strategies that are engaging, and empower your team to solve problems effectively?

In this blog post, we share a series of problem-solving tools you can use in your next workshop or team meeting. You’ll also find some tips for facilitating the process and how to enable others to solve complex problems.

Let’s get started! 

How do you identify problems?

How do you identify the right solution.

  • Tips for more effective problem-solving

Complete problem-solving methods

  • Problem-solving techniques to identify and analyze problems
  • Problem-solving techniques for developing solutions

Problem-solving warm-up activities

Closing activities for a problem-solving process.

Before you can move towards finding the right solution for a given problem, you first need to identify and define the problem you wish to solve. 

Here, you want to clearly articulate what the problem is and allow your group to do the same. Remember that everyone in a group is likely to have differing perspectives and alignment is necessary in order to help the group move forward. 

Identifying a problem accurately also requires that all members of a group are able to contribute their views in an open and safe manner. It can be scary for people to stand up and contribute, especially if the problems or challenges are emotive or personal in nature. Be sure to try and create a psychologically safe space for these kinds of discussions.

Remember that problem analysis and further discussion are also important. Not taking the time to fully analyze and discuss a challenge can result in the development of solutions that are not fit for purpose or do not address the underlying issue.

Successfully identifying and then analyzing a problem means facilitating a group through activities designed to help them clearly and honestly articulate their thoughts and produce usable insight.

With this data, you might then produce a problem statement that clearly describes the problem you wish to be addressed and also state the goal of any process you undertake to tackle this issue.  

Finding solutions is the end goal of any process. Complex organizational challenges can only be solved with an appropriate solution but discovering them requires using the right problem-solving tool.

After you’ve explored a problem and discussed ideas, you need to help a team discuss and choose the right solution. Consensus tools and methods such as those below help a group explore possible solutions before then voting for the best. They’re a great way to tap into the collective intelligence of the group for great results!

Remember that the process is often iterative. Great problem solvers often roadtest a viable solution in a measured way to see what works too. While you might not get the right solution on your first try, the methods below help teams land on the most likely to succeed solution while also holding space for improvement.

Every effective problem solving process begins with an agenda . A well-structured workshop is one of the best methods for successfully guiding a group from exploring a problem to implementing a solution.

In SessionLab, it’s easy to go from an idea to a complete agenda . Start by dragging and dropping your core problem solving activities into place . Add timings, breaks and necessary materials before sharing your agenda with your colleagues.

The resulting agenda will be your guide to an effective and productive problem solving session that will also help you stay organized on the day!

problem solving type of question

Tips for more effective problem solving

Problem-solving activities are only one part of the puzzle. While a great method can help unlock your team’s ability to solve problems, without a thoughtful approach and strong facilitation the solutions may not be fit for purpose.

Let’s take a look at some problem-solving tips you can apply to any process to help it be a success!

Clearly define the problem

Jumping straight to solutions can be tempting, though without first clearly articulating a problem, the solution might not be the right one. Many of the problem-solving activities below include sections where the problem is explored and clearly defined before moving on.

This is a vital part of the problem-solving process and taking the time to fully define an issue can save time and effort later. A clear definition helps identify irrelevant information and it also ensures that your team sets off on the right track.

Don’t jump to conclusions

It’s easy for groups to exhibit cognitive bias or have preconceived ideas about both problems and potential solutions. Be sure to back up any problem statements or potential solutions with facts, research, and adequate forethought.

The best techniques ask participants to be methodical and challenge preconceived notions. Make sure you give the group enough time and space to collect relevant information and consider the problem in a new way. By approaching the process with a clear, rational mindset, you’ll often find that better solutions are more forthcoming.  

Try different approaches  

Problems come in all shapes and sizes and so too should the methods you use to solve them. If you find that one approach isn’t yielding results and your team isn’t finding different solutions, try mixing it up. You’ll be surprised at how using a new creative activity can unblock your team and generate great solutions.

Don’t take it personally 

Depending on the nature of your team or organizational problems, it’s easy for conversations to get heated. While it’s good for participants to be engaged in the discussions, ensure that emotions don’t run too high and that blame isn’t thrown around while finding solutions.

You’re all in it together, and even if your team or area is seeing problems, that isn’t necessarily a disparagement of you personally. Using facilitation skills to manage group dynamics is one effective method of helping conversations be more constructive.

Get the right people in the room

Your problem-solving method is often only as effective as the group using it. Getting the right people on the job and managing the number of people present is important too!

If the group is too small, you may not get enough different perspectives to effectively solve a problem. If the group is too large, you can go round and round during the ideation stages.

Creating the right group makeup is also important in ensuring you have the necessary expertise and skillset to both identify and follow up on potential solutions. Carefully consider who to include at each stage to help ensure your problem-solving method is followed and positioned for success.

Document everything

The best solutions can take refinement, iteration, and reflection to come out. Get into a habit of documenting your process in order to keep all the learnings from the session and to allow ideas to mature and develop. Many of the methods below involve the creation of documents or shared resources. Be sure to keep and share these so everyone can benefit from the work done!

Bring a facilitator 

Facilitation is all about making group processes easier. With a subject as potentially emotive and important as problem-solving, having an impartial third party in the form of a facilitator can make all the difference in finding great solutions and keeping the process moving. Consider bringing a facilitator to your problem-solving session to get better results and generate meaningful solutions!

Develop your problem-solving skills

It takes time and practice to be an effective problem solver. While some roles or participants might more naturally gravitate towards problem-solving, it can take development and planning to help everyone create better solutions.

You might develop a training program, run a problem-solving workshop or simply ask your team to practice using the techniques below. Check out our post on problem-solving skills to see how you and your group can develop the right mental process and be more resilient to issues too!

Design a great agenda

Workshops are a great format for solving problems. With the right approach, you can focus a group and help them find the solutions to their own problems. But designing a process can be time-consuming and finding the right activities can be difficult.

Check out our workshop planning guide to level-up your agenda design and start running more effective workshops. Need inspiration? Check out templates designed by expert facilitators to help you kickstart your process!

In this section, we’ll look at in-depth problem-solving methods that provide a complete end-to-end process for developing effective solutions. These will help guide your team from the discovery and definition of a problem through to delivering the right solution.

If you’re looking for an all-encompassing method or problem-solving model, these processes are a great place to start. They’ll ask your team to challenge preconceived ideas and adopt a mindset for solving problems more effectively.

  • Six Thinking Hats
  • Lightning Decision Jam
  • Problem Definition Process
  • Discovery & Action Dialogue
Design Sprint 2.0
  • Open Space Technology

1. Six Thinking Hats

Individual approaches to solving a problem can be very different based on what team or role an individual holds. It can be easy for existing biases or perspectives to find their way into the mix, or for internal politics to direct a conversation.

Six Thinking Hats is a classic method for identifying the problems that need to be solved and enables your team to consider them from different angles, whether that is by focusing on facts and data, creative solutions, or by considering why a particular solution might not work.

Like all problem-solving frameworks, Six Thinking Hats is effective at helping teams remove roadblocks from a conversation or discussion and come to terms with all the aspects necessary to solve complex problems.

2. Lightning Decision Jam

Featured courtesy of Jonathan Courtney of AJ&Smart Berlin, Lightning Decision Jam is one of those strategies that should be in every facilitation toolbox. Exploring problems and finding solutions is often creative in nature, though as with any creative process, there is the potential to lose focus and get lost.

Unstructured discussions might get you there in the end, but it’s much more effective to use a method that creates a clear process and team focus.

In Lightning Decision Jam, participants are invited to begin by writing challenges, concerns, or mistakes on post-its without discussing them before then being invited by the moderator to present them to the group.

From there, the team vote on which problems to solve and are guided through steps that will allow them to reframe those problems, create solutions and then decide what to execute on. 

By deciding the problems that need to be solved as a team before moving on, this group process is great for ensuring the whole team is aligned and can take ownership over the next stages. 

Lightning Decision Jam (LDJ)   #action   #decision making   #problem solving   #issue analysis   #innovation   #design   #remote-friendly   The problem with anything that requires creative thinking is that it’s easy to get lost—lose focus and fall into the trap of having useless, open-ended, unstructured discussions. Here’s the most effective solution I’ve found: Replace all open, unstructured discussion with a clear process. What to use this exercise for: Anything which requires a group of people to make decisions, solve problems or discuss challenges. It’s always good to frame an LDJ session with a broad topic, here are some examples: The conversion flow of our checkout Our internal design process How we organise events Keeping up with our competition Improving sales flow

3. Problem Definition Process

While problems can be complex, the problem-solving methods you use to identify and solve those problems can often be simple in design. 

By taking the time to truly identify and define a problem before asking the group to reframe the challenge as an opportunity, this method is a great way to enable change.

Begin by identifying a focus question and exploring the ways in which it manifests before splitting into five teams who will each consider the problem using a different method: escape, reversal, exaggeration, distortion or wishful. Teams develop a problem objective and create ideas in line with their method before then feeding them back to the group.

This method is great for enabling in-depth discussions while also creating space for finding creative solutions too!

Problem Definition   #problem solving   #idea generation   #creativity   #online   #remote-friendly   A problem solving technique to define a problem, challenge or opportunity and to generate ideas.

4. The 5 Whys 

Sometimes, a group needs to go further with their strategies and analyze the root cause at the heart of organizational issues. An RCA or root cause analysis is the process of identifying what is at the heart of business problems or recurring challenges. 

The 5 Whys is a simple and effective method of helping a group go find the root cause of any problem or challenge and conduct analysis that will deliver results. 

By beginning with the creation of a problem statement and going through five stages to refine it, The 5 Whys provides everything you need to truly discover the cause of an issue.

The 5 Whys   #hyperisland   #innovation   This simple and powerful method is useful for getting to the core of a problem or challenge. As the title suggests, the group defines a problems, then asks the question “why” five times, often using the resulting explanation as a starting point for creative problem solving.

5. World Cafe

World Cafe is a simple but powerful facilitation technique to help bigger groups to focus their energy and attention on solving complex problems.

World Cafe enables this approach by creating a relaxed atmosphere where participants are able to self-organize and explore topics relevant and important to them which are themed around a central problem-solving purpose. Create the right atmosphere by modeling your space after a cafe and after guiding the group through the method, let them take the lead!

Making problem-solving a part of your organization’s culture in the long term can be a difficult undertaking. More approachable formats like World Cafe can be especially effective in bringing people unfamiliar with workshops into the fold. 

World Cafe   #hyperisland   #innovation   #issue analysis   World Café is a simple yet powerful method, originated by Juanita Brown, for enabling meaningful conversations driven completely by participants and the topics that are relevant and important to them. Facilitators create a cafe-style space and provide simple guidelines. Participants then self-organize and explore a set of relevant topics or questions for conversation.

6. Discovery & Action Dialogue (DAD)

One of the best approaches is to create a safe space for a group to share and discover practices and behaviors that can help them find their own solutions.

With DAD, you can help a group choose which problems they wish to solve and which approaches they will take to do so. It’s great at helping remove resistance to change and can help get buy-in at every level too!

This process of enabling frontline ownership is great in ensuring follow-through and is one of the methods you will want in your toolbox as a facilitator.

Discovery & Action Dialogue (DAD)   #idea generation   #liberating structures   #action   #issue analysis   #remote-friendly   DADs make it easy for a group or community to discover practices and behaviors that enable some individuals (without access to special resources and facing the same constraints) to find better solutions than their peers to common problems. These are called positive deviant (PD) behaviors and practices. DADs make it possible for people in the group, unit, or community to discover by themselves these PD practices. DADs also create favorable conditions for stimulating participants’ creativity in spaces where they can feel safe to invent new and more effective practices. Resistance to change evaporates as participants are unleashed to choose freely which practices they will adopt or try and which problems they will tackle. DADs make it possible to achieve frontline ownership of solutions.

7. Design Sprint 2.0

Want to see how a team can solve big problems and move forward with prototyping and testing solutions in a few days? The Design Sprint 2.0 template from Jake Knapp, author of Sprint, is a complete agenda for a with proven results.

Developing the right agenda can involve difficult but necessary planning. Ensuring all the correct steps are followed can also be stressful or time-consuming depending on your level of experience.

Use this complete 4-day workshop template if you are finding there is no obvious solution to your challenge and want to focus your team around a specific problem that might require a shortcut to launching a minimum viable product or waiting for the organization-wide implementation of a solution.

8. Open space technology

Open space technology- developed by Harrison Owen – creates a space where large groups are invited to take ownership of their problem solving and lead individual sessions. Open space technology is a great format when you have a great deal of expertise and insight in the room and want to allow for different takes and approaches on a particular theme or problem you need to be solved.

Start by bringing your participants together to align around a central theme and focus their efforts. Explain the ground rules to help guide the problem-solving process and then invite members to identify any issue connecting to the central theme that they are interested in and are prepared to take responsibility for.

Once participants have decided on their approach to the core theme, they write their issue on a piece of paper, announce it to the group, pick a session time and place, and post the paper on the wall. As the wall fills up with sessions, the group is then invited to join the sessions that interest them the most and which they can contribute to, then you’re ready to begin!

Everyone joins the problem-solving group they’ve signed up to, record the discussion and if appropriate, findings can then be shared with the rest of the group afterward.

Open Space Technology   #action plan   #idea generation   #problem solving   #issue analysis   #large group   #online   #remote-friendly   Open Space is a methodology for large groups to create their agenda discerning important topics for discussion, suitable for conferences, community gatherings and whole system facilitation

Techniques to identify and analyze problems

Using a problem-solving method to help a team identify and analyze a problem can be a quick and effective addition to any workshop or meeting.

While further actions are always necessary, you can generate momentum and alignment easily, and these activities are a great place to get started.

We’ve put together this list of techniques to help you and your team with problem identification, analysis, and discussion that sets the foundation for developing effective solutions.

Let’s take a look!

  • The Creativity Dice
  • Fishbone Analysis
  • Problem Tree
  • SWOT Analysis
  • Agreement-Certainty Matrix
  • The Journalistic Six
  • LEGO Challenge
  • What, So What, Now What?
  • Journalists

Individual and group perspectives are incredibly important, but what happens if people are set in their minds and need a change of perspective in order to approach a problem more effectively?

Flip It is a method we love because it is both simple to understand and run, and allows groups to understand how their perspectives and biases are formed. 

Participants in Flip It are first invited to consider concerns, issues, or problems from a perspective of fear and write them on a flip chart. Then, the group is asked to consider those same issues from a perspective of hope and flip their understanding.  

No problem and solution is free from existing bias and by changing perspectives with Flip It, you can then develop a problem solving model quickly and effectively.

Flip It!   #gamestorming   #problem solving   #action   Often, a change in a problem or situation comes simply from a change in our perspectives. Flip It! is a quick game designed to show players that perspectives are made, not born.

10. The Creativity Dice

One of the most useful problem solving skills you can teach your team is of approaching challenges with creativity, flexibility, and openness. Games like The Creativity Dice allow teams to overcome the potential hurdle of too much linear thinking and approach the process with a sense of fun and speed. 

In The Creativity Dice, participants are organized around a topic and roll a dice to determine what they will work on for a period of 3 minutes at a time. They might roll a 3 and work on investigating factual information on the chosen topic. They might roll a 1 and work on identifying the specific goals, standards, or criteria for the session.

Encouraging rapid work and iteration while asking participants to be flexible are great skills to cultivate. Having a stage for idea incubation in this game is also important. Moments of pause can help ensure the ideas that are put forward are the most suitable. 

The Creativity Dice   #creativity   #problem solving   #thiagi   #issue analysis   Too much linear thinking is hazardous to creative problem solving. To be creative, you should approach the problem (or the opportunity) from different points of view. You should leave a thought hanging in mid-air and move to another. This skipping around prevents premature closure and lets your brain incubate one line of thought while you consciously pursue another.

11. Fishbone Analysis

Organizational or team challenges are rarely simple, and it’s important to remember that one problem can be an indication of something that goes deeper and may require further consideration to be solved.

Fishbone Analysis helps groups to dig deeper and understand the origins of a problem. It’s a great example of a root cause analysis method that is simple for everyone on a team to get their head around. 

Participants in this activity are asked to annotate a diagram of a fish, first adding the problem or issue to be worked on at the head of a fish before then brainstorming the root causes of the problem and adding them as bones on the fish. 

Using abstractions such as a diagram of a fish can really help a team break out of their regular thinking and develop a creative approach.

Fishbone Analysis   #problem solving   ##root cause analysis   #decision making   #online facilitation   A process to help identify and understand the origins of problems, issues or observations.

12. Problem Tree 

Encouraging visual thinking can be an essential part of many strategies. By simply reframing and clarifying problems, a group can move towards developing a problem solving model that works for them. 

In Problem Tree, groups are asked to first brainstorm a list of problems – these can be design problems, team problems or larger business problems – and then organize them into a hierarchy. The hierarchy could be from most important to least important or abstract to practical, though the key thing with problem solving games that involve this aspect is that your group has some way of managing and sorting all the issues that are raised.

Once you have a list of problems that need to be solved and have organized them accordingly, you’re then well-positioned for the next problem solving steps.

Problem tree   #define intentions   #create   #design   #issue analysis   A problem tree is a tool to clarify the hierarchy of problems addressed by the team within a design project; it represents high level problems or related sublevel problems.

13. SWOT Analysis

Chances are you’ve heard of the SWOT Analysis before. This problem-solving method focuses on identifying strengths, weaknesses, opportunities, and threats is a tried and tested method for both individuals and teams.

Start by creating a desired end state or outcome and bare this in mind – any process solving model is made more effective by knowing what you are moving towards. Create a quadrant made up of the four categories of a SWOT analysis and ask participants to generate ideas based on each of those quadrants.

Once you have those ideas assembled in their quadrants, cluster them together based on their affinity with other ideas. These clusters are then used to facilitate group conversations and move things forward. 

SWOT analysis   #gamestorming   #problem solving   #action   #meeting facilitation   The SWOT Analysis is a long-standing technique of looking at what we have, with respect to the desired end state, as well as what we could improve on. It gives us an opportunity to gauge approaching opportunities and dangers, and assess the seriousness of the conditions that affect our future. When we understand those conditions, we can influence what comes next.

14. Agreement-Certainty Matrix

Not every problem-solving approach is right for every challenge, and deciding on the right method for the challenge at hand is a key part of being an effective team.

The Agreement Certainty matrix helps teams align on the nature of the challenges facing them. By sorting problems from simple to chaotic, your team can understand what methods are suitable for each problem and what they can do to ensure effective results. 

If you are already using Liberating Structures techniques as part of your problem-solving strategy, the Agreement-Certainty Matrix can be an invaluable addition to your process. We’ve found it particularly if you are having issues with recurring problems in your organization and want to go deeper in understanding the root cause. 

Agreement-Certainty Matrix   #issue analysis   #liberating structures   #problem solving   You can help individuals or groups avoid the frequent mistake of trying to solve a problem with methods that are not adapted to the nature of their challenge. The combination of two questions makes it possible to easily sort challenges into four categories: simple, complicated, complex , and chaotic .  A problem is simple when it can be solved reliably with practices that are easy to duplicate.  It is complicated when experts are required to devise a sophisticated solution that will yield the desired results predictably.  A problem is complex when there are several valid ways to proceed but outcomes are not predictable in detail.  Chaotic is when the context is too turbulent to identify a path forward.  A loose analogy may be used to describe these differences: simple is like following a recipe, complicated like sending a rocket to the moon, complex like raising a child, and chaotic is like the game “Pin the Tail on the Donkey.”  The Liberating Structures Matching Matrix in Chapter 5 can be used as the first step to clarify the nature of a challenge and avoid the mismatches between problems and solutions that are frequently at the root of chronic, recurring problems.

Organizing and charting a team’s progress can be important in ensuring its success. SQUID (Sequential Question and Insight Diagram) is a great model that allows a team to effectively switch between giving questions and answers and develop the skills they need to stay on track throughout the process. 

Begin with two different colored sticky notes – one for questions and one for answers – and with your central topic (the head of the squid) on the board. Ask the group to first come up with a series of questions connected to their best guess of how to approach the topic. Ask the group to come up with answers to those questions, fix them to the board and connect them with a line. After some discussion, go back to question mode by responding to the generated answers or other points on the board.

It’s rewarding to see a diagram grow throughout the exercise, and a completed SQUID can provide a visual resource for future effort and as an example for other teams.

SQUID   #gamestorming   #project planning   #issue analysis   #problem solving   When exploring an information space, it’s important for a group to know where they are at any given time. By using SQUID, a group charts out the territory as they go and can navigate accordingly. SQUID stands for Sequential Question and Insight Diagram.

16. Speed Boat

To continue with our nautical theme, Speed Boat is a short and sweet activity that can help a team quickly identify what employees, clients or service users might have a problem with and analyze what might be standing in the way of achieving a solution.

Methods that allow for a group to make observations, have insights and obtain those eureka moments quickly are invaluable when trying to solve complex problems.

In Speed Boat, the approach is to first consider what anchors and challenges might be holding an organization (or boat) back. Bonus points if you are able to identify any sharks in the water and develop ideas that can also deal with competitors!   

Speed Boat   #gamestorming   #problem solving   #action   Speedboat is a short and sweet way to identify what your employees or clients don’t like about your product/service or what’s standing in the way of a desired goal.

17. The Journalistic Six

Some of the most effective ways of solving problems is by encouraging teams to be more inclusive and diverse in their thinking.

Based on the six key questions journalism students are taught to answer in articles and news stories, The Journalistic Six helps create teams to see the whole picture. By using who, what, when, where, why, and how to facilitate the conversation and encourage creative thinking, your team can make sure that the problem identification and problem analysis stages of the are covered exhaustively and thoughtfully. Reporter’s notebook and dictaphone optional.

The Journalistic Six – Who What When Where Why How   #idea generation   #issue analysis   #problem solving   #online   #creative thinking   #remote-friendly   A questioning method for generating, explaining, investigating ideas.

18. LEGO Challenge

Now for an activity that is a little out of the (toy) box. LEGO Serious Play is a facilitation methodology that can be used to improve creative thinking and problem-solving skills. 

The LEGO Challenge includes giving each member of the team an assignment that is hidden from the rest of the group while they create a structure without speaking.

What the LEGO challenge brings to the table is a fun working example of working with stakeholders who might not be on the same page to solve problems. Also, it’s LEGO! Who doesn’t love LEGO! 

LEGO Challenge   #hyperisland   #team   A team-building activity in which groups must work together to build a structure out of LEGO, but each individual has a secret “assignment” which makes the collaborative process more challenging. It emphasizes group communication, leadership dynamics, conflict, cooperation, patience and problem solving strategy.

19. What, So What, Now What?

If not carefully managed, the problem identification and problem analysis stages of the problem-solving process can actually create more problems and misunderstandings.

The What, So What, Now What? problem-solving activity is designed to help collect insights and move forward while also eliminating the possibility of disagreement when it comes to identifying, clarifying, and analyzing organizational or work problems. 

Facilitation is all about bringing groups together so that might work on a shared goal and the best problem-solving strategies ensure that teams are aligned in purpose, if not initially in opinion or insight.

Throughout the three steps of this game, you give everyone on a team to reflect on a problem by asking what happened, why it is important, and what actions should then be taken. 

This can be a great activity for bringing our individual perceptions about a problem or challenge and contextualizing it in a larger group setting. This is one of the most important problem-solving skills you can bring to your organization.

W³ – What, So What, Now What?   #issue analysis   #innovation   #liberating structures   You can help groups reflect on a shared experience in a way that builds understanding and spurs coordinated action while avoiding unproductive conflict. It is possible for every voice to be heard while simultaneously sifting for insights and shaping new direction. Progressing in stages makes this practical—from collecting facts about What Happened to making sense of these facts with So What and finally to what actions logically follow with Now What . The shared progression eliminates most of the misunderstandings that otherwise fuel disagreements about what to do. Voila!

20. Journalists  

Problem analysis can be one of the most important and decisive stages of all problem-solving tools. Sometimes, a team can become bogged down in the details and are unable to move forward.

Journalists is an activity that can avoid a group from getting stuck in the problem identification or problem analysis stages of the process.

In Journalists, the group is invited to draft the front page of a fictional newspaper and figure out what stories deserve to be on the cover and what headlines those stories will have. By reframing how your problems and challenges are approached, you can help a team move productively through the process and be better prepared for the steps to follow.

Journalists   #vision   #big picture   #issue analysis   #remote-friendly   This is an exercise to use when the group gets stuck in details and struggles to see the big picture. Also good for defining a vision.

Problem-solving techniques for developing solutions 

The success of any problem-solving process can be measured by the solutions it produces. After you’ve defined the issue, explored existing ideas, and ideated, it’s time to narrow down to the correct solution.

Use these problem-solving techniques when you want to help your team find consensus, compare possible solutions, and move towards taking action on a particular problem.

  • Improved Solutions
  • Four-Step Sketch
  • 15% Solutions
  • How-Now-Wow matrix
  • Impact Effort Matrix

21. Mindspin  

Brainstorming is part of the bread and butter of the problem-solving process and all problem-solving strategies benefit from getting ideas out and challenging a team to generate solutions quickly. 

With Mindspin, participants are encouraged not only to generate ideas but to do so under time constraints and by slamming down cards and passing them on. By doing multiple rounds, your team can begin with a free generation of possible solutions before moving on to developing those solutions and encouraging further ideation. 

This is one of our favorite problem-solving activities and can be great for keeping the energy up throughout the workshop. Remember the importance of helping people become engaged in the process – energizing problem-solving techniques like Mindspin can help ensure your team stays engaged and happy, even when the problems they’re coming together to solve are complex. 

MindSpin   #teampedia   #idea generation   #problem solving   #action   A fast and loud method to enhance brainstorming within a team. Since this activity has more than round ideas that are repetitive can be ruled out leaving more creative and innovative answers to the challenge.

22. Improved Solutions

After a team has successfully identified a problem and come up with a few solutions, it can be tempting to call the work of the problem-solving process complete. That said, the first solution is not necessarily the best, and by including a further review and reflection activity into your problem-solving model, you can ensure your group reaches the best possible result. 

One of a number of problem-solving games from Thiagi Group, Improved Solutions helps you go the extra mile and develop suggested solutions with close consideration and peer review. By supporting the discussion of several problems at once and by shifting team roles throughout, this problem-solving technique is a dynamic way of finding the best solution. 

Improved Solutions   #creativity   #thiagi   #problem solving   #action   #team   You can improve any solution by objectively reviewing its strengths and weaknesses and making suitable adjustments. In this creativity framegame, you improve the solutions to several problems. To maintain objective detachment, you deal with a different problem during each of six rounds and assume different roles (problem owner, consultant, basher, booster, enhancer, and evaluator) during each round. At the conclusion of the activity, each player ends up with two solutions to her problem.

23. Four Step Sketch

Creative thinking and visual ideation does not need to be confined to the opening stages of your problem-solving strategies. Exercises that include sketching and prototyping on paper can be effective at the solution finding and development stage of the process, and can be great for keeping a team engaged. 

By going from simple notes to a crazy 8s round that involves rapidly sketching 8 variations on their ideas before then producing a final solution sketch, the group is able to iterate quickly and visually. Problem-solving techniques like Four-Step Sketch are great if you have a group of different thinkers and want to change things up from a more textual or discussion-based approach.

Four-Step Sketch   #design sprint   #innovation   #idea generation   #remote-friendly   The four-step sketch is an exercise that helps people to create well-formed concepts through a structured process that includes: Review key information Start design work on paper,  Consider multiple variations , Create a detailed solution . This exercise is preceded by a set of other activities allowing the group to clarify the challenge they want to solve. See how the Four Step Sketch exercise fits into a Design Sprint

24. 15% Solutions

Some problems are simpler than others and with the right problem-solving activities, you can empower people to take immediate actions that can help create organizational change. 

Part of the liberating structures toolkit, 15% solutions is a problem-solving technique that focuses on finding and implementing solutions quickly. A process of iterating and making small changes quickly can help generate momentum and an appetite for solving complex problems.

Problem-solving strategies can live and die on whether people are onboard. Getting some quick wins is a great way of getting people behind the process.   

It can be extremely empowering for a team to realize that problem-solving techniques can be deployed quickly and easily and delineate between things they can positively impact and those things they cannot change. 

15% Solutions   #action   #liberating structures   #remote-friendly   You can reveal the actions, however small, that everyone can do immediately. At a minimum, these will create momentum, and that may make a BIG difference.  15% Solutions show that there is no reason to wait around, feel powerless, or fearful. They help people pick it up a level. They get individuals and the group to focus on what is within their discretion instead of what they cannot change.  With a very simple question, you can flip the conversation to what can be done and find solutions to big problems that are often distributed widely in places not known in advance. Shifting a few grains of sand may trigger a landslide and change the whole landscape.

25. How-Now-Wow Matrix

The problem-solving process is often creative, as complex problems usually require a change of thinking and creative response in order to find the best solutions. While it’s common for the first stages to encourage creative thinking, groups can often gravitate to familiar solutions when it comes to the end of the process. 

When selecting solutions, you don’t want to lose your creative energy! The How-Now-Wow Matrix from Gamestorming is a great problem-solving activity that enables a group to stay creative and think out of the box when it comes to selecting the right solution for a given problem.

Problem-solving techniques that encourage creative thinking and the ideation and selection of new solutions can be the most effective in organisational change. Give the How-Now-Wow Matrix a go, and not just for how pleasant it is to say out loud. 

How-Now-Wow Matrix   #gamestorming   #idea generation   #remote-friendly   When people want to develop new ideas, they most often think out of the box in the brainstorming or divergent phase. However, when it comes to convergence, people often end up picking ideas that are most familiar to them. This is called a ‘creative paradox’ or a ‘creadox’. The How-Now-Wow matrix is an idea selection tool that breaks the creadox by forcing people to weigh each idea on 2 parameters.

26. Impact and Effort Matrix

All problem-solving techniques hope to not only find solutions to a given problem or challenge but to find the best solution. When it comes to finding a solution, groups are invited to put on their decision-making hats and really think about how a proposed idea would work in practice. 

The Impact and Effort Matrix is one of the problem-solving techniques that fall into this camp, empowering participants to first generate ideas and then categorize them into a 2×2 matrix based on impact and effort.

Activities that invite critical thinking while remaining simple are invaluable. Use the Impact and Effort Matrix to move from ideation and towards evaluating potential solutions before then committing to them. 

Impact and Effort Matrix   #gamestorming   #decision making   #action   #remote-friendly   In this decision-making exercise, possible actions are mapped based on two factors: effort required to implement and potential impact. Categorizing ideas along these lines is a useful technique in decision making, as it obliges contributors to balance and evaluate suggested actions before committing to them.

27. Dotmocracy

If you’ve followed each of the problem-solving steps with your group successfully, you should move towards the end of your process with heaps of possible solutions developed with a specific problem in mind. But how do you help a group go from ideation to putting a solution into action? 

Dotmocracy – or Dot Voting -is a tried and tested method of helping a team in the problem-solving process make decisions and put actions in place with a degree of oversight and consensus. 

One of the problem-solving techniques that should be in every facilitator’s toolbox, Dot Voting is fast and effective and can help identify the most popular and best solutions and help bring a group to a decision effectively. 

Dotmocracy   #action   #decision making   #group prioritization   #hyperisland   #remote-friendly   Dotmocracy is a simple method for group prioritization or decision-making. It is not an activity on its own, but a method to use in processes where prioritization or decision-making is the aim. The method supports a group to quickly see which options are most popular or relevant. The options or ideas are written on post-its and stuck up on a wall for the whole group to see. Each person votes for the options they think are the strongest, and that information is used to inform a decision.

All facilitators know that warm-ups and icebreakers are useful for any workshop or group process. Problem-solving workshops are no different.

Use these problem-solving techniques to warm up a group and prepare them for the rest of the process. Activating your group by tapping into some of the top problem-solving skills can be one of the best ways to see great outcomes from your session.

  • Check-in/Check-out
  • Doodling Together
  • Show and Tell
  • Constellations
  • Draw a Tree

28. Check-in / Check-out

Solid processes are planned from beginning to end, and the best facilitators know that setting the tone and establishing a safe, open environment can be integral to a successful problem-solving process.

Check-in / Check-out is a great way to begin and/or bookend a problem-solving workshop. Checking in to a session emphasizes that everyone will be seen, heard, and expected to contribute. 

If you are running a series of meetings, setting a consistent pattern of checking in and checking out can really help your team get into a groove. We recommend this opening-closing activity for small to medium-sized groups though it can work with large groups if they’re disciplined!

Check-in / Check-out   #team   #opening   #closing   #hyperisland   #remote-friendly   Either checking-in or checking-out is a simple way for a team to open or close a process, symbolically and in a collaborative way. Checking-in/out invites each member in a group to be present, seen and heard, and to express a reflection or a feeling. Checking-in emphasizes presence, focus and group commitment; checking-out emphasizes reflection and symbolic closure.

29. Doodling Together  

Thinking creatively and not being afraid to make suggestions are important problem-solving skills for any group or team, and warming up by encouraging these behaviors is a great way to start. 

Doodling Together is one of our favorite creative ice breaker games – it’s quick, effective, and fun and can make all following problem-solving steps easier by encouraging a group to collaborate visually. By passing cards and adding additional items as they go, the workshop group gets into a groove of co-creation and idea development that is crucial to finding solutions to problems. 

Doodling Together   #collaboration   #creativity   #teamwork   #fun   #team   #visual methods   #energiser   #icebreaker   #remote-friendly   Create wild, weird and often funny postcards together & establish a group’s creative confidence.

30. Show and Tell

You might remember some version of Show and Tell from being a kid in school and it’s a great problem-solving activity to kick off a session.

Asking participants to prepare a little something before a workshop by bringing an object for show and tell can help them warm up before the session has even begun! Games that include a physical object can also help encourage early engagement before moving onto more big-picture thinking.

By asking your participants to tell stories about why they chose to bring a particular item to the group, you can help teams see things from new perspectives and see both differences and similarities in the way they approach a topic. Great groundwork for approaching a problem-solving process as a team! 

Show and Tell   #gamestorming   #action   #opening   #meeting facilitation   Show and Tell taps into the power of metaphors to reveal players’ underlying assumptions and associations around a topic The aim of the game is to get a deeper understanding of stakeholders’ perspectives on anything—a new project, an organizational restructuring, a shift in the company’s vision or team dynamic.

31. Constellations

Who doesn’t love stars? Constellations is a great warm-up activity for any workshop as it gets people up off their feet, energized, and ready to engage in new ways with established topics. It’s also great for showing existing beliefs, biases, and patterns that can come into play as part of your session.

Using warm-up games that help build trust and connection while also allowing for non-verbal responses can be great for easing people into the problem-solving process and encouraging engagement from everyone in the group. Constellations is great in large spaces that allow for movement and is definitely a practical exercise to allow the group to see patterns that are otherwise invisible. 

Constellations   #trust   #connection   #opening   #coaching   #patterns   #system   Individuals express their response to a statement or idea by standing closer or further from a central object. Used with teams to reveal system, hidden patterns, perspectives.

32. Draw a Tree

Problem-solving games that help raise group awareness through a central, unifying metaphor can be effective ways to warm-up a group in any problem-solving model.

Draw a Tree is a simple warm-up activity you can use in any group and which can provide a quick jolt of energy. Start by asking your participants to draw a tree in just 45 seconds – they can choose whether it will be abstract or realistic. 

Once the timer is up, ask the group how many people included the roots of the tree and use this as a means to discuss how we can ignore important parts of any system simply because they are not visible.

All problem-solving strategies are made more effective by thinking of problems critically and by exposing things that may not normally come to light. Warm-up games like Draw a Tree are great in that they quickly demonstrate some key problem-solving skills in an accessible and effective way.

Draw a Tree   #thiagi   #opening   #perspectives   #remote-friendly   With this game you can raise awarness about being more mindful, and aware of the environment we live in.

Each step of the problem-solving workshop benefits from an intelligent deployment of activities, games, and techniques. Bringing your session to an effective close helps ensure that solutions are followed through on and that you also celebrate what has been achieved.

Here are some problem-solving activities you can use to effectively close a workshop or meeting and ensure the great work you’ve done can continue afterward.

  • One Breath Feedback
  • Who What When Matrix
  • Response Cards

How do I conclude a problem-solving process?

All good things must come to an end. With the bulk of the work done, it can be tempting to conclude your workshop swiftly and without a moment to debrief and align. This can be problematic in that it doesn’t allow your team to fully process the results or reflect on the process.

At the end of an effective session, your team will have gone through a process that, while productive, can be exhausting. It’s important to give your group a moment to take a breath, ensure that they are clear on future actions, and provide short feedback before leaving the space. 

The primary purpose of any problem-solving method is to generate solutions and then implement them. Be sure to take the opportunity to ensure everyone is aligned and ready to effectively implement the solutions you produced in the workshop.

Remember that every process can be improved and by giving a short moment to collect feedback in the session, you can further refine your problem-solving methods and see further success in the future too.

33. One Breath Feedback

Maintaining attention and focus during the closing stages of a problem-solving workshop can be tricky and so being concise when giving feedback can be important. It’s easy to incur “death by feedback” should some team members go on for too long sharing their perspectives in a quick feedback round. 

One Breath Feedback is a great closing activity for workshops. You give everyone an opportunity to provide feedback on what they’ve done but only in the space of a single breath. This keeps feedback short and to the point and means that everyone is encouraged to provide the most important piece of feedback to them. 

One breath feedback   #closing   #feedback   #action   This is a feedback round in just one breath that excels in maintaining attention: each participants is able to speak during just one breath … for most people that’s around 20 to 25 seconds … unless of course you’ve been a deep sea diver in which case you’ll be able to do it for longer.

34. Who What When Matrix 

Matrices feature as part of many effective problem-solving strategies and with good reason. They are easily recognizable, simple to use, and generate results.

The Who What When Matrix is a great tool to use when closing your problem-solving session by attributing a who, what and when to the actions and solutions you have decided upon. The resulting matrix is a simple, easy-to-follow way of ensuring your team can move forward. 

Great solutions can’t be enacted without action and ownership. Your problem-solving process should include a stage for allocating tasks to individuals or teams and creating a realistic timeframe for those solutions to be implemented or checked out. Use this method to keep the solution implementation process clear and simple for all involved. 

Who/What/When Matrix   #gamestorming   #action   #project planning   With Who/What/When matrix, you can connect people with clear actions they have defined and have committed to.

35. Response cards

Group discussion can comprise the bulk of most problem-solving activities and by the end of the process, you might find that your team is talked out! 

Providing a means for your team to give feedback with short written notes can ensure everyone is head and can contribute without the need to stand up and talk. Depending on the needs of the group, giving an alternative can help ensure everyone can contribute to your problem-solving model in the way that makes the most sense for them.

Response Cards is a great way to close a workshop if you are looking for a gentle warm-down and want to get some swift discussion around some of the feedback that is raised. 

Response Cards   #debriefing   #closing   #structured sharing   #questions and answers   #thiagi   #action   It can be hard to involve everyone during a closing of a session. Some might stay in the background or get unheard because of louder participants. However, with the use of Response Cards, everyone will be involved in providing feedback or clarify questions at the end of a session.

Save time and effort discovering the right solutions

A structured problem solving process is a surefire way of solving tough problems, discovering creative solutions and driving organizational change. But how can you design for successful outcomes?

With SessionLab, it’s easy to design engaging workshops that deliver results. Drag, drop and reorder blocks  to build your agenda. When you make changes or update your agenda, your session  timing   adjusts automatically , saving you time on manual adjustments.

Collaborating with stakeholders or clients? Share your agenda with a single click and collaborate in real-time. No more sending documents back and forth over email.

Explore  how to use SessionLab  to design effective problem solving workshops or  watch this five minute video  to see the planner in action!

problem solving type of question

Over to you

The problem-solving process can often be as complicated and multifaceted as the problems they are set-up to solve. With the right problem-solving techniques and a mix of creative exercises designed to guide discussion and generate purposeful ideas, we hope we’ve given you the tools to find the best solutions as simply and easily as possible.

Is there a problem-solving technique that you are missing here? Do you have a favorite activity or method you use when facilitating? Let us know in the comments below, we’d love to hear from you! 

' src=

thank you very much for these excellent techniques

' src=

Certainly wonderful article, very detailed. Shared!

' src=

Your list of techniques for problem solving can be helpfully extended by adding TRIZ to the list of techniques. TRIZ has 40 problem solving techniques derived from methods inventros and patent holders used to get new patents. About 10-12 are general approaches. many organization sponsor classes in TRIZ that are used to solve business problems or general organiztational problems. You can take a look at TRIZ and dwonload a free internet booklet to see if you feel it shound be included per your selection process.

Leave a Comment Cancel reply

Your email address will not be published. Required fields are marked *

cycle of workshop planning steps

Going from a mere idea to a workshop that delivers results for your clients can feel like a daunting task. In this piece, we will shine a light on all the work behind the scenes and help you learn how to plan a workshop from start to finish. On a good day, facilitation can feel like effortless magic, but that is mostly the result of backstage work, foresight, and a lot of careful planning. Read on to learn a step-by-step approach to breaking the process of planning a workshop into small, manageable chunks.  The flow starts with the first meeting with a client to define the purposes of a workshop.…

problem solving type of question

How does learning work? A clever 9-year-old once told me: “I know I am learning something new when I am surprised.” The science of adult learning tells us that, in order to learn new skills (which, unsurprisingly, is harder for adults to do than kids) grown-ups need to first get into a specific headspace.  In a business, this approach is often employed in a training session where employees learn new skills or work on professional development. But how do you ensure your training is effective? In this guide, we'll explore how to create an effective training session plan and run engaging training sessions. As team leader, project manager, or consultant,…

problem solving type of question

Effective online tools are a necessity for smooth and engaging virtual workshops and meetings. But how do you choose the right ones? Do you sometimes feel that the good old pen and paper or MS Office toolkit and email leaves you struggling to stay on top of managing and delivering your workshop? Fortunately, there are plenty of online tools to make your life easier when you need to facilitate a meeting and lead workshops. In this post, we’ll share our favorite online tools you can use to make your job as a facilitator easier. In fact, there are plenty of free online workshop tools and meeting facilitation software you can…

Design your next workshop with SessionLab

Join the 150,000 facilitators using SessionLab

Sign up for free

Status.net

What is Problem Solving? (Steps, Techniques, Examples)

By Status.net Editorial Team on May 7, 2023 — 5 minutes to read

What Is Problem Solving?

Definition and importance.

Problem solving is the process of finding solutions to obstacles or challenges you encounter in your life or work. It is a crucial skill that allows you to tackle complex situations, adapt to changes, and overcome difficulties with ease. Mastering this ability will contribute to both your personal and professional growth, leading to more successful outcomes and better decision-making.

Problem-Solving Steps

The problem-solving process typically includes the following steps:

  • Identify the issue : Recognize the problem that needs to be solved.
  • Analyze the situation : Examine the issue in depth, gather all relevant information, and consider any limitations or constraints that may be present.
  • Generate potential solutions : Brainstorm a list of possible solutions to the issue, without immediately judging or evaluating them.
  • Evaluate options : Weigh the pros and cons of each potential solution, considering factors such as feasibility, effectiveness, and potential risks.
  • Select the best solution : Choose the option that best addresses the problem and aligns with your objectives.
  • Implement the solution : Put the selected solution into action and monitor the results to ensure it resolves the issue.
  • Review and learn : Reflect on the problem-solving process, identify any improvements or adjustments that can be made, and apply these learnings to future situations.

Defining the Problem

To start tackling a problem, first, identify and understand it. Analyzing the issue thoroughly helps to clarify its scope and nature. Ask questions to gather information and consider the problem from various angles. Some strategies to define the problem include:

  • Brainstorming with others
  • Asking the 5 Ws and 1 H (Who, What, When, Where, Why, and How)
  • Analyzing cause and effect
  • Creating a problem statement

Generating Solutions

Once the problem is clearly understood, brainstorm possible solutions. Think creatively and keep an open mind, as well as considering lessons from past experiences. Consider:

  • Creating a list of potential ideas to solve the problem
  • Grouping and categorizing similar solutions
  • Prioritizing potential solutions based on feasibility, cost, and resources required
  • Involving others to share diverse opinions and inputs

Evaluating and Selecting Solutions

Evaluate each potential solution, weighing its pros and cons. To facilitate decision-making, use techniques such as:

  • SWOT analysis (Strengths, Weaknesses, Opportunities, Threats)
  • Decision-making matrices
  • Pros and cons lists
  • Risk assessments

After evaluating, choose the most suitable solution based on effectiveness, cost, and time constraints.

Implementing and Monitoring the Solution

Implement the chosen solution and monitor its progress. Key actions include:

  • Communicating the solution to relevant parties
  • Setting timelines and milestones
  • Assigning tasks and responsibilities
  • Monitoring the solution and making adjustments as necessary
  • Evaluating the effectiveness of the solution after implementation

Utilize feedback from stakeholders and consider potential improvements. Remember that problem-solving is an ongoing process that can always be refined and enhanced.

Problem-Solving Techniques

During each step, you may find it helpful to utilize various problem-solving techniques, such as:

  • Brainstorming : A free-flowing, open-minded session where ideas are generated and listed without judgment, to encourage creativity and innovative thinking.
  • Root cause analysis : A method that explores the underlying causes of a problem to find the most effective solution rather than addressing superficial symptoms.
  • SWOT analysis : A tool used to evaluate the strengths, weaknesses, opportunities, and threats related to a problem or decision, providing a comprehensive view of the situation.
  • Mind mapping : A visual technique that uses diagrams to organize and connect ideas, helping to identify patterns, relationships, and possible solutions.

Brainstorming

When facing a problem, start by conducting a brainstorming session. Gather your team and encourage an open discussion where everyone contributes ideas, no matter how outlandish they may seem. This helps you:

  • Generate a diverse range of solutions
  • Encourage all team members to participate
  • Foster creative thinking

When brainstorming, remember to:

  • Reserve judgment until the session is over
  • Encourage wild ideas
  • Combine and improve upon ideas

Root Cause Analysis

For effective problem-solving, identifying the root cause of the issue at hand is crucial. Try these methods:

  • 5 Whys : Ask “why” five times to get to the underlying cause.
  • Fishbone Diagram : Create a diagram representing the problem and break it down into categories of potential causes.
  • Pareto Analysis : Determine the few most significant causes underlying the majority of problems.

SWOT Analysis

SWOT analysis helps you examine the Strengths, Weaknesses, Opportunities, and Threats related to your problem. To perform a SWOT analysis:

  • List your problem’s strengths, such as relevant resources or strong partnerships.
  • Identify its weaknesses, such as knowledge gaps or limited resources.
  • Explore opportunities, like trends or new technologies, that could help solve the problem.
  • Recognize potential threats, like competition or regulatory barriers.

SWOT analysis aids in understanding the internal and external factors affecting the problem, which can help guide your solution.

Mind Mapping

A mind map is a visual representation of your problem and potential solutions. It enables you to organize information in a structured and intuitive manner. To create a mind map:

  • Write the problem in the center of a blank page.
  • Draw branches from the central problem to related sub-problems or contributing factors.
  • Add more branches to represent potential solutions or further ideas.

Mind mapping allows you to visually see connections between ideas and promotes creativity in problem-solving.

Examples of Problem Solving in Various Contexts

In the business world, you might encounter problems related to finances, operations, or communication. Applying problem-solving skills in these situations could look like:

  • Identifying areas of improvement in your company’s financial performance and implementing cost-saving measures
  • Resolving internal conflicts among team members by listening and understanding different perspectives, then proposing and negotiating solutions
  • Streamlining a process for better productivity by removing redundancies, automating tasks, or re-allocating resources

In educational contexts, problem-solving can be seen in various aspects, such as:

  • Addressing a gap in students’ understanding by employing diverse teaching methods to cater to different learning styles
  • Developing a strategy for successful time management to balance academic responsibilities and extracurricular activities
  • Seeking resources and support to provide equal opportunities for learners with special needs or disabilities

Everyday life is full of challenges that require problem-solving skills. Some examples include:

  • Overcoming a personal obstacle, such as improving your fitness level, by establishing achievable goals, measuring progress, and adjusting your approach accordingly
  • Navigating a new environment or city by researching your surroundings, asking for directions, or using technology like GPS to guide you
  • Dealing with a sudden change, like a change in your work schedule, by assessing the situation, identifying potential impacts, and adapting your plans to accommodate the change.
  • How to Resolve Employee Conflict at Work [Steps, Tips, Examples]
  • How to Write Inspiring Core Values? 5 Steps with Examples
  • 30 Employee Feedback Examples (Positive & Negative)

Problem solving test: Pre-employment screening assessment to hire the best candidates

Summary of the problem solving test.

This Problem Solving test evaluates candidates’ ability to define problems and analyze data and textual information to make correct decisions. This test helps you identify candidates who use analytical skills to evaluate and respond to complex situations.

Covered skills

Creating and adjusting schedules, interpreting data and applying logic to make decisions, prioritizing and applying order based on a given set of rules, analyzing textual and numerical information to draw conclusions, use the problem solving test to hire.

Any role that involves managing constantly shifting variables with tight deadlines. This may include administrative assistants, project managers, planners, and people working in hospitality or sales.

You can use this test for free when you sign up for a free plan

graphic image for cognitive ability tests

About the Problem Solving test

Effective problem-solving requires a broad skill set that enables individuals, teams, and businesses to advance towards stated objectives. It involves the ability to define a problem, to break it down into manageable parts, to develop approaches to solve the (sub)problem using creativity and analytical thinking, and to execute flawlessly.

This problem solving test allows you to identify candidates who display these abilities. The test presents candidates with typical problem-solving scenarios like scheduling on the basis of a diverse set of conditions, identifying the right sequence of actions based on a number of business rules, and drawing conclusions based on textual and numerical information.

The test requires candidates to identify the right answers to the questions in a limited amount of time. Successful candidates can quickly identify the key elements of the problem and work through the problem at speed without making mistakes. This is a great test to include to check candidates' overall analytical skills.

problem solving type of question

The test is made by a subject-matter expert

The global IT industry has benefited from Anirban’s talents for over two decades. With a flawless reputation that precedes him, Anirban has earned a status as a sought-after agile project manager and consultant. He’s worked internationally as a Senior Project Manager with companies such as Ericsson, IBM, and T-Mobile.

Anirban’s love for learning helps him keep his skills sharp. He holds an MBA and a degree in engineering, is a certified Scrum Master, and has certifications in Prince2 and ITIL.

Crafted with expert knowledge

TestGorilla’s tests are created by subject matter experts. We assess potential subject-matter experts based on their knowledge, ability, and reputation. Before being published, each test is peer-reviewed by another expert, then calibrated using hundreds of test takers with relevant experience in the subject.

Our feedback mechanisms and unique algorithms allow our subject-matter experts to constantly improve their tests.

What our customers are saying

TestGorilla helps me to assess engineers rapidly. Creating assessments for different positions is easy due to pre-existing templates. You can create an assessment in less than 2 minutes. The interface is intuitive and it’s easy to visualize results per assessment.

G2

VP of engineering, mid-market (51-1000 FTE)

Any tool can have functions—bells and whistles. Not every tool comes armed with staff passionate about making the user experience positive.

The TestGorilla team only offers useful insights to user challenges, they engage in conversation.

For instance, I recently asked a question about a Python test I intended to implement. Instead of receiving “oh, that test would work perfectly for your solution,” or, “at this time we’re thinking about implementing a solution that may or may not…” I received a direct and straightforward answer with additional thoughts to help shape the solution.

I hope that TestGorilla realizes the value proposition in their work is not only the platform but the type of support that’s provided.

For a bit of context—I am a diversity recruiter trying to create a platform that removes bias from the hiring process and encourages the discovery of new and unseen talent.

Chief Talent Connector, small business (50 or fewer FTE)

Use TestGorilla to hire the best faster, easier and bias-free

Our screening tests identify the best candidates and make your hiring decisions faster, easier, and bias-free.

Learn how each candidate performs on the job using our library of 400+ scientifically validated tests.

Test candidates for job-specific skills like coding or digital marketing, as well as general skills like critical thinking. Our unique personality and culture tests allow you to get to know your applicants as real people – not just pieces of paper.

Give all applicants an equal, unbiased opportunity to showcase their skills with our data-driven and performance-based ranking system.

With TestGorilla, you’ll get the best talent from all walks of life, allowing for a stronger, more diverse workplace.

Our short, customizable assessments and easy-to-use interface can be accessed from any device, with no login required.

Add your company logo, color theme, and more to leave a lasting impression that candidates will appreciate.

problem solving type of question

Watch what TestGorilla can do for you

Create high-quality assessments, fast.

Building assessments is a breeze with TestGorilla. Get started with these simple steps.

Building assessments is quick and easy with TestGorilla. Just pick a name, select the tests you need, then add your own custom questions.

You can customize your assessments further by adding your company logo, color theme, and more. Build the assessment that works for you.

Send email invites directly from TestGorilla, straight from your ATS, or connect with candidates by sharing a direct link.

Have a long list of candidates? Easily send multiple invites with a single click. You can also customize your email invites.

Discover your strongest candidates with TestGorilla’s easy-to-read output reports, rankings, and analytics.

Easily switch from a comprehensive overview to a detailed analysis of your candidates. Then, go beyond the data by watching personalized candidate videos.

problem solving type of question

View a sample report

The Problem Solving test will be included in a PDF report along with the other tests from your assessment. You can easily download and share this report with colleagues and candidates.

problem solving type of question

Why problem-solving is a crucial skill

We’ve all been thrown a curveball at work. An unexpected problem crops up and we need to make a plan to solve it. This is called problem-solving and it’s an important skill in most job roles because employees will encounter difficult or complex situations or problems that need to be solved.

Interestingly, unlike some career skills, problem-solving translates to both an individual’s personal and professional lives, making it crucial to potential success. But this may make it harder for recruiters to find the right candidate for a job where problem-solving skills are needed. In this case, a problem-solving test can help you find the best candidate capable of handling situations that influence business functions.

Problem-solving in the workplace

In business, problem-solving relies on a candidate’s ability to create processes that mitigate or remove obstacles that prevent the company from achieving its goals. Consequently, these issues or situations can create a gap between desired outcomes and actual results. This means that problem-solving plays an important role in how employees meet this challenge and work through it.

Roles such as project management, administrative assistance, and planning work with changing circumstances and tight deadlines on a day-to-day basis. When recruiting for these roles, finding candidates who have good problem-solving skills is crucial to their success in the role.

To gain insight into a candidate’s skill in this area, you can use a problem-solving test. Through expert formulation, a skill-specific test can help you understand a candidate’s level of proficiency. And testing your applicants before you start the interview process can highlight the candidates with the skills most relevant to the role.

problem solving type of question

A process-driven skill

In the workplace, there are important steps that can contribute to a candidate’s ability to successfully solve problems. Let’s take a look:

Identify the problem

Problem-solving begins with accurately identifying the problem. This determining factor looks at whether a candidate can find the origin and the implications of the problem. It includes:

• Differentiating between fact and opinion

• Compiling data to determine the problem

• Identifying underlying causes

• Recognizing which processes are affected• Pinpointing the process standard

By accurately identifying the problem, individuals can proceed to the next step to solve the problem.

Determine alternative solutions

Once an individual has established the source of the problem, they can determine alternative solutions. The goal of plotting solutions to the problem is to remedy it and realign it with business goals. A creative problem-solving test may identify whether an individual has the competency to determine solutions. Key competencies in seeking solutions include:

• Establishing alternative solutions that align with business goals

• Determining whether a problem needs short- or long-term solutions

• Evaluating how solutions may impact on resources; and determining if there are any barriers to implementing the solutions.

Although any problem can have multiple solutions, the simplest or fastest one may not always be the best course of action. This is where solution comparison comes into play.

Compare solutions and plot a course

Once all possible solutions are determined, it is important to compare them. This involves evaluating each solution without bias to determine the optimal solution to the problem.

Through the evaluation process, the individual should rule out options that do not align with company goals, may take too much time and/or resources, or are unrealistic in their approach.

Some considerations when determining the best solution include the likelihood of solution implementation, whether all parties involved will accept the solution, and how it fits in with business goals. Additionally, it is important to note that the goal of the optimal solution is to solve the problem without causing additional or unanticipated problems.

In essence, problem-solving is about finding solutions that cause as little disruption as possible and correcting a project’s course.

Implement the solution

The last stage in problem-solving is the implementation of the final step. This step focuses on the remedial solution and requires continuous evaluation to ensure its effective implementation. For you as a recruiter, knowing if a candidate can find a solution as well as implement it may be important to the goals of the role.

Continually evaluating the solution will give the individual insight into whether the project goals are aligned, whether all stakeholders accept the new solution and whether the outcomes are managed effectively.

problem solving type of question

Considerations for recruiters

When hiring for a role in which problem-solving skills are crucial, it may be beneficial to test a candidate’s ability to define problems and analyze data and textual information to make decisions that best serve the business. Some of the considerations for a problem-solving test include:

Schedules are living documents that need to adapt as eventualities come into play. Candidates should be able to understand what they can realistically achieve with the time and how to adjust schedules to account for variable outcomes.

Data-driven decision-making should inform a course of action before an individual commits to it. For recruiters, this means candidates should have an aptitude for aligning data with business goals and making actionable decisions.

By using prioritization rules and supporting information, candidates can determine which project tasks take priority. This system aims to optimize resources for project delivery.

Examining textual and numerical information to reveal patterns, relationships, and trends can tell the candidate what connection exists among variables. Conclusions can then be drawn from the data to gain an accurate assessment of the overall situation.

When broken down, problem-solving is a skill that relies on a variety of disciplines to achieve success. Although this skill is transferable to many job roles, determining candidates’ proficiency can be difficult, so it can be beneficial to recruiters to use a problem-solving test to review candidates’ aptitude when recruiting for a role.

Using a pre-formulated problem-solving test will enable you to quickly assess your candidates and help you recruit the best person for the role.

An assessment is the total package of tests and custom questions that you put together to evaluate your candidates. Each individual test within an assessment is designed to test something specific, such as a job skill or language. An assessment can consist of up to 5 tests and 20 custom questions. You can have candidates respond to your custom questions in several ways, such as with a personalized video.

Yes! Custom questions are great for testing candidates in your own unique way. We support the following question types: video, multiple-choice, coding, file upload, and essay. Besides adding your own custom questions, you can also create your own tests.

A video question is a specific type of custom question you can add to your assessment. Video questions let you create a question and have your candidates use their webcam to record a video response. This is an excellent way to see how a candidate would conduct themselves in a live interview, and is especially useful for sales and hospitality roles. Some good examples of things to ask for video questions would be "Why do you want to work for our company?" or "Try to sell me an item you have on your desk right now."

Besides video questions, you can also add the following types of custom questions: multiple-choice, coding, file upload, and essay. Multiple-choice lets your candidates choose from a list of answers that you provide, coding lets you create a coding problem for them to solve, file upload allows your candidates to upload a file that you request (such as a resume or portfolio), and essay allows an open-ended text response to your question. You can learn more about different custom question types here .

Yes! You can add your own logo and company color theme to your assessments. This is a great way to leave a positive and lasting brand impression on your candidates.

Our team is always here to help. After you sign up, we’ll reach out to guide you through the first steps of setting up your TestGorilla account. If you have any further questions, you can contact our support team via email, chat or call. We also offer detailed guides in our extensive help center .

It depends! We offer five free tests, or unlimited access to our library of 400+ tests with the price based on your company size. Find more information on our pricing plans here , or speak to one of our sales team for your personalized demo and learn how we can help you revolutionize hiring today.

Yes. You can add up to five tests to each assessment.

We recommend using our assessment software as a pre-screening tool at the beginning of your recruitment process. You can add a link to the assessment in your job post or directly invite candidates by email.

TestGorilla replaces traditional resume screening with a much more reliable and efficient process, designed to find the most skilled candidates earlier and faster.

We offer the following cognitive ability tests : Numerical Reasoning, Problem Solving, Attention to Detail, Reading Comprehension, and Critical Thinking.

Our cognitive ability tests allow you to test for skills that are difficult to evaluate in an interview. Check out our blog on why these tests are so useful and how to choose the best one for your assessment.

Related tests

Computational thinking, basic math calculations, mechanical reasoning, understanding instructions, attention to detail (visual), intermediate math, basic triple-digit math, basic double-digit math, attention to detail (textual), numerical reasoning.

Watch CBS News

Teens come up with trigonometry proof for Pythagorean Theorem, a problem that stumped math world for centuries

By Bill Whitaker

May 5, 2024 / 7:00 PM EDT / CBS News

As the school year ends, many students will be only too happy to see math classes in their rearview mirrors. It may seem to some of us non-mathematicians that geometry and trigonometry were created by the Greeks as a form of torture, so imagine our amazement when we heard two high school seniors had proved a mathematical puzzle that was thought to be impossible for 2,000 years. 

We met Calcea Johnson and Ne'Kiya Jackson at their all-girls Catholic high school in New Orleans. We expected to find two mathematical prodigies.

Instead, we found at St. Mary's Academy , all students are told their possibilities are boundless.

Come Mardi Gras season, New Orleans is alive with colorful parades, replete with floats, and beads, and high school marching bands.

In a city where uniqueness is celebrated, St. Mary's stands out – with young African American women playing trombones and tubas, twirling batons and dancing - doing it all, which defines St. Mary's, students told us.

Junior Christina Blazio says the school instills in them they have the ability to accomplish anything. 

Christina Blazio: That is kinda a standard here. So we aim very high - like, our aim is excellence for all students. 

The private Catholic elementary and high school sits behind the Sisters of the Holy Family Convent in New Orleans East. The academy was started by an African American nun for young Black women just after the Civil War. The church still supports the school with the help of alumni.

In December 2022, seniors Ne'Kiya Jackson and Calcea Johnson were working on a school-wide math contest that came with a cash prize.

Ne'Kiya Jackson and Calcea Johnson

Ne'Kiya Jackson: I was motivated because there was a monetary incentive.

Calcea Johnson: 'Cause I was like, "$500 is a lot of money. So I-- I would like to at least try."

Both were staring down the thorny bonus question.

Bill Whitaker: So tell me, what was this bonus question?

Calcea Johnson: It was to create a new proof of the Pythagorean Theorem. And it kind of gave you a few guidelines on how would you start a proof.

The seniors were familiar with the Pythagorean Theorem, a fundamental principle of geometry. You may remember it from high school: a² + b² = c². In plain English, when you know the length of two sides of a right triangle, you can figure out the length of the third.

Both had studied geometry and some trigonometry, and both told us math was not easy. What no one told  them  was there had been more than 300 documented proofs of the Pythagorean Theorem using algebra and geometry, but for 2,000 years a proof using trigonometry was thought to be impossible, … and that was the bonus question facing them.

Bill Whitaker: When you looked at the question did you think, "Boy, this is hard"?

Ne'Kiya Jackson: Yeah. 

Bill Whitaker: What motivated you to say, "Well, I'm going to try this"?

Calcea Johnson: I think I was like, "I started something. I need to finish it." 

Bill Whitaker: So you just kept on going.

Calcea Johnson: Yeah.

For two months that winter, they spent almost all their free time working on the proof.

CeCe Johnson: She was like, "Mom, this is a little bit too much."

CeCe and Cal Johnson are Calcea's parents.

CeCe Johnson:   So then I started looking at what she really was doing. And it was pages and pages and pages of, like, over 20 or 30 pages for this one problem.

Cal Johnson: Yeah, the garbage can was full of papers, which she would, you know, work out the problems and-- if that didn't work she would ball it up, throw it in the trash. 

Bill Whitaker: Did you look at the problem? 

Neliska Jackson is Ne'Kiya's mother.

Neliska Jackson: Personally I did not. 'Cause most of the time I don't understand what she's doing (laughter).

Michelle Blouin Williams: What if we did this, what if I write this? Does this help? ax² plus ….

Their math teacher, Michelle Blouin Williams, initiated the math contest.

Michelle Blouin Williams

Bill Whitaker: And did you think anyone would solve it?

Michelle Blouin Williams: Well, I wasn't necessarily looking for a solve. So, no, I didn't—

Bill Whitaker: What were you looking for?

Michelle Blouin Williams: I was just looking for some ingenuity, you know—

Calcea and Ne'Kiya delivered on that! They tried to explain their groundbreaking work to 60 Minutes. Calcea's proof is appropriately titled the Waffle Cone.

Calcea Johnson: So to start the proof, we start with just a regular right triangle where the angle in the corner is 90°. And the two angles are alpha and beta.

Bill Whitaker: Uh-huh

Calcea Johnson: So then what we do next is we draw a second congruent, which means they're equal in size. But then we start creating similar but smaller right triangles going in a pattern like this. And then it continues for infinity. And eventually it creates this larger waffle cone shape.

Calcea Johnson: Am I going a little too—

Bill Whitaker: You've been beyond me since the beginning. (laughter) 

Bill Whitaker: So how did you figure out the proof?

Ne'Kiya Jackson: Okay. So you have a right triangle, 90° angle, alpha and beta.

Bill Whitaker: Then what did you do?

Bill Whitaker with Calcea Johnson and Ne'Kiya Jackson

Ne'Kiya Jackson: Okay, I have a right triangle inside of the circle. And I have a perpendicular bisector at OP to divide the triangle to make that small right triangle. And that's basically what I used for the proof. That's the proof.

Bill Whitaker: That's what I call amazing.

Ne'Kiya Jackson: Well, thank you.

There had been one other documented proof of the theorem using trigonometry by mathematician Jason Zimba in 2009 – one in 2,000 years. Now it seems Ne'Kiya and Calcea have joined perhaps the most exclusive club in mathematics. 

Bill Whitaker: So you both independently came up with proof that only used trigonometry.

Ne'Kiya Jackson: Yes.

Bill Whitaker: So are you math geniuses?

Calcea Johnson: I think that's a stretch. 

Bill Whitaker: If not genius, you're really smart at math.

Ne'Kiya Jackson: Not at all. (laugh) 

To document Calcea and Ne'Kiya's work, math teachers at St. Mary's submitted their proofs to an American Mathematical Society conference in Atlanta in March 2023.

Ne'Kiya Jackson: Well, our teacher approached us and was like, "Hey, you might be able to actually present this," I was like, "Are you joking?" But she wasn't. So we went. I got up there. We presented and it went well, and it blew up.

Bill Whitaker: It blew up.

Calcea Johnson: Yeah. 

Ne'Kiya Jackson: It blew up.

Bill Whitaker: Yeah. What was the blowup like?

Calcea Johnson: Insane, unexpected, crazy, honestly.

It took millenia to prove, but just a minute for word of their accomplishment to go around the world. They got a write-up in South Korea and a shout-out from former first lady Michelle Obama, a commendation from the governor and keys to the city of New Orleans. 

Bill Whitaker: Why do you think so many people found what you did to be so impressive?

Ne'Kiya Jackson: Probably because we're African American, one. And we're also women. So I think-- oh, and our age. Of course our ages probably played a big part.

Bill Whitaker: So you think people were surprised that young African American women, could do such a thing?

Calcea Johnson: Yeah, definitely.

Ne'Kiya Jackson: I'd like to actually be celebrated for what it is. Like, it's a great mathematical achievement.

Achievement, that's a word you hear often around St. Mary's academy. Calcea and Ne'Kiya follow a long line of barrier-breaking graduates. 

The late queen of Creole cooking, Leah Chase , was an alum. so was the first African-American female New Orleans police chief, Michelle Woodfork …

And judge for the Fifth Circuit Court of Appeals, Dana Douglas. Math teacher Michelle Blouin Williams told us Calcea and Ne'Kiya are typical St. Mary's students.  

Bill Whitaker: They're not unicorns.

Michelle Blouin Williams: Oh, no no. If they are unicorns, then every single lady that has matriculated through this school is a beautiful, Black unicorn.

Pamela Rogers: You're good?

Pamela Rogers, St. Mary's president and interim principal, told us the students hear that message from the moment they walk in the door.

St. Mary's Academy president and interim principal Pamela Rogers

Pamela Rogers: We believe all students can succeed, all students can learn. It does not matter the environment that you live in. 

Bill Whitaker: So when word went out that two of your students had solved this almost impossible math problem, were they universally applauded?

Pamela Rogers: In this community, they were greatly applauded. Across the country, there were many naysayers.

Bill Whitaker: What were they saying?

Pamela Rogers: They were saying, "Oh, they could not have done it. African Americans don't have the brains to do it." Of course, we sheltered our girls from that. But we absolutely did not expect it to come in the volume that it came.  

Bill Whitaker: And after such a wonderful achievement.

Pamela Rogers: People-- have a vision of who can be successful. And-- to some people, it is not always an African American female. And to us, it's always an African American female.

Gloria Ladson-Billings: What we know is when teachers lay out some expectations that say, "You can do this," kids will work as hard as they can to do it.

Gloria Ladson-Billings, professor emeritus at the University of Wisconsin, has studied how best to teach African American students. She told us an encouraging teacher can change a life.

Bill Whitaker: And what's the difference, say, between having a teacher like that and a whole school dedicated to the excellence of these students?

Gloria Ladson-Billings: So a whole school is almost like being in Heaven. 

Bill Whitaker: What do you mean by that?

Bill Whitaker and Gloria Ladson-Billings

Gloria Ladson-Billings: Many of our young people have their ceilings lowered, that somewhere around fourth or fifth grade, their thoughts are, "I'm not going to be anything special." What I think is probably happening at St. Mary's is young women come in as, perhaps, ninth graders and are told, "Here's what we expect to happen. And here's how we're going to help you get there."

At St. Mary's, half the students get scholarships, subsidized by fundraising to defray the $8,000 a year tuition. Here, there's no test to get in, but expectations are high and rules are strict: no cellphones, modest skirts, hair must be its natural color.

Students Rayah Siddiq, Summer Forde, Carissa Washington, Tatum Williams and Christina Blazio told us they appreciate the rules and rigor.

Rayah Siddiq: Especially the standards that they set for us. They're very high. And I don't think that's ever going to change.

Bill Whitaker: So is there a heart, a philosophy, an essence to St. Mary's?

Summer Forde: The sisterhood—

Carissa Washington: Sisterhood.

Tatum Williams: Sisterhood.

Bill Whitaker: The sisterhood?

Voices: Yes.

Bill Whitaker: And you don't mean the nuns. You mean-- (laughter)

Christina Blazio: I mean, yeah. The community—

Bill Whitaker: So when you're here, there's just no question that you're going to go on to college.

Rayah Siddiq: College is all they talk about. (laughter) 

Pamela Rogers: … and Arizona State University (Cheering)

Principal Rogers announces to her 615 students the colleges where every senior has been accepted.

Bill Whitaker: So for 17 years, you've had a 100% graduation rate—

Pamela Rogers: Yes.

Bill Whitaker: --and a 100% college acceptance rate?

Pamela Rogers: That's correct.

Last year when Ne'Kiya and Calcea graduated, all their classmates went to college and got scholarships. Ne'Kiya got a full ride to the pharmacy school at Xavier University in New Orleans. Calcea, the class valedictorian, is studying environmental engineering at Louisiana State University.

Bill Whitaker: So wait a minute. Neither one of you is going to pursue a career in math?

Both: No. (laugh)

Calcea Johnson: I may take up a minor in math. But I don't want that to be my job job.

Ne'Kiya Jackson: Yeah. People might expect too much out of me if (laugh) I become a mathematician. (laugh)

But math is not completely in their rear-view mirrors. This spring they submitted their high school proofs for final peer review and publication … and are still working on further proofs of the Pythagorean Theorem. Since their first two …

Calcea Johnson: We found five. And then we found a general format that could potentially produce at least five additional proofs.

Bill Whitaker: And you're not math geniuses?

Bill Whitaker: I'm not buying it. (laughs)

Produced by Sara Kuzmarov. Associate producer, Mariah B. Campbell. Edited by Daniel J. Glucksman.

Bill Whitaker

Bill Whitaker is an award-winning journalist and 60 Minutes correspondent who has covered major news stories, domestically and across the globe, for more than four decades with CBS News.

More from CBS News

Israel preparing for Rafah invasion in Gaza amid increasing tension with U.S.

60 Minutes Archive: Coverage of North Korea

How much does it cost to file for bankruptcy?

As a Social Security cut looms, should seniors buy long-term care insurance now?

IMAGES

  1. 10 Problem Solving Skills Examples: How To Improve

    problem solving type of question

  2. Problem-Solving Strategies: Definition and 5 Techniques to Try

    problem solving type of question

  3. 39 Best Problem-Solving Examples (2024)

    problem solving type of question

  4. Introduction to Problem Solving Skills

    problem solving type of question

  5. Problem Solving Questions: What are They

    problem solving type of question

  6. Advanced Problem Solving Test: Questions and Answers

    problem solving type of question

VIDEO

  1. Problem solving type of CAI

  2. NLPP Problem solving| Type 2:With N Variable & one equality constraint #nlpp #m4 #engineering

  3. Unit 6

  4. Types of People

  5. solving selection problems if statement part 1

  6. solving selection problems if statement part 2

COMMENTS

  1. 50 Interview Questions About Problem Solving (With Answers)

    Demonstrating your ability to tackle challenges effectively can set you apart from other applicants. Here are five tips to help you showcase your problem-solving skills during an interview: 1. Use the STAR Method. Structure your responses using the Situation, Task, Action, and Result (STAR) method.

  2. 8 Common Problem-Solving Interview Questions and Answers

    Also known as analytical skills interview questions, these questions will often focus on specific instances when the candidate analyzed a situation or had to solve a problem, including what steps they took to gather and understand the necessary information before solving the problem. These types of questions help employers better understand how ...

  3. Top 20 Problem Solving Interview Questions (Example Answers Included)

    MIKE'S TIP: When you're answering this question, quantify the details. This gives your answer critical context and scale, showcasing the degree of challenge and strength of the accomplishment. That way, your answer is powerful, compelling, and, above all, thorough. 2. Describe a time where you made a mistake.

  4. Problem-Solving Interview Questions: How-to + Examples

    To put these skills to the test, recruiters use "problem-solving" job interview questions, also known as analytical questions. Here are some common ones: Tell me about a situation where you had to solve a difficult problem. Give me a specific example of a time when you used good judgment and logic in solving a problem.

  5. 10 Proven Problem-solving Interview Questions [+Answers]

    Problem-solving interview questions show how candidates: Approach complex issues. Analyze data to understand the root of the problem. Perform under stressful and unexpected situations. React when their beliefs are challenged. Identify candidates who are results-oriented with interview questions that assess problem-solving skills. Look for ...

  6. 15 Common Problem-Solving Interview Questions

    Here are a few examples of technical problem-solving questions: 1. Mini-Max Sum. This well-known challenge, which asks the interviewee to find the maximum and minimum sum among an array of given numbers, is based on a basic but important programming concept called sorting, as well as integer overflow.

  7. 6 Common Problem-Solving Interview Questions and Answers

    Types of Problem-Solving Interview Questions Fact-Finding Questions. These questions focus on your ability to collect and analyze information, as well as make deductions based on your findings. Employers want to see that you can dig deep and uncover relevant points before arriving at a conclusion. A couple examples of fact-finding questions ...

  8. Problem-Solving Interview Questions And Answers (With Examples)

    Problem-solving questions assess a candidate's ability to think on their feet, handle pressure, and find creative solutions to complex problems. ... Regardless of the type of interview question you're responding to, you should always make it your top priority to be as honest as possible about your strengths and weaknesses.

  9. 8 Problem-Solving Interview Questions You Should Ask

    8 Examples of Common Problem-Solving Interview Questions and Answers. Now for the main course of this article. We're going to dive into eight types of example problem-solving questions that you can use during interviews, explaining why they are relevant and what makes a strong answer. 1. The challenging situation.

  10. The Best Interview Questions for Assessing Problem Solving Skills

    A candidate's problem-solving skills are only as good as their ability to understand the quality of their solutions and the tradeoffs of their impact. Summary. Carefully crafted problem-solving questions can play a crucial role in assessing a candidate's ability to navigate difficult situations and devise effective solutions.

  11. 25 Problem Solving Interview Questions & Answers

    Practice 25 Problem Solving Interview Questions. Written by professional interviewers with 175 answer examples and 10 community answer examples. MockQuestions. Go. ... Whether you are the type to jump right into solving a problem or you are more methodical in your approach, highlight to the interviewer that you are capable of handling issues ...

  12. 26 Expert-Backed Problem Solving Examples

    The example interview responses are structured using the STAR method and are categorized into the top 5 key problem-solving skills recruiters look for in a candidate. 1. Analytical Thinking. Situation: In my previous role as a data analyst, our team encountered a significant drop in website traffic.

  13. Top 17 Problem Solving Interview Questions with Answers (2024)

    Problem-Solving Interview Questions: Common Mistakes to Avoid. Below are relevant tips to aid you in answering problem-solving interview questions. Avoid Giving Easy Responses- Individuals who opt for easier responses are considered to lack critical thinking. Avoid Giving Hasty Responses- Take your time in addressing the issue at hand and make ...

  14. Problem-Solving Strategies and Obstacles

    Problem-solving helps you figure out how to achieve these desires. The problem-solving process involves: Discovery of the problem. Deciding to tackle the issue. Seeking to understand the problem more fully. Researching available options or solutions. Taking action to resolve the issue.

  15. What is Problem Solving? Steps, Process & Techniques

    Finding a suitable solution for issues can be accomplished by following the basic four-step problem-solving process and methodology outlined below. Step. Characteristics. 1. Define the problem. Differentiate fact from opinion. Specify underlying causes. Consult each faction involved for information. State the problem specifically.

  16. The Problem-Solving Process

    Problem-solving is a mental process that involves discovering, analyzing, and solving problems. The ultimate goal of problem-solving is to overcome obstacles and find a solution that best resolves the issue. The best strategy for solving a problem depends largely on the unique situation. In some cases, people are better off learning everything ...

  17. Effective Questions to Support Problem-Solving Thinking

    Beware of asking prompting questions. Prompting questions can take away the ownership of the thinking from the problem solver. These are closed-ended questions, multiple-choice questions, or your idea or suggestion with a question mark at the end. These questions are like a wolf dressed up in sheep's clothing — they are really your idea or ...

  18. 31 Situational Interview Questions (With Example Answers)

    Situational interview questions focus on how you'll handle real-life scenarios you may encounter in the workplace and how you've handled similar situations in previous roles. Asking these questions helps employers better understand your thought process and assess your problem-solving, self-management and communication skills. They also give you ...

  19. 35 problem-solving techniques and methods for solving complex problems

    6. Discovery & Action Dialogue (DAD) One of the best approaches is to create a safe space for a group to share and discover practices and behaviors that can help them find their own solutions. With DAD, you can help a group choose which problems they wish to solve and which approaches they will take to do so.

  20. 25 Powerful Problem-Solving Questions You Can Use for Any ...

    Asking powerful questions can be a highly effective method for enhancing group discovery and creative problem solving. The right question, asked at the appropriate moment, can transform the ...

  21. What is Problem Solving? (Steps, Techniques, Examples)

    The problem-solving process typically includes the following steps: Identify the issue: Recognize the problem that needs to be solved. Analyze the situation: Examine the issue in depth, gather all relevant information, and consider any limitations or constraints that may be present. Generate potential solutions: Brainstorm a list of possible ...

  22. Different Problem-Solving Styles: What Type of Problem Solver Are You

    It is classic inconsistent-type behavior to lose time dithering between all three ideas, lost in indecision. Instead, write them down in a chart. Then, give each one a score out of 5 according to its strength in whatever categories are relevant to the problem. For example, expense, time, elegance, effort.

  23. Problem Solving Skills Test

    The test requires candidates to identify the right answers to the questions in a limited amount of time. Successful candidates can quickly identify the key elements of the problem and work through the problem at speed without making mistakes. This is a great test to include to check candidates' overall analytical skills. Explore the test science.

  24. How two high school students solved a 2,000-year-old math puzzle

    A high school math teacher at St. Mary's Academy in New Orleans, Michelle Blouin Williams, was looking for ingenuity when she and her colleagues set a school-wide math contest with a challenging ...

  25. Teens come up with trigonometry proof for Pythagorean Theorem, a

    A high school teacher didn't expect a solution when she set a 2,000-year-old Pythagorean Theorem problem in front of her students. Then Calcea Johnson and Ne'Kiya Jackson stepped up to the challenge.